You are on page 1of 230

VIKAS

Handbook
for Simpletons
Book 3
5th Semester

Innovations Unlimited Training Services


Bangalore-560097

CREA – All Rights Reserved I U Training Services


1
Innovations Unlimited Training Services

Published by I U Training Services


th
No. 744, 6 Main, V Block, HMT Layout
Vidyaranyapura, Bangalore – 560 097

www.iutraining.com

First edition published in 2006


Second Edition published in 2008
Third Edition published in 2010
Fourth Edition published in 2011
Fifth edition published in 2015

Printed in India by Padmashree Printers, Bangalore

This is a copy of free course material for distribution among the


students of I U Training Services to be given along the CREA training
programs rendered by I U Training Services for Engineering,
Management and other such programs.

CREA – All Rights Reserved I U Training Services


2
Part I – English Comprehension 5 - 96
1. Reading Comprehension 6

Part II – Logical Ability 56 - 94


1. Analytical Puzzles 58
2. Ready to Get Stumped 95
3. Get Your Figures Right 104
4. Know Thyself 115

Part III – Number Math 125 - 211


1. Improving Your Percentages 126
2. Gain and Loss 142
3. Averages & QnQ of Mixtures 155
4. Permutations and Combinations 170
5. Taking Time to Work on the Distances 190

Part IV – Let’s End the Suspense 212 - 230


1. English Comprehension 213
2. Logical Ability 215
3. Number Math 229

CREA – All Rights Reserved I U Training Services


3
This Page is Intentionally Left
Blank

CREA – All Rights Reserved I U Training Services


4
English
Comprehension

CREA – All Rights Reserved I U Training Services


5
1

Comprehending What You


Read

CREA – All Rights Reserved I U Training Services


6
Reading Comprehension Vocabulary & Comprehension

Please Mrs Butler


Allan Ahlberg

Please Mrs. Butler


This boy Derek Drew
Keeps copying my work, Miss.
What shall I do?
Go and sit in the hall, dear.
Go and sit in the sink.
Take your books on the roof, my lamb.
Do whatever you think.

Please Mrs. Butler


This boy Derek Drew
Keeps taking my rubber, Miss.
What shall I do?
Keep it in your hand, dear.
Hide it up your vest.
Swallow it if you like, my love.
Do what you think best.

Please Mrs. Butler


This boy Derek Drew
Keeps calling me rude names, Miss.
What shall I do?
Lock yourself in the cupboard, dear.
Run away to sea.
Do whatever you can, my flower.
But don't ask me!

CREA – All Rights Reserved I U Training Services


7
Reading Comprehension Vocabulary & Comprehension

In This Chapter
 Learn how to comprehend unseen passages quickly
 Strategies for understanding and answering the questions
under reading comprehension

Reading comprehension questions test your ability to understand what


you read-both content and technique. Each verbal section usually
includes two or more passages of different length, followed by two to
five questions of assorted types. There will be six passages in all. One
passage on the test will be narrative (a passage from a novel, a short
story, a biography, or a personal essay). One passage will deal with the
biological sciences (including medicine, genetics, botany, zoology):
another, usually in a different test section, will deal with the physical
sciences (including art, literature, music, philosophy, folklore); another
on the social sciences (including history, economics, sociology,
government). Finally, one passage will be what the College Board calls
argumentative: it will present a definite point of view on a subject. In
addition, one of these six passages will be ―ethnic‖ in content: whether
it is a history passage, a personal narrative, or a passage on music, art,
or literature, it will deal with the concerns of a particular minority
group. Whatever the subject, the passage will contain all the
information you‘ll need to answer the questions of it.
In group of reading passages, the easiest passages generally come first.
You should find the first passage of a group of four much easier than
the last. However, the questions that come after each passage are not
arranged in order of difficulty. They are arranged to suit the way the
passage‘s content is organized; a question based on information found at
the beginning of the passage will come before a question based on
information at the passage‘s end. If you are stumped by a tough reading
question, don‘t skip the other questions on that passage. A tough
question may be just one question away from an easy one.

CREA – All Rights Reserved I U Training Services


8
Reading Comprehension Vocabulary & Comprehension

This chapter contains three recently-published reading passages


together with several types of questions about them. Some of the
questions are factual, asking you about specific details in the passages.
Others ask you to interpret the passages, to make judgment about
them. Many of these questions are actual questions from the. Others
are new questions, created for this book, but based on the actual
passages. Strategies 1-3 tell you how to deal with reading questions in
general. Strategies 4-11 give you solid hints about how to answer each
type of reading question, plus shorts lists of key words that occur and
reoccur in major question types.
Testing Strategies

Strategy 1: Save the Reading Comprehension


Questions for Last

To answer an antonym question takes you seconds; to answer a reading


comprehension question, you have to spend minutes going over the
passage before you ever get to the questions at all.
On the, you get the same points for answering a ―quick and easy‖
question correctly as you do for answering a time-consuming one. Each
correct answer on the verbal section is worth roughly 10 points to you.
The more questions you answer correctly, the higher your score will be.
Therefore, it makes sense for you to tackle the quick-to-answer-
questions-the antonyms, the analogies, the sentence completions-first.
Get as many of them right as you can, and then settle down to answering
the reading questions. Knowing you‘ve done everything possible to
maximize your score.
One word of caution: remember that in a 45-question verbal section,
the reading questions typically occur in the middle of the section. If you
plan to skip them and come back to them later, be very careful in
marking your answer sheet. Check the numbering of your answer sheet

Strategy 2: Tackle Passages with Familiar


Subjects before Passages with Unfamiliar Ones

CREA – All Rights Reserved I U Training Services


9
Reading Comprehension Vocabulary & Comprehension

often.
Just as it is common sense for you to tackle quick-to-answer questions
before you tackle time-consuming ones, it is also common sense for you
to tackle reading passages with familiar subjects before you tackle
reading passages with unfamiliar ones. If you know very little about
botany or are uninterested in it, you are all too likely to run into trouble
reading a passage about plant life. It is hard to concentrate when you
read about something wholly unfamiliar to you. Give yourself a break.
Concentrate on the reading passages that interest you or that deal with
topics you are well grounded in. there is nothing wrong in skipping
questions. Just remember to check the numbering of your answer sheet.
You should, of course, go back to the questions you skipped if you have
time.

Strategy 3: First Read the Passage; Then Read


the Questions

Students often ask whether it is better to read the passage first or


the questions first. Those who want to read the questions before
reading the passage think it will save time. Ninety-nine times out of a
hundred they are wrong.
Reading the questions before you read the passage will not save you
time. It will cost you time. If you read the questions first, when you
turn to the passage you will have a number of question words and
phrases dancing around in your head. These phrases won‘t focus you;
they‘ll distract you. You will be so involved in trying to spot the places
they occur in the passage that you‘ll be unable to concentrate on
comprehending the passage as a whole. Why increase your anxiety and
decrease your capacity to think? First read the passage, using the
following technique.

1. Read as rapidly as you can with understanding, but do not force


yourself. Do not worry about the time element. If you worry
about not finishing the test, you will begin to take short cuts

CREA – All Rights Reserved I U Training Services


10
Reading Comprehension Vocabulary & Comprehension

and miss the correct answer in your haste. Remember, if you


have followed Tactic-1 and answered the quick questions first,
you‘ve maximized your score and made the best use of your
time.
2. As you read the opening sentenced, try to anticipate what the
passage will be about. Who or what is the author talking about?
3. As you continue reading, try to remember in what part of the
passage the author makes major point. In that way, when you
start looking for the phrase or sentence which will justify your
choice of answer, you will be able to save time by going to that
section of the passage immediately rather than having to
reread the entire selection. (This is particularly important
when the passages become longer than 250 words)
4. Your first reading of the passage should give you a general
impression of the theme of the passage and the location of its
major subdivisions. In order to answer each question properly,
you must go back to the passage to verify your choice of
answer. Double-check yourself; do not rely on your memory.
5. Underline sparingly, if at all. Underlining is great when you‘re
reading a textbook chapter that you want to review at a later
time. On the underlining is not so great. It slows you down. It
also can confuse you. If you underline everything in sight, it
will be harder for you to spot an important word or phrase. A 
or  in the margin is all you need to locate an important point.

If you have any serious reservations about this tactic, feel free to try
alternate approaches doing some of the practice exercises at the end
of this chapter. Compare the scores you get using each different
approach. Reading is a highly individual skill. See what approach works
best for you. The important thing is to know yourself and to feel
comfortable with what you do.

Strategy 4: Learn to Spot the Major


Reading question Types

CREA – All Rights Reserved I U Training Services


11
Reading Comprehension Vocabulary & Comprehension

Just as it will help you to know the directions for the antonym, analogy,
and sentence completion questions on the, it will also help you to
familiarize yourself with the major types of reading questions on the
test.

If you can recognize just what a given question is asking for, you‘ll be
better able to tell which particular reading tactic to apply.
Here are six categories of reading questions you are sure to face.
1. Main Idea questions that test your ability to find the central
thought of a passage or to judge its significance often take the
following form:
 The main point of the passage is to
 The passage is primarily concerned with
 The author‘s primary purpose in this passage is to
 The chief theme of the passage can best be described as
 Which of the following titles best describes the content of
thepassage?
 Which of the following statements best expresses the main
idea of the passage?

2. Finding Specific Details questions that test your ability to


understand what the author states explicitly are often worded:
 According to the author…
 The author states all the following EXCEPT…
 According to the passage, which of the following is true of
the…
 According to the passage, the chief characteristic of the
subject is…
 Which of the following statements is (are) best supported
by the passage?
 Which of the following is NOT cited in the passages as
evidence of?

CREA – All Rights Reserved I U Training Services


12
Reading Comprehension Vocabulary & Comprehension

3. Drawing Inferences questions that test your ability to go


beyond the author‘s explicit statements and see what these
statements imply may be worded:
 It can be inferred from the passage that…
 The passage suggests that the author would support which
of the following views?
 The author implies that…
 According to the passage, it is likely that…
 The passage is most likely directed toward an audience of…
 Which of the following statements about…can be inferred
from the passage?

4. Tone/Attitude questions that test your ability to sense an


author‘s or character‘s emotional state often take the form;
 The author‘s attitude to the problem can best be described
as…
 Which of the following best describes the author‘s tone in
the passage?
 The author‘s tone in the passage is that of a person
attempting to…
 The author‘s presentation is marked by a tone of…
 The passage indicates that the author experience a feeling
of…

5. Determining the Meaning of Words from Their Context


questions that test your ability to work out the meaning of
unfamiliar words from their context often are worded:
 As it is used in the passage, the term…..can best be
described as…
 The phrase…is used in the passage to mean that…
 In the passage, the word___________means…
 The author uses the phrase_________, to describe…

Strategy 5: When Asked To Find the Main Idea, Be


Sure to Check the Opening and Summary Sentences
of Each Paragraph

CREA – All Rights Reserved I U Training Services


13
Reading Comprehension Vocabulary & Comprehension

The opening and closing sentences of a paragraph are key sentences for
you to read. They can serve as guideposts for you, pointing out the
author‘s main idea.
Whenever you are asked to determine a passage‘s main idea, always
check the opening and summary sentences of each paragraph. Authors
typically provide readers with a sentence which expresses a paragraph‘s
main idea succinctly. Although such topic sentences may appear
anywhere in the paragraph, readers customarily look for them in the
opening or closing sentences.
Notice the impact of words like again, also, as well as, furthermore,
moreover and significantly in the passage. These signal words may call
your attention to the main idea.
If, however you are unable to find a topic sentence, ask yourself these
questions:
1. Who or what is this passage about?
2. What aspect of this subject is the author trying to get across
about this aspect of the subject?
3. What is the author trying to get across about this aspect of
the subject?

Strategy 6: Familiarize yourself with the


Technical Terms Used to Describe a passage‘s
Organization Sentences of Each Paragraph

Another part of understanding the author‘s point is, understanding how


the author organizes what he or she has to say. To do so, often you have
to figure out how the opening sentence or paragraph is connected to the
passage as a whole.

Strategy 7: When Asked To Choose a Title, Watch


Out for Choices that are too specific or too Broad

CREA – All Rights Reserved I U Training Services


14
Reading Comprehension Vocabulary & Comprehension

A paragraph has been defined as a group of sentences revolving around


a central theme. An appropriate title for a paragraph, therefore, must
include this central theme that each of the sentences in the paragraph
is developing. It should be neither too broad nor too narrow in its scope;
it should be specific and yet comprehensive enough to include all the
essential ideas presented by the sentences. A good title for a passage
of two or more paragraphs should include all the thoughts of ALL the
paragraphs.

Strategy 7: When Asked About Specific Details in


the Passage, Spot Key Words in the Question and
Scan the Passage to Find Them (or Their Synonyms)

In developing the main idea of a passage, a writer will make statements


to support his or her point. To answer questions about such supporting
details, you must find a word or a group of words in the passage which
supports your choice of answer. The words ―according to the passage‖
or ―according to the author‖ should focus your attention on what the
passage explicitly states. Do not be misled into choosing an answer (even
one that makes good sense) if you cannot find it supported in the text.

Brain Drill - 1
Each passage below is followed by questions based on its content.
Answer all questions following a passage on the basis of what is stated
or implied in that passage.

Questions 1 to 5

The best Eskimo carvings of all ages seem to possess a powerful ability
to reach across the great barriers of language and time and
communicate directly with us. The more we look at these carvings, the

CREA – All Rights Reserved I U Training Services


15
Reading Comprehension Vocabulary & Comprehension

more life we perceive hidden within them. We discover subtle living


forms of the animal, human, and mystical world. These arctic carvings
are not the cold sculptures of a frozen world. Instead, they reveal to us
the passionate feelings of a vital people well aware of all the joys,
terrors, tranquility, and wildness of life around them.

Eskimo carvers are people moved by dreams. In spite of all


their new contacts with outsiders, they are still concerned with their
own kind of mystical imagery. The most skillful carvers possess a bold
confidence, a direct approach to their art that has a special freedom
unsullied by any kind of formalized training.

Eskimo carvers have strong skilled hands, used to forcing hard


materials with their simple tools. Their hunting life and the northern
environment invigorates them. Bad weather often imposes a special kind
of leisure, giving them time in which to perfect their carvings .

They are among the last of the hunting societies that have
retained some part of the keen sense of observation that we have so
long forgotten. The carvers are also butchers of meat, and therefore
masters in the understanding of animal anatomy. Flesh and bones and
sheaths of muscle seem to move in their works. They show us how to
drive the caribou, how to hold a child, how to walk cautiously on thin ice.
Through their eyes we understand the dangerous power of a polar bear.
In the very best of Eskimo art we see vibrant animal and human forms
that stand quietly or tensely, strongly radiating a sense of life. We can
see, and even feel with our hands, the cold sleekness of seals, the
hulking weight of walrus, the icy swiftness of trout, the flowing rhythm
in a flight of geese. In their art we catch brief glimpses of a people who
have long possessed a very different approach to the whole question of
life and death.

In Eskimo art there is much evidence of humour which the


carvers have in abundance. Some of the carvings are caricatures of
themselves, of ourselves, and of situations, or records of ancient
legends. Their laughter may be subtle, or broad and Chaucerian.

CREA – All Rights Reserved I U Training Services


16
Reading Comprehension Vocabulary & Comprehension

Perhaps no one can accurately define the right way or wrong


way to create a carving. Each carver must follow his own way, in his own
time. Technique in itself is meaningless unless it serves to express
content. According to the Eskimo, the best carvings possess a sense of
movement that seems to come from within the material itself, a feeling
of tension, a living excitement.

1. The author is primarily concerned with

(A) showing how Eskimo carvings achieve their effects


(B) describing how Eskimo artists resist the influence of outsiders
(C) discussing the significant characteristics of Eskimo art
(D) explaining how Eskimo carvers use their strength to manipulate hard
materials
(E) interpreting the symbolism of Eskimo art

2. The author‘s attitude toward Eskimo art is one of

(A) condescension (B) awe (C) admiration


(D) regret (E) bewilderment

3. With which of the following statements would the author most likely
agree?

(A) training may often destroy an artist‘s originality.


(B) Artists should learn their craft by studying the works of experts.
(C) The content of a work of art is insignificant.
(D) Caricatures have no place in serious art.
(E) Eskimo art is interesting more as an expression of a life view than as
a serious art form.

4. The author gives examples of the subjects of Eskimo carvings


primarily to

(A) show that they have no relevance to modern life


(B) indicate the artist‘s lack of imagination
(C) imply that other artists have imitated them
(D) prove that the artists‘ limited experience of life has been a
handicap
(E) suggest the quality and variety of the work

CREA – All Rights Reserved I U Training Services


17
Reading Comprehension Vocabulary & Comprehension

5. According to the passage, Eskimo carvings have all the following


characteristics EXCEPT

(A) wit (B) subtlety (C) passion (D) formality


(E) terror

Questions 6 and 7

The whole atmosphere of the world in which we live is tinged by science,


as is shown most immediately and strikingly by our modern conveniences
and material resources. A little deeper thinking shows that the
influence of science goes much farther and colors the entire mental
outlook of modern civilized man on the world about him. Perhaps one of
the most telling evidences of this is his growing freedom from
superstition. Freedom from superstition is the result of the conviction
that the world is not governed by caprice, but that it is a world of order
and can be understood by man if he will only try hard enough and be
clever enough. This conviction that the world is understandable is,
doubtless, the most important single gift of science to civilization. The
widespread acceptance of this view can be dated to the discovery by
Newton of the universal sway of the law of gravitation; and for this
reason Newton may be justly regarded as the most important single
contributor to modern life.

6. Which of the following best expresses the author‘s purpose in writing


this passage?

(A) To give an opinion of the chief benefit of science


(B) To enumerate the modem conveniences produced by science
(C) To inform readers about important scientific principles
(D) To describe Newton‘s discoveries
(E) To persuade readers that science leads to civilization

7. According to the author, Newton‘s chief contribution was his

(A) encouragement of intelligent inquiry


(B) great book on the laws of gravity
(C) understanding of a world based on order
(D) early development of scientific methodology

CREA – All Rights Reserved I U Training Services


18
Reading Comprehension Vocabulary & Comprehension

(E) many practical inventions that have given rise to modern


conveniences

Questions 8 to 11

―But perhaps you are telling lies?‖ Raskolnikov put in.

‗I rarely lie,‖ answered Svidrigaїlov thoughtfully, apparently not noticing


the rudeness of the question.

―And in the past, have you ever seen ghosts before?‖

―Y-yes, I have seen them, but only once in my life, six years ago. I had a
serf, Filka; just after his burial I called out forgetting ‗Filka, my pipe!‘
He came in and went to the cupboard where my pipes were. I sat still
and thought ‗he is doing it out of revenge,‘ because we had a violent
quarrel just before his death. ‗How dare you come in with a hole in your
elbow,‘ I said. ‗Go away, you scamp!‘ He turned and went out, and never
came again. I didn‘t tell Marfa Petrovna at the time. I wanted to have a
service sung for him, but I was ashamed.‖

―You should go to a doctor.‖

―I know I am not well, without your telling me. Though I don‘t know
what‘s wrong; I believe I am five times as strong as you are. I didn‘t ask
you whether you believe that ghosts are seen, but whether you believe
that they exist.‖

― No, I won‘t believe it!‖ Raskolnikov cried, with positive anger.

―What do people generally say?‖ muttered Svidrigaїlov, as though


speaking to himself, looking aside and bowing his head: ―They say, ‗You
are ill, so what appears to you is only unreal fantasy.‘ But that‘s not
strictly logical. I agree that ghosts only appear to the sick, but that
only proves that they are unable to appear except to the sick, not that
they don‘t exist.‖

―Nothing of the sort,‖ Raskolnikov insisted irritably.

―No? You don‘t think so?‖ Svidrigaїlov went on, looking at him
deliberately. ―But what do you say to this argument (help me with it):

CREA – All Rights Reserved I U Training Services


19
Reading Comprehension Vocabulary & Comprehension

ghosts are as it were shreds and fragments of other worlds, the


beginning of them. A man in health has, of course, no reason to see
them, because he is above all a man of this earth and is bound for the
sake of completeness and order to live only in this life. But as soon as
one is ill, as soon as the normal earthly order of the organism is broken,
one begins to realise the possibility of another world; and the more
seriously ill one is, the closer becomes one‘s contact with that other
world, so that as soon as the man dies he steps straight into that world.
I thought of that long ago. If you believe in a future life, you could
believe in that, too.‖

8. The passage indicates that Raskolnikov thinks Svidrigaїlov‘s belief in


ghosts is

(A) justified by the experiences he relates


(B) a symptom of some illness
(C) totally inexplicable
(D) the result of a dream
(E) a fit subject for scientific experiment

9. Raskolnikov‘s anger suggests that

(A) he is bored with Svidrigaїlov‘s conversation


(B) he is eager to recount an experience of his own
(C) he is unaccustomed to polite conversation
(D) he has some personal distaste for the subject
(E) Svidrigaїlov‘s theories would annoy anyone

10. According to Svidrigaїlov, most people do not see ghosts because


they

(A) are unworthy of seeing them


(B) are too concerned with worldly affairs
(C) have been taught that ghosts do not exist
(D) do not know anyone who has died
(E) do not know how to summon ghosts

11. Which of the following definitions of ―ghost‘ ‗ is closest to


Svidrigaїlov‘s view as presented in the passage?

CREA – All Rights Reserved I U Training Services


20
Reading Comprehension Vocabulary & Comprehension

(A) A ghost is a visitor from another world.


(B) A ghost is a symbol of evil.
(C) A ghost is a product of the imagination.
(D) A ghost is a messenger of doom.
(E) A ghost is a lost spirit.

Questions 12 to 15

The passage below was written by Samuel Taylor Coleridge.

During the first year that Mr. Wordsworth and I were


neighbours, our conversations turned frequently on the two cardinal
points of poetry, the power of exciting the sympathy of the reader by a
faithful adherence to the truth of nature, and the power of giving the
interest of novelty by the modifying colours of imagination. The sudden
charm, which accidents of light and shade, which moonlight or sunset
diffused over a known and familiar landscape, appeared to represent the
practicability of combining both. These are the poetry of nature. The
thought suggested itself— (to which of us I do not recollect)—that a
series of poems might be composed of two sorts. In the one, the
incidents and agents were to be, in part at least, supernatural; and the
excellence aimed at was to consist in the interesting of the affections
by the dramatic truth of such emotions, as would naturally accompany
such situations, supposing them real. And real in this sense they have
been to every human being who, from whatever source of delusion, has
at any time believed himself under supernatural agency. For the second
class, subjects were to be chosen from ordinary life: the characters and
incidents were to be such as will be found in every village and its
vicinity, where there is a meditative and feeling mind to seek after
them, or to notice them, when they present themselves.

In this idea originated the plan of the LYRICAL BALLADS; in


which it was agreed, that my endeavours should be directed to persons
and characters supernatural, or at least romantic; yet so as to transfer
from our inward nature a human interest and a semblance of truth
sufficient to procure for these shadows of imagination that willing
suspension of disbelief for the moment, which constitutes poetic faith.
Mr. Wordsworth, on the other hand, was to propose to himself as his

CREA – All Rights Reserved I U Training Services


21
Reading Comprehension Vocabulary & Comprehension

object, to give the charm of novelty to things of every day, and to


excite a feeling analogous to the supernatural, by awakening the mind‘s
attention to the lethargy of custom, and directing it to the loveliness
and the wonders of the world before us; an inexhaustible treasure, but
for which, in consequence of the film of familiarity and selfish
solicitude, we have eyes, yet see not, ears that hear not, and hearts
that neither feel nor understand.

12. Which of the following is the best title for this passage?

(A) Suspension of Disbelief


(B) A Great Poetic Collaboration
(C) Adherence to Nature
(D) Two Great Minds
(E) Wordsworth and Coleridge

13. The two poets differed in their

(A) ideas of the two main points of poetry


(B) desire to write a book of poems
(C) choice of subject matter
(D) plans for the organization of Lyrical Ballads
(E) evaluation of the merits of their poems

14. From the author‘s discussion of the supernatural, which of the


following can be inferred?

(A) He believes that the supernatural is real.


(B) He does not believe that supernatural agencies exist.
(C) He thinks supernatural events have no place in poetry.
(D) He feels readers will doubt the truth of supernatural events.
(E) He thinks the supernatural is boring.

15. The last three lines of the passage can best be restated as

(A) beauty is in the eye of the beholder


(B) people spend too much time alone
(C) only a poet can appreciate nature
(D) familiarity breeds contempt
(E) out of sight, out of mind

CREA – All Rights Reserved I U Training Services


22
Reading Comprehension Vocabulary & Comprehension

Questions 16 to 20
In the warm enclosed waters of farm ponds, conditions are very likely
to be lethal for fish when insecticides are applied in the vicinity. As
many examples show, the poison is carried in by rains and runoff from
surrounding lands. Sometimes the ponds receive not only contaminated
runoff but also a direct dose as crop-dusting pilots neglect to shut off
the duster in passing over a pond. Even without such complications,
normal agricultural use subjects fish to far heavier concentrations of
chemicals than would be required to kill them. In other words, a marked
reduction in the poundages used would hardly alter the lethal situation,
for applications of over 0. 1 pound per acre to the pond itself are
generally considered hazardous. And the poison, once introduced is hard
to get rid of. One pond that had been treated with DDT to remove
unwanted shiners remained so poisonous through repeated drainings and
flushings that it killed 94 percent of the sunfish with which it was later
stocked. Apparently the chemical remained in the mud of the pond
bottom.

In some parts of the world the cultivation of fish in ponds provides an


indispensable source of food. In such places the use of insecticides
without regard for the effects on fish creates immediate problems. In
Rhodesia, for example, the young of an important food fish, the Kafue
bream, are killed by exposure to only 0.04 parts per million of DDT in
shallow pools. Even smaller doses of many other insecticides would be
lethal. The shallow waters in which these fish live are favorable
mosquito-breeding places. The problem of controlling mosquitoes and at
the same time conserving a fish important in the Central African diet
has obviously not been solved satisfactorily.

16. Which of the following would be the best title for this passage?

(A) The Water of Farm Ponds


(B) The Effects of Insecticides on Fish
(C) The Uses of DDT
(D) The Cultivation of Fish as Food
(E) Saving African Food Supplies

17. The author‘s tone in this passage can best be described as

CREA – All Rights Reserved I U Training Services


23
Reading Comprehension Vocabulary & Comprehension

(A) reportorial (B) sarcastic (C) angry


(D) condemnatory (E) mournful

18. According to the passage, which of the following are responsible for
the presence of insecticides in ponds?

I. The weather II. Human error


III. Common farming methods

(A) I only (B) III only (C) I and II only


(D) I and III only (E) I, II, and III

19. The author uses the case of the Rhodesian fish primarily in order to

(A) the harmful effects of killing fish


(B) prove that problems are the same everywhere
(C) evaluate African farming practices
(D) consider the problem of controlling mosquitoes
(E) contrast African and American insecticide use

20. In this passage, the author does all of the following EXCEPT

(A) state a problem (B) propose a solution


(C) give examples (D) relate causes
(E) state effects

Questions 21 to 25
When a new movement in Art attains a certain vogue, it is
advisable to find out what its advocates are aiming at, for however
farfetched and unreasonable their tenets may seem today, it is possible
that in years to come they may be regarded as normal. With regard to
Futurist poetry, however, the case is rather different; for whatever
Futurist poetry may be—even admitting that the theory on which it is
based may be right—it can hardly be classed as Literature. (7)

This, in brief, is what the Futurist says: for a century past (8)
conditions of life have been continually speeding up, till now we live in a
world of noise and violence and speed. Consequently, our feelings,
thoughts and emotions have undergone a corresponding change.(11) This
speeding up of life, says the Futurist requires a new form of expression.

CREA – All Rights Reserved I U Training Services


24
Reading Comprehension Vocabulary & Comprehension

We must speed up our literature too, if we want to interpret modern


stress. We must pour out a cataract of essential words, unhampered by
stops, or qualifying adjectives, or (15) finite verbs. Instead of
describing sounds we must make up words that imitate them; we must
use many sizes of type and different colored inks on the same page, and
shorten or lengthen words at will. Certainly their descriptions of battles
are vividly chaotic. But it is a little disconcerting to read in the
explanatory notes that a certain line describes a fight between a
Turkish and a Bulgarian officer on a bridge over which they both fall
into the river—and then to find that the line consists of the noise of
their falling and the weights of the officers: ―Pluff! Pluff! a hundred
and eighty-five (24) kilograms.‖ This, though it fulfils the laws and
requirements of Futurist poetry, can hardly be classed as Literature. All
the same, no thinking man can refuse to accept their first proposition:
that a great change in our emotional life calls for a change of
expression. The whole question is really this: have we essentially
changed? (29)

21 . The main idea of this selection is best expressed as

(A) the Past versus the Future


(B) changes in modern life
(C) merits of the Futurist movement
(D) what constitutes literature
(E) an evaluation of Futurist poetry

22. When novel ideas appear, it is desirable, according to the writer, to

(A) discover the goals of their adherents


(B) ignore them entirely
(C) follow the fashion of the moment
(D) regard them as normal until proven otherwise
(E) adopt them slowly

23. According to the passage, the Futurist poet advocates using all of
the following devices EXCEPT

CREA – All Rights Reserved I U Training Services


25
Reading Comprehension Vocabulary & Comprehension

(A) imitative sounds (B) modifying adjectives


(C) inks of various hues (D) a flood of nouns
(E) lengthened words

24. It can be inferred that the author quotes a Futurist poem (lines 8 -
11) to

(A) emphasize the importance of good literature


(B) indicate that Futurists describe battles vividly
(C) show that it is in keeping with the speed of , modern life
(D) suggest that it is not great poetry
(E) identify the source of poetic imagination

25. The last two sentences in the passage (lines 24 — 29) chiefly
suggest which of the following?

(A) A new type of writing is needed in the modern world.


(B) Futurist theories are correct.
(C) Traditional literature still expresses human emotions.
(D) When people change, they will be better able to appreciate poetry.
(E) No literature can fully express human emotions.

Brain Drill - 2
Each passage below is followed by questions based on its content.
Answer all questions following a passage on the basis of what is stated
or implied in that passage.

Questions 1 to 5

Great mural painting begins in Mexico after the Revolution. Having


already given proof of his artistic ability in the delicate lines of pre-
Spanish reliefs and in the religious pictures of the colonial period, the
Mexican was ready for the notable achievements of the mural painting
of today. A wave of social fervor, of passionate convictions, and of
beauty animates the forms of the great Mexican frescoes. On Mexican
walls were written the life of the people and the history of the nation.

CREA – All Rights Reserved I U Training Services


26
Reading Comprehension Vocabulary & Comprehension

This painting was the perfect union of a strong art and a living thought.
Diego Rivera drew the life and the history of the country with a
richness of composition, a formal harmony, and a sense of mass and
space that no one has surpassed in our time. José Clemente Orozco,
penetrating yet deeper, painted with a generous cruelty and a rough
tenderness the bold and broken symbols of contemporary wickedness,
truth everlasting, and innate tragedy. With these men, there came a
whole new generation of painters. Mexican mural painting of the
twentieth century is not only Mexico‘s greatest contribution to the art
of our time but one of the most vigorous and original contemporary
esthetic manifestations.

1. The passage primarily concerns the

(A) influence of Mexican art on the rest of the world


(B) art of Mexican wall paintings
(C) paintings of Diego Rivera and José Clemente Orozco
(D) life and history of the Mexican people
(E) history of Mexican art

2. The word ‗ ‗frescoes‖ means

(A) beautiful paintings (B) Mexican art


(C) wall paintings (D) convictions (E) modern art

3. The author‘s attitude toward the art of Diego Rivera and José
Clemente Orozco is

(A) sarcastic (B) condescending (C) wondering


(D) exultant (E) approving

4. Based on the information in the passage, an artist inspired by the


Mexican muralists would most likely produce which of the following?

(A) A painting of a bowl of fruit


(B) An abstract pattern of stripes
(C) ―A panorama of historic figures‖
(D) A portrait of the artist‘s family
(E) A realistic study of a landscape

CREA – All Rights Reserved I U Training Services


27
Reading Comprehension Vocabulary & Comprehension

5. With which of the following statements would the author most likely
agree?

(A) The subject matter of a painting is unimportant.


(B) The twentieth-century muralists were the first great artists in
Mexico.
(C) Diego Rivera was a better artist than José Clemente Orozco.
(D) Composition and harmony are important elements in painting.
(E) Mural painting is Mexico‘s only contribution to the art of our time.

Questions 6 to 8

Geometry is a very old science. We are told by Herodotus, a Greek


historian, that geometry had its origin in Egypt along the banks of the
river Nile. The first record we have of its study is found in a manuscript
written by Ahmes, an Egyptian scholar, about 1550 B.C. This manuscript
is believed to be a copy of a treatise which dated back probably more
than a thousand years, and describes the use of geometry at that time
in a very crude form of surveying or measurement. In fact, geometry,
which means ―earth measurement,‖ received its name in this manner.
This re of the land was necessary due to the annual overflow of the
river Nile and the consequent destroying of the boundaries of farm
lands. This early geometry was very largely a list of rules or formulas
for finding the areas of plane figures. Many of these rules were
inaccurate, but, in the main, they were fairly satisfactory.

6. The title below that best expresses the ideas of this passage is

(A) Plane Figures (B) Beginnings of Geometry


(C) Manuscript of Ahmes (D) Surveying in Egypt
(E) Importance of the Study of Geometry

7. According to the passage, in developing geometry the early Egyptians


were primarily concerned with

(A) discovering why formulas used in measuring were true


(B) determining property boundaries

CREA – All Rights Reserved I U Training Services


28
Reading Comprehension Vocabulary & Comprehension

(C) measuring the overflow of the Nile


(D) generalizing formulas
(E) constructing a logical system of geometry

8. It can be inferred that one of the most important factors in the


development of geometry as a science was

(A) Ahmes‘ treatise


(B) the inaccuracy of the early rules and formulas
(C) the annual flooding of the Nile Valley
(D) the destruction of farm crops by the Nile
(E) an ancient manuscript copied by Ahmes

Questions 9 to 13

The single business of Henry Thoreau, during forty- odd years of eager
activity, was to discover an economy calculated to provide a satisfying
life. His one concern, that gave to his ramblings in Concord fields a value
of high adventure, was to explore the true meaning of wealth. As he
understood the problem of economics, there were three possible
solutions open to him: to exploit himself, to exploit his fellows, or to
reduce the problem to its lowest denominator. The first was quite
impossible—to imprison oneself in a treadmill when the morning called to
great adventure. To exploit one‘s fellows seemed to Thoreau‘s sensitive
social conscience an even greater infidelity. Freedom with abstinence
seemed to him better than serfdom with material well-being, and he was
content to move to Walden Pond and so set about the high business of
living. ―To front only the essential facts of life and to see what it had to
teach.‖ He did not advocate that other men should build cabins and live
isolated. He had no wish to dogmatize concerning the best mode of
living—each must settle that for himself. But that a satisfying life
should be lived. He was vitally concerned. The story of his emancipation
from the lower economics is the one romance of his life, and Walden is
his great book. It is a book in praise of life rather than of Nature, a
record of calculating economies that studied saving in order to spend
more largely. But it is a book of social criticism as well, in spite of its
explicit denial of such a purpose. In considering the true nature of
economy he concluded, with Ruskin, that the cost of a thing is the

CREA – All Rights Reserved I U Training Services


29
Reading Comprehension Vocabulary & Comprehension

amount of life which is required in exchange for it, immediately or in


the long run. In Walden Thoreau elaborated the text: ―The only wealth
is life.‖

9. The author‘s primary purpose in this passage is to

(A) discuss and assess economic problems


(B) describe Thoreau‘s philosophy of life
(C) prove that Walden was Thoreau‘s greatest book
(D) show how Thoreau was able to live in isolation
(E) reevaluate life at Walden Pond

10. On the basis of the passage, Thoreau was all of the following
EXCEPT

(A) dogmatic (B) liberated (C) energetic


(D) self-denying (E) critical

11. It can be inferred that the author thinks of Thoreau‘s ―emancipation


from the lower economics‖ as a romance because it

(A) wholly captured Thoreau‘s imagination


(B) involved him in the love of a woman
(C) entailed social criticism
(D) was an adventure story
(E) was embodied in Thoreau‘s greatest book

12. On the basis of the passage, it can be inferred that Thoreau


believed the wealth of an individual is measured by

(A) the money he or she makes


(B) the experience he or she gains
(C) the amount he or she saves
(D) his or her good deeds
(E) his or her social standing in the community

13. The author‘s tone in speaking of Thoreau is

(A) ironic (B) critical (C) indifferent


(D) admiring (E) effusive

CREA – All Rights Reserved I U Training Services


30
Reading Comprehension Vocabulary & Comprehension

Questions 14 to 18

Newman promised himself to pay Mademoiselle Noémie another visit at


the Louvre. He was curious about the progress of his copies, but it must
be added that he was still more curious about the progress of the young
lady herself. He went one afternoon to the great museum, and wandered
through several of the rooms in fruitless quest of her. He was ending
his steps to the long hall of the Italian masters, when suddenly he found
himself face to face with Valentin de Bellegarde. The young Frenchman
greeted him with ardor, and assured him that he was a godsend. He
himself was in the worst of humors and he wanted someone to
contradict. ―In a bad humor among all these beautiful things?‖ said
Newman. ―I thought you were so fond of pictures, especially the old
black ones. There are two or three here that ought to keep you in
spirits.‖ ―Oh, to-day,‖ answered Valentin, ―I am not in a mood for
pictures, and the more beautiful they are the less I like them. Their
great staring eyes and fixed positions irritate me. I feel as if I were at
some big, dull party, in a room full of people I shouldn‘t wish to speak to.
What should I care for their beauty? It‘s a bore, and, worse still, it‘s a
reproach. I have a great many ennuis; I feel vicious.‖

―If the Louvre has so little comfort for you, why in the world did you
come here?‖ Newman asked.

―That is one of my ennuis. I came to meet my cousin—a dreadful English


cousin, a member of my mother‘s family—who is in Paris for a week with
her husband, and who wishes me to point out the ‗principal beauties.‘
Imagine a woman who wears a green crape bonnet in December and has
straps sticking out of the ankles of her interminable boots! My mother
begged I would do something to oblige them. I have undertaken to play
valet de place this afternoon. They were to have met me here at two
o‘clock, and I have been waiting for them twenty minutes. Why doesn‘t
she arrive? She has at least a pair of feet to carry her. I don‘t know
whether to be furious at their playing me false, or delighted to have
escaped them.‖

―I think in your place I would be furious,‖ said Newman, ―because they


may arrive yet, and then your fury will still be of use to you. Whereas, if

CREA – All Rights Reserved I U Training Services


31
Reading Comprehension Vocabulary & Comprehension

you were delighted and they were afterwards to turn up, you might not
know what to do with your delight.‖

―You give me excellent advice, and I already feel better. I will be


furious; I will let them go to the deuce and I myself will go with you—
unless by chance you too have a rendezvous.‖

14. The passage indicates that Newman has gone to the Louvre in order
to

(A) meet Valentin (B) look at the paintings


(C) explore Paris (D) keep an appointment
(E) see Mademoiselle Noémie

15. According to the passage, Valentin is unhappy about being at the


Louvre because he

(A) hates the paintings of the Italian masters


(B) has accidentally met Newman in the long hall
(C) wishes to be at a party
(D) feels that beauty should be that of nature
(E) is supposed to guide his cousin through it

16. It can be inferred from the passage that Valentin is expressing his
annoyance by

(A) walking out of the Louvre in a fit of temper


(B) making insulting remarks about a woman
(C) not accepting Newman‘s advice
(D) criticizing the paintings
(E) refusing to do as his mother wishes

17. With which of the following statements would Valentin most likely
agree?

I. Clothes make the man. II. Blood is thicker than water.

III. Better late than never.

(A) I only (B) II only (C) III only


(D) I and II only (E) I, II, and III

CREA – All Rights Reserved I U Training Services


32
Reading Comprehension Vocabulary & Comprehension

18. Newman‘s role in the conversation is that of


(A) a heckler (B) a gossiper (C) a confidant (D) an enemy (E) a doubter

Questions 19 and 20

Both plants and animals of many sorts show remarkable changes in form,
structure, growth habits, and even mode of reproduction in becoming
adapted to different climatic (3) environments, types of food supply, or
modes of living. This divergence in response to evolution. is commonly
expressed by altering the form and function of some part or parts of
the organism, the original identity of which is clearly (6) discernible. For
example, the creeping foot of the snail is seen in related marine
pteropods to be modified into a flapping organ useful for swimming, and
is changed into prehensile arms that bear suctorial disks in the squids
and other (10) cephalopods. The limbs of various mammals are modified
according to several different modes of life—for swift running
(cursorial) as in the horse and antelope, for swinging in trees (arboreal)
as in the monkeys, for digging (fossorial) as in the (14) moles and
gophers, for flying (volant) as in the bats, for swimming (aquatic) as in
the seals, whales and dolphins, and for other adaptations. The
structures or organs that show the main change in connection with this
adaptive (18) divergence are commonly identified readily as homologous,
in spite of great alterations. Thus, the finger and wristbones of a bat
and whale, for instance, have virtually nothing in common ex— cept that
they are definitely equivalent elements (21) of the mammalian limb.

19. The primary purpose of the passage is to

(A) describe a phenomenon (B) analyze a method


(C) support a theory (D) refute a mistaken notion
(E) speculate about causes

20. The word ―homologous‖ (line 18) means

(A) having the same origin (B) used for the same purpose
(C) identical (D) dissimilar (E) greatly altered

Questions 21 to 25

CREA – All Rights Reserved I U Training Services


33
Reading Comprehension Vocabulary & Comprehension

Music and literature, the two temporal arts, contrive their pattern of
sounds in time; or, in other words, of sounds and pauses. Communication
may be made in broken words, the business of life be carried on with
substantives alone; but that is not what we call literature; and the true
business of the literary artist is to plait or weave his meaning, involving
it around itself; so that each sentence, by successive phrases, shall
first come into a kind of knot, and then, after a moment of suspended
meaning, solve and clear itself. In every properly constructed sentence
there should be observed this knot or hitch; so that (however
delicately) we are led to foresee, to expect, and then to welcome the
successive phrases. The pleasure may be heightened by an element of
surprise, as, very grossly, in the common figure of the antithesis, or,
with much greater subtlety, where an antithesis is first suggested and
then deftly evaded. Each phrase, besides, is to be comely in itself; and
between the implication and the evolution of the sentence there should
be a satisfying equipoise of sound; for nothing more often disappoints
the ear than a sentence solemnly and sonorously prepared, and hastily
and weakly finished. Nor should the balance be too striking and exact,
for the one rule is to be infinitely various; to interest, to disappoint, to
surprise, and yet still to gratify; to be ever changing, as it were, the
stitch, and yet still to give the effect of an ingenious neatness.

21 . According to the author, great literature depends on its

(A) clarity of thought (B) subtleties of sound


(C) sense of logic (D) preciseness of balance
(E) use of broad contrasts

22. The author calls music and literature ―the two temporal arts‖
because

(A) they are impermanent by nature


(B) they are both used in temples
(C) they are both based on antithesis
(D) they are composed of periods of sound and silence
(E) they both depend on balance

CREA – All Rights Reserved I U Training Services


34
Reading Comprehension Vocabulary & Comprehension

23. The author implies that the function of the writer is to

(A) communicate with the reader


(B) carry on the business of life
(C) move the reader to action
(D) present ideas attractively
(E) capture balance and antithesis

24. The author believes that antithesis is a means of securing

(A) surprise (B) subtlety (C) comeliness


(D) balance (E) solemnity

25. According to the passage, writers should do all of the following


EXCEPT

(A) lead readers to anticipate what is coming next


(B) surprise readers
(C) disappoint readers
(D) construct sentences carefully
(E) balance sounds exactly

Brain Drill - 3
Questions 1 to 5

When you first saw a piece of African art, it impressed you as a unit;
you did not see it as a collection of shapes or forms. This, of course,
means that the shapes and volumes within the sculpture itself were
coordinated so successfully that the viewer was affected emotionally.
It is entirely valid to ask how, from a purely artistic point of view, this
unity was achieved. And we must also inquire whether there is a
recurrent pattern or rules or a plastic language and vocabulary which is
responsible for the powerful communication of emotion which the best
African sculpture achieves. If there is such a pattern or rules, are
these rules applied consciously or instinctively to obtain so many works
of such high artistic quality?

CREA – All Rights Reserved I U Training Services


35
Reading Comprehension Vocabulary & Comprehension

It is obvious from the study of art history that its content. Answer all
questions following a passage on the an intense and unified emotional
experience, such as the Christian credo of the Byzantine or 12th or
13th century Europe, when expressed in art forms, gave great unity,
coherence, and power to art. But such an integrated feeling was only the
inspirational element for the artist, only the starting point of the
creative act. The expression of this emotion and its realization in the
work could be done only with discipline and thorough knowledge of the
craft. And the African sculptor was a highly trained workman. He
started his apprenticeship with a master when a child and he learned
the tribal styles and the use of tools and the nature of woods so
thoroughly that his carving became what Boas calls ―motor action.‖ He
carved automatically and instinctively. The African carver followed his
rules without thinking of them; indeed, they never seem to have been
formulated in words. But such rules existed, for accident and
coincidence cannot explain the common plastic language of African
sculpture. There is too great a consistency from one work to another.
Yet, although the African, with amazing insight into art, used these
rules, I am certain that he was not conscious of them. This is the great
mystery of such a traditional art; talent, or the ability certain people
have, without conscious effort, to follow the rules which later the
analyst can discover only from the work of art which has already been
created.

1. The author is primarily concerned with

(A) discussing how African sculptors achieved their effects


(B) listing the rules followed in African art
(C) relating African art to the art of 12th or 13th century Europe
(D) integrating emotion and realization
(E) expressing the beauty of African art

2. According to the passage, one of the outstanding features of African


sculpture is

(A) its subject matter (B) the feelings it arouses


(C) the training of the artists (D) its strangeness
(E) its emphasis on movement

CREA – All Rights Reserved I U Training Services


36
Reading Comprehension Vocabulary & Comprehension

3. The word ―plastic‖ in means

(A) synthetic (B) linguistic (C) consistent


(D) sculptural (E) repetitive

4. According to the information in the passage, an African carver can


best be compared to a

(A) chef following a recipe


(B) fluent speaker of English who is just beginning to study French
(C) batter who hits a homerun in his or her first baseball game
(D) concert pianist performing a well-rehearsed concerto
(E) writer who is grammatically expert but stylistically uncreative

5. Which of the following titles best summarizes the content of the


passage?

(A) The Apprenticeship of the African Sculptor


(B) The History of African Sculpture
(C) How African Art Achieved Unity
(D) Analyzing African Art
(E) The Unconscious Rules of African Art

Questions 6 to 8

One of the most urgent problems in teaching handwriting is presented


by the left-handed child. The traditional policy has been to attempt to
induce all children to write with their right hands. Parents and teachers
alike have an antipathy to the child‘s using his left hand. On the other
hand, psychologists have shown beyond a doubt that some persons are
naturally left-handed and that it is much more difficult for them to do
any skillful act with the right hand than with the left hand. Some
believe, furthermore, that to compel a left-handed child to write with
his right hand may make him nervous and may cause stammering. There
seems to be some cases in which this is true, although in the vast
majority of children who change over, no ill effects are noticed. In
addition to these difficulties, left-handedness sometimes seems to
cause mirror writing—writing from right to left—and reversals in
reading, as reading ―was‘ ‗ for ―saw.‖

CREA – All Rights Reserved I U Training Services


37
Reading Comprehension Vocabulary & Comprehension

6. The title below that best expresses the ideas of this passage is

(A) Nervous Aspects Connected with Handwriting


(B) Teaching Handwriting
(C) The Problems of the Left-handed Child
(D) A Special Problem in Teaching Handwriting
(E) Stammering, Mirror Writing and Reversals

7. The author implies that children to write with their right hands.

(A) parents should break children of left-handed- ness


(B) left-handed children need special consideration
(C) left-handed persons are inclined to stutter
(D) left-handed persons are not more brilliant than right-handed ones
(E) left-handed persons are less skillful than right- handed ones

8. According to the passage, the traditional policy in teaching


handwriting has

(A) dismayed the experts


(B) resulted in failure to learn to write
(C) aimed at mirror writing
(D) made many children skillful with both hands
(E) resulted in unsolved problems

Questions 9 to 13

‗My father is not very well,‘ said Eleanor. John Bold was very sorry—so
sorry. He hoped it was nothing serious, and put on the unmeaningly
solemn face which people usually use on such occasions.

‗I especially want to speak to you about my father, Mr. Bold. Indeed, I


am now here on purpose to do so. Papa is very unhappy, very unhappy
indeed, about this affair of the hospital. You would pity him, Mr. Bold, if
you could see how wretched it has made him.‘

‗Oh, Miss Harding!‘

‗Indeed you would—anyone would pity him; but a friend, an old friend as
you are — indeed you would. He is an altered man; his cheerfulness has
all gone, and his sweet temper, and his kind happy tone of voice; you

CREA – All Rights Reserved I U Training Services


38
Reading Comprehension Vocabulary & Comprehension

would hardly know him if you saw him, Mr. Bold, he is so much altered;
and… and… if this goes on, he will

One of the most urgent problems in teaching handwriting is presented


by the left-handed child. The traditional policy has been to attempt to
induce all die.‘ Here Eleanor had recourse to her handkerchief, and so
also had her auditors; but she plucked up her courage, and went on with
her tale. ‗He will break his heart, and die. I am sure, Mr. Bold, it was not
you who wrote those cruel things in the newspaper…‘

John Bold eagerly protested that it was not, but his heart smote him as
to his intimate alliance with Tom Towers.

‗No, I am sure it was not; and papa has not for a moment thought so; you
would not be so cruel… but it has nearly killed him. Papa cannot bear to
think that people should so speak of him, and that everybody should
hear him so spoken of. They have called him avaricious, and dishonest,
and they say he is robbing the old men, and taking the money of the
hospital for nothing.‘

‗I have never said so, Miss Harding. I…‘

‗No,‘ continued Eleanor, interrupting him, for she was now in the full
flood tide of her eloquence; ‗no, I am sure you have not; but others have
said so; and if this goes on, if such things are written again, it will kill
papa. Oh! Mr. Bold, if you only knew the state he is in! Now papa does
not care much about money.‘

Both her auditors, brother and sister, assented to this, and declared on
their own knowledge that no man lived less addicted to filthy lucre than
the warden.

9. According to the passage, Eleanor believes her father will die


because

(A) he is suffering from a deadly disease


(B) he has told her so
(C) he is very depressed
(D) John Bold intends to kill him
(E) he plans to commit suicide

CREA – All Rights Reserved I U Training Services


39
Reading Comprehension Vocabulary & Comprehension

10. Of which of the following has Eleanor‘s father been accused?

I. Greed II Theft III. Blackmail

(A) I only (B) 11 only (C) III only


(D) I and II only (E) I and III only

11. Eleanor believes all the following about her father EXCEPT that he

(A) has been greatly transformed


(B) is guilty of the charges against him
(C) cares what people think of him
(D} is pitiable in his anguish
(E) is greatly afflicted by his troubles

12. It can be inferred that John Bold

(A) had something to do with Eleanor‘s father‘s problems


(B) is indifferent to what Eleanor thinks about him
(C) lied about Eleanor‘s father for his own persona! Gain
(D) believes Eleanor‘s father is an evil person
(E) is one of the old men cheated by Eleanor‘s father

13. In the last line, the term ―filthy lucre‖ means

(A) tobacco (B) stealing (C) extravagance


(D) money (E) gambling

Questions 14 to 18

Our theory and practice in the area of sentencing have undergone a


gradual but dramatic metamorphosis through the years. Primitive man
believed that a crime created an imbalance which could be rectified only
by punishing the wrongdoer. Thus, sentencing was initially vengeance-
oriented. Gradually, emphasis began to be placed on the deterrent value
of a sentence upon future wrongdoing.

Though deterrence is still an important consideration, increased


emphasis on the possibility of reforming the offender—of returning him
to the community a useful citizen — bars the harsh penalties once
imposed and brings into play a new set of sentencing criteria. Today,

CREA – All Rights Reserved I U Training Services


40
Reading Comprehension Vocabulary & Comprehension

each offender is viewed as a unique individual, and the sentencing judge


seeks to know why he has committed the crime and what are the
chances of a repetition of the offense. The judge‘s prime objective is
not to punish but to treat.

This emphasis on treatment of the individual has created a host of new


problems. In seeking to arrive at the best treatment for individual
prisoners, judges must weigh an imposing array of factors. I believe
that the primary aim of every sentence is the prevention of future
crime. Little can be done to correct past damage, and a sentence will
achieve its objective to the extent that it upholds general respect for
the law, discourages those tempted to commit similar crimes, and leads
to the rehabilitation of the offender, so that he will not run afoul of
the law again. Where the offender is so hardened that rehabilitation is
plainly impossible, the sentence may be designed to segregate the
offender from society so that he will be unable to do any future harm.
The balancing of these interacting, and often mutually antagonistic,
factors requires more than a good heart and a sense of fair play on the
judge‘s part, although these are certainly prerequisites. It requires the
judge to know as much as he can about the prisoner before him. He
should know the probable effects of sentences upon those who might
commit similar crimes and how the prisoner is likely to react to
imprisonment or probation. Because evaluation of these various factors
may differ from judge to judge, the same offense will be treated
differently by different judges.

The task of improving our sentencing techniques is so important to the


nation‘s moral health that it deserves far more careful attention than it
now receives from the bar and the general public. Some of those at the
bar and many civic-minded individuals who usually lead even the judges in
the fight for legal reform approach this subject with apathy or with
erroneous preconceptions. For example, I have observed the sentiment
shared by many that, after a judge has sentenced several hundred
defendants, the whole process becomes one of callous routine. I have
heard this feeling expressed even by attorneys who know better.

CREA – All Rights Reserved I U Training Services


41
Reading Comprehension Vocabulary & Comprehension

14. The author‘s purpose in this passage is to

(A) entertain readers by telling anecdotes about criminal sentencing


(B) inform readers of sentencing practices in the past
(C) convince judges of the need for harsher sentences
(D) tell people not to commit crimes for which they might receive unfair
sentences
(E) persuade readers that it is important to improve sentencing
techniques

15. The word ―metamorphosis‖ in the first sentence means

(A) restoration (B) interpretation (C) lethargy


(D) change (E) fault

16. A situation comparable to the ancient idea of sentencing would be

(A) a lion stalking and killing a deer


(B) a traffic officer putting a ticket on an illegally parked car
(C) a child slapping a playmate who has slapped him or her
(D) a customer returning defective merchandise
(E) an owner scolding a pet that has tried to run away

17. The author would agree with each of the following statements
EXCEPT

(A) a judge should treat each offender as an individual


(B) a judge should try to correct past damage
(C) the problem of sentencing deserves study
(D) a judge should refrain from imposing harsh penalties
(E) a judge has to be a student of human nature

18. According to the passage, judges today should impose sentences for
any of the following reasons EXCEPT

(A) upholding respect for the law (B) segregating the offender
(C) rehabilitating the offender (D) avenging the victim
(E) discouraging others from committing crime

Questions 19 and 21

CREA – All Rights Reserved I U Training Services


42
Reading Comprehension Vocabulary & Comprehension

All living organisms possess energy. Energy that is not being used is
known as potential energy. As the organism uses it, potential energy
becomes transformed into kinetic energy. Kinetic energy can take many
forms, including sound, light, and motion, but, in living organisms, all
energy eventually is changed into heat. Energy is therefore measured in
calories, one calorie being the amount of heat needed to warm a gram of
water one degree Celsius.

It is thus possible to measure the potential energy of food supplies.


Plants, for example, may be dried and then burned. The heat they
produce while burning is used to heat a known amount of water. The
heat energy, or calories, contained in the plant material can then be
calculated. It is also possible to tell how much energy is required to
produce each calorie of food. Plants get some of their energy from the
sun. However, farmers give plants that are grown for food additional
energy in the form of fertilizers and other materials. In other words,
the plants are given energy subsidies. In the past, energy subsidies to
agricultural products were provided mainly through the labor of people
and domesticated animals. Now, in our o agriculture depends to a large
extent on machines that use coal, petroleum, and natural gas, thus
consuming energy sources from the past. Energy is also spent in
packaging, preserving, and transporting agricultural products. It has
therefore been calculated that each calorie of food we eat costs ten
calories of subsidized energy.

19. Which of the following is the best title for the passage?

(A) Getting the Most from Potential Energy

(B) What Is a Calorie?

(C) Conserving Energy

(D) The Future of Energy Subsidies

(E) Measuring Energy and Food Supplies

CREA – All Rights Reserved I U Training Services


43
Reading Comprehension Vocabulary & Comprehension

20. Which of the following comparisons most closely parallels the


relationship between potential and kinetic energy described in the first
paragraph?

(A) A savings account to money spent on purchases


(B) Computer memory to human intelligence
(C) An atom to a molecule
(D) A parasite to its host
(E) A mountain to a valley

21. It can be inferred from the passage that the definition of energy
subsidies for plants includes which of the following?

I. Solar energy II. Human and animal labor

III. Energy supplied by machines

(A) III only (B) I and II only (C) I and III only
(D) II and III only (E) I, II and III

Questions 22 to 25

The great question that this paper will, but feebly, attempt to answer
is: What is the creative process? Though much theory has accumulated,
little is really known about the power that lies at the bottom of poetic
creation. It is true that great poets and artists produce beauty by
employing all the powers of personality and by fusing emotions, reason
and intuitions. But, what is the magical synthesis that joins and arranges
these complex parts into poetic unity?

John L. Lowes, in his justly famous The Road to Xanadu, developed one
of the earliest and still generally acceptable answers to this tantalizing
question. Imaginative creation, he concludes, is a complex process in
which the conscious and unconscious minds ―jointly operate.‖ ―There is .
. . the deep well with its chaos of fortuitously blending images; but
there is likewise the Vision which sees shining in and through the chaos,
the potential lines of Form, and with the Vision the controlling Will,
which gives to that potential beauty actuality.‖ The Deep Well is the
unconscious mind that is peopled with the facts, ideas, and feelings of
conscious activity. The imaginative vision, an unconscious activity, shines

CREA – All Rights Reserved I U Training Services


44
Reading Comprehension Vocabulary & Comprehension

through this land of chaos, of lights and shadows, silently seeking


pattern and form. Finally, the conscious mind again, through Will,
captures and embodies the idea in the final work of art. In this way is
unity born out of chaos. Though there can be no absolute certainty,
there is general agreement that the periods in the development of a
creative work parallel, to some extent, Lowes‘ theory of Well, Vision,
Form and Will. There are at least three stages in the creative process:
preparation, inspiration, work. In a sense, the period of preparation is
all of the writer‘s life. It is the Deep Well. It is especially a period of
concentration which gives the unconscious mind an opportunity to
communicate with the conscious mind. When remembrances of things
past reach the conscious level of the writer‘s mind, he is ready to go on
with the process. Part of this preparation involves learning a medium—
learning a language, learning how to write, learning literary forms. It is
important to note here that form cannot be imposed upon the idea.
Evidence, though sparse, shows that the idea gives birth to the form
that can best convey it. It is the Vision, according to Lowes, ―which sees
shining in and through the chaos the potential lines of Form . . .‖

22. According to the author, when remembrances of things past reach


the conscious level, the poet has reached the stage called

(A) Well (B) Vision (C) Form


(D) Will (E) Magical synthesis

23. According to the passage, which of the following statements is


true?

(A) The form determines the subject matter.


(B) The idea determines the form.
(C) Vision makes beauty an actuality.
(D) A writer‘s period of preparation is spent at school.
(E) A writer is unconscious when he prepares his work.

24. The author‘s attitude toward the work of John L. Lowes can best be
described as one of

(A) judicious admiration (B) mild indifference


(C) uninformed enthusiasm (D) reluctant tolerance

CREA – All Rights Reserved I U Training Services


45
Reading Comprehension Vocabulary & Comprehension

(E) feigned objectivity

25. In the passage, the author does all of the following EXCEPT

(A) cite an authority (B) define a term


(C) propose a solution (D) provide an example
(E) pose a question

Super Brain Drill


Questions 1 to 4

Solitude is a great chastener when once you accept it. It quietly


eliminates all sorts of traits that were a part of you—among others, the
desire to pose, to keep your best foot forever in evidence, to impress
people as being something you would like to have them think you are
even when you aren‘t. Some men I know are able to pose even in solitude;
had they valets they no doubt would be heroes to them. But I find it the
hardest kind of work myself, and as I am lazy I have stopped trying. To
act without an audience is so tiresome and profitless that you gradually
give it up and at last forget how to act at all. For you become more
interested in making the acquaintance of yourself as you really are,
which is a meeting that, in the haunts of men, rarely takes place. It is
gratifying, for example, to discover that you prefer to be clean rather
than dirty even when there is no one but God to care which you are; just
as it is amusing to note, however, that for scrupulous cleanliness you are
not inclined to make superhuman sacrifices, although you used to believe
you were. Clothes, you learn, with something of a shock, have for you no
interest whatsoever. You learn to regard dress merely as covering, a
precaution. For its color and its cut you care nothing.

1. Which of the following best expresses the main idea of the passage?

(A) Clothes make the man


(B) No man is a hero to his valet

CREA – All Rights Reserved I U Training Services


46
Reading Comprehension Vocabulary & Comprehension

(C) Seclusion permits self


(D) One‘s own company is the best company.
(E) One should always live as one pleases.

2. The word ―chastener‖ in the first sentence means

(A) discomfort (B) means of discipline (C) troublemaker


(D) misgiving (E) method of saving time

3. The author points out that solitude primarily gives us the chance to

(A) learn our own peculiarities (B) keep our best foot forward
(C) impress people (D) dress as we would like
(E) be immaculately clean

4. In describing his self the author‘s tone can best be described as

(A) arrogant (B) sullen (C) self-mocking


(D) defeated (E) abashed

Questions 5 to 9

The near-legendary history of the American West might have been


quite different had the Mexican not brought cattle-raising to New
Mexico and Texas. The Spanish style of herding cattle on open ranges
was different from the style of other Europeans, particularly the
English. The American rancho was possible because of the lack of
enough water for normal agricultural practices, and because of the easy
availability of large amounts of land. This land- extensive form of
cattle-raising required different techniques and brought forth the
vaquero, the cowboy (from the Spanish vaca, cow) who tended the
widely- scattered herds of Spanish longhorn cattle. Because of the
American penchant to be considered the inventors of nearly everything,
the wide-open style of cattle- ranching was appropriated from the
Mexican originators. As popular a folk-hero as the American cowboy is,
he owes his development to the Spanish and the Mexicans, not to the
English. It is quite probable, as McWilliams asserts, that ―with the
exception of the capital required to expand the industry, there seems
to have been nothing the American rancher or cowboy contributed to
the development of catt1e in the Southwest.‖

CREA – All Rights Reserved I U Training Services


47
Reading Comprehension Vocabulary & Comprehension

Other contributions of the Mexican cowboy were: the western-style


saddle with a large, ornate horn; chaparejos, or chaps; lazo, lasso; Ia
reata, lariat; the cinch; the halter; the mecate, or horsehair rope; chin
strap for the hat; feed bag for the horse; ten-gallon hat (which comes
from a mistranslation of a Spanish phrase ―su sombrero galoneado‖ that
really meant a ‗ ‗festooned‘ ‗ or ‗ ‗galooned‘ ‗ hat). Cowboy slang came
from such words as: juzgado, hoosegow; ranchero, rancher; estampida ,
stampede ; calabozo, calaboose; and pinto for a painted horse.

Just as the Mexican associations for the protection of the rights of


sheepherders gave rise to the American Sheepmen‘s Associations, the
Spanish system of branding range animals and registering these brands
became standard practice among Anglo stockmen. The idea of brands
originated in North Africa and was brought to Spain by the Moors, along
with their stocky ponies. The Mexican brands are of great antiquity,
having been copied from earlier Indian signs which include symbols of
the sky—sun, moon and stars. Hernando Cortez is said to have been the
first to use a brand on the continent.

5. Which of the following would be the best title for this passage?

(A) How to Herd Cattle


(B) The American Cowboy: A Romantic Figure
(C) Farming Practices in Europe and America
(D) Hispanic Contributions to Western Ranching
(E) Spanish Influence on American Culture

6. It can be inferred from the passage that American ranches


developed in the West rather than the East because

(A) more Spanish-speaking people lived in the West


(B) there was more money available in the West
(C) people in the East were more bound by tradition
(D) many jobless men in the East wanted to become cowboys
(E) there was more unsettled land available in the West

7. The author gives examples of cowboy slang in order to

(A) arouse the reader‘s interest

CREA – All Rights Reserved I U Training Services


48
Reading Comprehension Vocabulary & Comprehension

(B) show that he is familiar with the subject


(C) prove that many cowboys lacked education
(D) point out the differences between America‘s East and West
(E) demonstrate how these terms originated

8. According to the author, which of the following did Mexicans


contribute to ranching?

I. Money to buy ranches II. Methods of handling animals

III. Items of riding equipment

(A) I only (B) II only (C) III only


(D) I and II only (E) II and III only

9. Which of the following best describes the development of this


passage?

(A) Major points, minor points


(B) Statement of problem, examples, proposed solution
(C) Introduction, positive factors, negative factors
(D) Cause, effects
(E) Comparison, contrast

Questions 10 to 13

I smiled: I thought to myself, Mr. Rochester is peculiar—he seems to


forget that he pays me £30 per annum for receiving his orders.

―The smile is very well,‖ said he, catching instantly the passing
expression; ―but speak too.‖

―I was thinking, sir, that very few masters would trouble themselves to
inquire whether or not their paid subordinates were piqued and hurt by
their orders.‖

―Paid subordinates! What, you are my paid subordinate, are you? Oh yes,
I had forgotten the salary! Well then, on that mercenary ground, will
you agree to let me browbeat you a little?‖

CREA – All Rights Reserved I U Training Services


49
Reading Comprehension Vocabulary & Comprehension

―No, sir, not on that ground: but, on the ground that you did forget it,
and that you care whether or not a dependent is comfortable in his
dependency, I agree heartily.‖

―And will you consent to dispense with a great many conventional forms
and phrases, without thinking that the omission arises from insolence?‖

―I am sure, sir, I should never mistake informality for insolence: one I


rather like, the other nothing free- born would submit to, even for a
salary.‖

―Humbug! Most things free-born will submit to anything for a salary;


therefore, keep to yourself, and don‘t venture on generalities of which
you are intensely ignorant. However, I mentally shake hands with you
for your answer, despite its inaccuracy; and as much for the manner in
which it was said, as for the substance of the speech: the manner was
frank and sincere; one does not often see such a manner: no, on the
contrary, affectation, or coldness, or stupid, coarse-minded
misapprehension of one‘s meaning are the usual rewards of candour. Not
three in three thousand raw school-girl governesses would have
answered me as you have just done. But I don‘t mean to flatter you: if
you are cast in a different mould to the majority, it is no merit of yours:
Nature did it. And then, after all, I go too fast in my conclusions: for
what I yet know, you may be no better than the rest: you may have
intolerable defects to counterbalance your few good points.‖

10. It can be inferred that Mr. Rochester is speaking to

(A) an enemy (B) a friend (C) a relative


(D) an employee (E) a colleague

11. According to Mr. Rochester, most people would agree with which of
the following?

(A) Honesty is the best policy.


(B) Virtue is its own reward.
(C) He who pays the piper calls the tune.
(D) A penny saved is a penny earned.
(E) Experience is the best teacher.

CREA – All Rights Reserved I U Training Services


50
Reading Comprehension Vocabulary & Comprehension

12. Which of the following does Mr. Rochester appear to value LEAST?

(A) honesty (B) informality (C) submissiveness


(D) openness (E) generosity

13. Mr. Rochester‘s tone in the passage can best be described as

(A) sympathetic (B) pompous (C) humorous


(D) loving (E) cynical

Questions 14 to 17

As the market for hay declined, other farmers looked west and saw
that in the flat lands of the prairie country the farmers were growing
rich by raising corn and hogs: and they said, without thought or wisdom
or knowledge, ‗ ‗If they can do it, we can. ―And so they plowed the grass
and meadowlands and even the pastures of that rolling, hilly country and
planted corn. They planted the corn in rows, running more often than
not up and down slopes and hills. Every time it rained, each furrow
between the standing corn became a miniature gully carrying off the
precious rainfall and bearing with it the good topsoil that remained and
the fertilizer the farmer had bought out of his hard-earned income.

14. The title that best expresses the ideas of this passage is

(A) Waste of the Soil


(B) Rotating Crops
(C) Rainfall and the Corn Crop
(D) Why Farmers Become Discouraged
(E) The Reward of Courage

15. The author implies that corn was a less satisfactory crop in the
more easterly section because the land in this section was less

(A) fertile (B) moist (C) flat


(D) thoroughly fertilized (E) arid

16. We may infer from the passage that the farmers would have been
wiser to

CREA – All Rights Reserved I U Training Services


51
Reading Comprehension Vocabulary & Comprehension

(A) irrigate the land (B) add topsoil to the fields


(C) plow furrows across the slope
(D) use more fertilizer (E) fill the gullies

17. The author‘s attitude toward the farmers could best be described
as

(A) humorous (B) critical (C) indifferent


(D) callous (E) philanthropic

Questions 18 to 21

India‘s river systems have played a dominant role. In the west, the
Indus traverses the Punjab, the land of five rivers, from Himalayan
snows through an alluvial plain to the Arabian Sea, a course of 1,800
miles. Below the junction of the five rivers, in the land of Sind, human
habitation has been possible only because of irrigation systems. Sind
normally receives five inches of rain annually. The upper Indus Valley of
Punjab receives eight to twelve inches of rain. Its natural ground cover
is that of a dry tropical forest while its dry hilly districts have desert
ecology. The Indus River Valley is separated from the Ganges River
basin to the east by the extensive Thar Desert which stretches along
the western border of present-day India. The highland corridor of
eastern Punjab, known historically as the cockpit of India, has always
provided easy access to the Ganges for the Indus hills people.
Conquerors, avoiding the Thar Desert, marched through the hills into
the fertile and heavily populated Ganges River Valley. This flood plain,
an area of over 300,000 square miles, is watered by the Ganges and its
five large tributaries. The great Hindu empires thrived along the length
of ‗ ‗Mother Ganges, ‗ ‗ the sacred river, from the uplands around Delhi
to the low lying cities of Calcutta and Dacca. The valley remains one of
the world‘s most heavily populated regions. The upland region, the
present Indian state of Uttar Pradesh, averages about twenty-five
inches of rainfall annually while Bengal‘s range is forty to sixty inches.
To the south of the Ganges River basin is the Deccan plateau which
extends southward to the Krishna River. Beyond this river is South
India which has, more or less, remained apart from the north, and has
maintained its own ancient culture. Along the western edge of the

CREA – All Rights Reserved I U Training Services


52
Reading Comprehension Vocabulary & Comprehension

Deccan plateau, facing the Arabian Sea, an eroded mountain wall, some
600 miles long, has isolated the people of the interior from those of the
coast. The survival of these inhabitants of the Deccan plateau and those
of South India depends upon the July to October seasonal monsoon.
During the monsoon, rain storms from the Arabian Sea cross over the
peninsula to the Himalayas. Unlike the snow-fed rivers in Northern
India, the rivers of the plateau and the south are dependent upon this
rain. Any lasting drought brings famine and death.

18. Which of the following titles best expresses the ideas of this
passage?

(A) Drought and Famine in India


(B) The Ganges: A Sacred River
(C) The Influence of Rainfall in India
(D) India: Its History and Geography
(E) Waterways of the Indian Subcontinent

19. The author‘s tone in this passage can best be described as

(A) scholarly (B) enthusiastic (C) sympathetic


(D) biased (E) nostalgic

20. It can be inferred from the passage that one reason the great
Hindu empires thrived was that they

(A) emphasized religion


(B) were irrigated by the Ganges River
(C) reached from Delhi to other cities
(D) had no enemies
(E) were great cultural centers

21. It can be inferred from the passage that

(A) the seasonal monsoon brings destruction and death


(B) the rain season in India lasts for one month
(C) it never rains in the Himalayas
(D) people in Southern India welcome the monsoon
(E) South India is under populated

CREA – All Rights Reserved I U Training Services


53
Reading Comprehension Vocabulary & Comprehension

Questions 22 to 25

In 1896 Henri Becquerel found that uranium salts emitted penetrating


radiations similar to those which Roentgen had produced only a year
earlier with a gas discharge tube. The tremendous importance of this
discovery was not apparent until a few years later when Pierre and
Marie Curie announced the isolation from a uranium mineral,
pitchblende. of two substances many times more radioactive than
uranium itself. These two substances were subsequently shown to be
two new elements, polonium and radium. Elements which are naturally
radioactive spontaneously emit radiations without the addition of any
energy to them. Later we shall see that artificial radioactivity can be
produced by adding energy to originally stable nuclei.

Rutherford and Soddy, investigating the phenomenon discovered by


Becquerel, found that the empirical facts of radioactivity could be
xp1ain by assuming that radioactive atoms were not stable but
disintegrated at characteristic rates to form new atoms of other
elements. As soon as the radioactive emissions were experimentally
identified and it was proved that alpha ―rays‖ are actually helium ions, it
became clear that the assumptions made by Rutherford and Soddy were
correct. It was soon found that the disintegration product of radium is
also naturally radioactive and investigations of decay products led to
the identification of other radioelement ranging in atomic number from
92 (uranium) to 81 (thallium). These radio elements are now known to be
intimately related to each other in the radioactive series.

During the early years of the Curie investigations uranium had only a
limited industrial use, chiefly in the glass industry, and most of the
material came from the Joachimstal mines in Czechoslovakia. As
industrial uses for radioactive elements developed, uranium ore was
found widely scattered throughout the world with extensive deposits in
the Belgian Congo and in the Great Bear Lake region in Canada. The
element is principally mined in the form of pitchblende, which may
contain a high percentage of U3O8. The ore presents a brown-black
appearance somewhat resembling pitch in luster. In the United States,

CREA – All Rights Reserved I U Training Services


54
Reading Comprehension Vocabulary & Comprehension

deposits of another uranium ore, known as carnotite, are found in


several Rocky Mountain States.

22. The title that best expresses the ideas of this passage is

(A) Scientists in the Atomic Age (B) The Curies


(C) The Discovery of Radiation
(D) Pitchblende and Its Products (E) Uranium and Radiation

23. According to the passage, Uranium

(A) was discovered in 1896


(B) is used in the glass industry
(C) is found in pure form in the United States
(D) is more radioactive than polonium
(E) contains helium atoms

24. According to the passage atoms that are naturally radioactive

(A) disintegrate over time


(B) have energy added to the nuclei
(C) were discovered by Rutherford
(D) were discovered by Roentgen
(E) are mined in the Congo

25. It can be inferred that more uranium ore has been found because

(A) its radioactivity makes it easy to find


(B) its value makes people look for it
(C) it is contained in pitchblende and carnotite
(D) there is more of it now than in the past
(E) it has a brown-black appearance in its natural state

CREA – All Rights Reserved I U Training Services


55
Logical Ability

CREA – All Rights Reserved I U Training Services


56
This Page is Intentionally
Left Blank

CREA – All Rights Reserved I U Training Services


57
1
Analytical Puzzles

CREA – All Rights Reserved I U Training Services


58
PRE-LESSON Challenge – How deep is your
sense of analysis?
Time Limit – 5 minutes
Exactly seven persons – P, Q, R, S, T U and V –
participate in and finish all of a series of swimming
races. There are no ties for any position at the finish
of any of the races.
V always finishes somewhere ahead of P.
P always finishes somewhere ahead of Q.
Either R finishes first and T finishes last, or S finishes first and U or
Q finishes last.
1. If in a race V finishes fifth, which of the following must be true ?
a) S finishes first. b) R finishes second.
c) T finishes third 4] Q finishes fourth.
2. If in a race R finishes first, V can finish no lower than
1] second b) third c) fourth d) fifth
3. If in a race S finishes second, which of the following can be true?
1] P finishes before R .b) V finishes before S.
c) P finishes before V 4) U finishes before V.

CREA – All Rights Reserved I U Training Services


59
Analytical Puzzles Logical Ability

In This Chapter
 Solve Ordering/Arrangement Puzzles
 Analyze and Solve puzzles based on family relationship
 Comparison Puzzles

One cannot classify puzzles into various categories but


generally speaking, the questions can be classified as follows.

1. Ordering / Arrangement Puzzles


2. Selection based on given conditions
3. Family Relationships
4. Comparison Puzzles

Type I
Ordering / Arrangement Puzzles
These puzzles require you to place persons or items in a linear
or circular manner. The criteria used to determine order can include
size, time, rank, space, weight etc. Seating / placing arrangement
questions are the easiest.
Let‘s consider the following puzzle.
Six persons A, B, C, D, E and F are sitting in two rows, three
in each. E is not at the end of any row. D is second to the left of F. C,
the neighbour of E is sitting diagonally opposite to D. B is the
neighbour of F.
Let us analyse the information given above.
There are six persons sitting in two rows, three in each, which can be
represented by the following diagram.
Row 1 __ __ __

Row 2 __ __ __
E is not at the end of any row means that the diagram becomes as
shown below.

Row 1 __ E __
OR

CREA – All Rights Reserved I U Training Services


60
Analytical Puzzles Logical Ability

Row 2 __ E __

D is second to the left of F. Now, there are only three persons in a


row. Therefore, naturally D and F are at the ends of any of the rows
with D being to the left of F.

Row 1 D E F
OR
Row 2 D E F

So C is the neighbour of E means that E cannot sit with D and F. Also,


C is diagonally opposite to D. Hence, taking all the clues into account
the diagram becomes as shown below.

Row 1 D __ F

Row 2 __ E C

Lastly, B is the neighbour of F completes the arrangement by making


A the neighbour of E as shown below.

Row 1 D B F

Row 2 A E C

Now let us try to answer the questions.


Which of the following are sitting diagonally opposite to each
other?
a) F and C b) D and A
c) A and C d) A and F
It is clear from the final diagram that D and C, and A and F are the
person sitting diagonally opposite to each other. Hence, the answer is
(d).

Who is facing B?

CREA – All Rights Reserved I U Training Services


61
Analytical Puzzles Logical Ability

a) A b) C c) D d) E
As the final diagram shows E is the person facing B. Hence is the
answer is (d).

Which of the following are in the same row?


a) A and E b) E and D
c) C and B d) A and B
The final diagram clearly illustrates (a) as the answer.

If C interchanges position with F, and D with A then, which of the


following are sitting diagonally opposite to each other?
a) F and C b) D and A
c) A and C d) A and F
Taking the above condition into account the final diagram changes to
Row 1 A B C

Row 2 D E F
Which means (d) is the right answer.

After interchanging position with E, who will be the neighbours of


B in the new position?
a) C and A b) F and B
c) Only B d) Only A
After the interchanging the final diagram becomes
Row 1 D E F

Row 2 A B C
Clearly, C and A are B‘s neighbours. Hence, the answer is (a).

Let‘s see a more complex one.

Five friends in their first year of degree took the same combination
of subjects. Each obtained a different grade in each subject taken,
and no two students had the same grade in the same subject. Read the
information given below:

CREA – All Rights Reserved I U Training Services


62
Analytical Puzzles Logical Ability

1. Aditya outscored Bhanu in Physics, and Neeraj in Math.


2. Sirisha was the only girl to get a ‗C‘ grade, but she managed no
‗A‘ grades.
3. The pupil with an ‗E‘ in Math gained a ‗B‘ in Chemistry, but was
not awarded a ‗C‘ in Physics.
4. Pankaj‘s Physics grade was a ‗D‘ and his highest grade was a ‗C‘.
5. The ‗B‘ in Math did not go to the same student as the ‗E‘ in
Physics.
6. Bhanu‘s best result was in Chemistry, but her Math grade was
lower than Pankaj‘s.
Who got grade ‗E‘ in Math?

It can be solved as follows.


Aditya outscored Neeraj in Math, so his Math grade was not an E, and
Neeraj grade in Math was not an A. From 2nd clue we understand
Sirisha did not manage an A in Math. Also, from clue 4 we gather that
Pankaj did not get the A grade in Math because his highest grade was
a C. Now Bhanu‘s Math result was lower than Pankaj, so it was not an A.
Therefore, Aditya got an A in Math.

We derive from the 4th and 6th clues that Bhanu‘s result in Math was
lower than Pankaj‘s whose highest grade was a ‗C‘. Therefore her
grade must be either D or E. Pankaj had a D in Physics so his maths
grade should be C because he cannot have same grade in two subjects.
From 1st clue we gather, Aditya got a better grade than Bhanu in
Physics and his Physics grade was not A since he had A in Math.
Therefore, Bhanu‘s grade in Physics was not B. Neeraj, therefore, got
an A in Physics and Bhanu got an A in Chemistry, E in Physics and D in
math.

Now, there are only Neeraj and sirisha remaining. According to 3 rd


clue the person with an E in math gained B in Chemistry and did not
have C in Physics, which means the person didn‘t manage a C. This
conditioned by Sirisha as seen in clue 2. Therefore it must be Neeraj
who had an E in Math

CREA – All Rights Reserved I U Training Services


63
Analytical Puzzles Logical Ability

Type II
Selection Based on Given Conditions

Grouping puzzles, as the term implies, require you to separate


elements--typically people—into groups. Some conditions of the puzzle
can apply to entire groups only, some to elements within a group only,
and some to both. This added complexity makes grouping puzzles, in
general, harder than ordering puzzles. Because grouping puzzles
partition elements into sets, the number of elements is often an issue.
Counting may have been one of man's first thought processes;
nevertheless, counting possibilities is deceptively hard. This tends to
make grouping puzzles more difficult than ordering puzzles.
Pay close attention to the maximum or minimum number of elements
in a group; this is often the heart of the puzzle.

Selection Puzzle

The starting line-up for the School Volleyball team is chosen from
the following two groups:

Group A: Jacky, Dhiraj, Basu, Eknath


Group B: Latif, Ram, Jagdish, Mahesh, Pappu

The following requirements must be meet:


1. Three players are chosen from Group A, and three from Group
B.
2. Jagdish plays only if Basu plays.
3. Dhiraj and Basu do not both play.
4. If Jagdish playss, then Mahesh does not.
5. Exactly 3 of the four spikers--Jacky, Basu, Jagdish, Pappu--
must be chosen.

It is best to solve this problem without a diagram; however, we will


still symbolize the conditions for clarity and easy reference. The

CREA – All Rights Reserved I U Training Services


64
Analytical Puzzles Logical Ability

condition "Jagdish plays only if Basu plays" implies only that if Jagdish
is playing then Basu must be playing as well. So we symbolize it as
Jagdish-->Basu
The condition "Dhiraj and Basu do not both play" means that if one
plays then the other does not. So we symbolize it as Dhiraj-->~Basu.

Students often misinterpret this condition to mean that


neither of them plays. To state that neither plays, put both at the
beginning of the sentence: Both Dhiraj and Basu do not play.
The condition "If Jagdish plays, then Mahesh does not" is naturally
symbolized as
Jagdish-->~Mahesh.
It tells us that if J plays then M do not, but tells us nothing when M
does not play.
Such a condition, where the two parts of an if-then statement do not
similarly affect each other, is called a nonreciprocal condition. On the
other hand, a condition such as: Jagdish<-->~Mahesh affects J and M
equally.
In this case, we are told that if J plays then M does not as
before, but we are told additionally that if M does not play then J
does.
It is important to keep the distinction between reciprocal and
nonreciprocal relations clear; a common mistake is to interpret a
nonreciprocal relation as reciprocal. The remaining conditions cannot
be easily written in symbol form, but we will paraphrase them in the
schematic:

Jagdish-->Basu
Dhiraj-->~Basu
Jagdish-->~Mahesh

3 from Group A. 3 from Group B


Spikers: Jacky, Basu, Jagdish, Pappu
3 Spikers Eknath, Latif, Ram are "wild"

CREA – All Rights Reserved I U Training Services


65
Analytical Puzzles Logical Ability

Note: Eknath, Latif, and Ram are independent because there are no
conditions that refer directly to them. We now turn to the questions.

1. If Jagdish plays, which of the following must also play?


(A) Mahesh or Jacky (B) Dhiraj or Latif
(C) Dhiraj or Jacky (D) Jacky or Pappu
(E) Mahesh or Ram
From the condition Jagdish-->Basu, we know that if Jagdish plays,
then Basu must play as well. Now both Jagdish and Basu are fast-
break specialists, and three of the four fast-break specialists must
play. So at least one of the remaining fast-break specialists--Jacky or
Pappu--must also play. The answer is (D).

2. All of the following pairs of players can play together EXCEPT:


(A) Eknath and Dhiraj (B) Jagdish and Jacky
(C) Ram and Jacky (D) Jacky and Basu
(E) Pappu and Mahesh
Begin with choice (A).
Both Eknath and Dhiraj are from Group A, so the remaining 3 players
must be chosen from
Group B. Additionally, they must all be fast-break specialists since
neither E nor D is--there are exactly 3 fast-break specialists. But
Jagdish and Pappu are the only fast-break specialists in Group B.

So the third fast-break specialist cannot be chosen. The answer


therefore is (A).

This type of question can be time consuming because you may have to
check all the answer-choices-- save these questions for last.

3. If Mahesh plays, which one of the following is a complete and


accurate list of the players from Group A. any one of whom could
also play?
(A) J (B) J, D (C) J, B
(D) J, D, B (E) J, E, B

CREA – All Rights Reserved I U Training Services


66
Analytical Puzzles Logical Ability

Jagdish cannot play with Mahesh according to the condition Jagdish--


>~Mahesh
To play three fast-break specialists, therefore, Jacky, Basu, and
Pappu are all required to play. Since both Jacky and Basu are from
Group A and exactly two players from that group play, these two
players comprise the complete list of players from Group A when
Mahesh also plays. The answer is (C).

4. Which one of the following players must play?


(A) Pappu (B) Jacky (C) Jagdish
(D) Mahesh (E) Basu
Suppose Basu does not play. Then the 3 Spikers must be Jacky,
Jagdish, and Pappu. But if Jagdish plays, then from the initial
conditions Basu must also play. Hence Basu must always play. The
answer is (E).
Let‘s look at another puzzle.
An artist has exactly seven paintings - T, U, V, W, X, Y and Z -
from which she must choose exactly five to be in an exhibit. Any
combination is acceptable provided it meets the following
conditions:
a. If T is chosen, X cannot be chosen.
b. If U is chosen, Y must also be chosen.
c. If V is chosen, X must also be chosen.

Let‘s rewrite the given conditions using codes.


1. T-->~X
2. U-->Y
3. V-->X
Now, let‘s answer the questions keeping these conditions in mind.
a. Which one of the following is an acceptable combination of
paintings for inclusion in the exhibit?
1. TUVXY 2. TUVYZ 3. TWXYZ
4. UVWYZ 5. UVWXY
From the first condition, wherever T comes, X cannot come.
Therefore, 1 is ruled out. Now from conditions 2 and 3, wherever U

CREA – All Rights Reserved I U Training Services


67
Analytical Puzzles Logical Ability

comes Y must come and with V, X must come. Hence, option 5 is the
correct answer.

b. If painting T is chosen to be among the paintings included in


the exhibit which one of the following cannot be chosen to be
among the paintings included in the exhibit?
1. U 2. V 3. W 4. Y 5. Z
If painting T is chosen, then X cannot come therefore, V cannot be
chosen. Hence the correct answer is 2.

c. Which one of the following substitutions can the artist always


make without violating restrictions affecting the combination of
paintings given that the painting mentioned first was not and the
painting mentioned second was originally going to be chosen?
1. T replaces V 2. U replaces Y 3. V replaces X
4. W replaces Y 5. Z replaces W.
As Z and W, are not bound by any conditions, option 5 is the correct
answer.

d. If the artist chooses painting V to be included among the


paintings in the exhibit, which one of the following must be true of
that combination of paintings?
1. T is not chosen 2. Y is not chosen
3. U is chosen 4. W is chosen 5. Z is chosen.

Solve yourself!

Task Assignment Puzzle

These puzzles involve assigning characteristics to the


elements, typically people. The most common task in these puzzles is
to assign a schedule. You probably have had some experience with
schedules; you may have written the weekly work-schedule for a
business. If so, you know how difficult the task can become, even when

CREA – All Rights Reserved I U Training Services


68
Analytical Puzzles Logical Ability

only a few conditions are placed on the employees: Babu will work
Monday, Tuesday, or Friday only. Sita will work evenings only. Sanjay
will not work with Babu. Add to this that the company must have full
staff weekdays, but only three people can work weekends.
Because the characteristics are typically assigned to groups
of elements, assignment puzzles can look very similar to grouping
puzzles. Additionally, in grouping puzzles the groups are often
identified by their characteristics. However, in assignment puzzles
you pair each element with one or more characteristics, whereas in
grouping puzzles you partition the elements into two or more groups.
Many assignment puzzles can be solved very efficiently by
using an elimination grid. An example will illustrate this method of
diagramming.

Elimination Grid

Professor Pathak, Head of the Math Department at Nehru University,


is making the term‘s teaching schedule time table. Besides himself
there are four other professors--Walia, Nayar, Dhar, and Ehsan.
Their availability is subject to the following constraints.
 Walia cannot teach on Monday or Thursday.
 Dhar cannot teach on Wednesday.
 Ehsan cannot teach on Monday or Friday.
 Associate Professor Nayar can teach at any time.
 Prof. Pathak cannot teach evening classes.
 Walia can teach only evening classes.
 Prof. Pathak cannot teach on Wednesday if Nayar teaches on
Thursday, and Nayar teaches on
 Thursday if Prof. Pathak cannot teach on Wednesday.
 At any given time there are always three classes being taught.

We indicate that a teacher does not work at a particular time by


placing an X on the elimination grid.
Placing the conditions on a grid yields

CREA – All Rights Reserved I U Training Services


69
Analytical Puzzles Logical Ability

M T W Th. F
Walia a.m. X X X X X
p.m. X X
Dhar a.m. X
p.m. X
Nair a.m.
p.m.
Ehsan a.m. X X
p.m. X X
Pathak a.m.
p.m. X X X X X

To answer the following questions, we will refer only to the table, not
the original problem.
1. At which one of the following times can Walia, Dhar, and Ehsan
all be teaching?
(A) Monday morning (B) Friday evening
(C) Tuesday evening (D) Friday morning
(E) Wednesday morning.
The table clearly shows that all three can work on Tuesday night.
The answer is (C).

2. For which day will the Professor have to hire a part-time


teacher?
(A) Monday (B) Tuesday (C) Wednesday
(D) Thursday (E) Friday
Dhar and Nayar are the only people, who can work Monday evenings,
and three classes are always in session, so extra help will be needed
for Monday evenings. The answer is (A).

3. Which one of the following must be false?


(A) Dhar does not work on Tuesday.
(B) Ehsan does not work on Tuesday morning.

CREA – All Rights Reserved I U Training Services


70
Analytical Puzzles Logical Ability

(C) Pathak works every day of the week except Wednesday


(D) Nayar works every day of the week except Wednesday
(E) Dhar works every day of the week except Wednesday
The condition "Prof. Pathak cannot teach on Wednesday if Nayar
teaches on Thursday, and
Nayar teaches on Thursday if Prof. Pathak cannot teach on
Wednesday" can be symbolized as
(P_W)<-->(N=TH)

Now, if Nayar works every day of the week, except Wednesday, then
in particular he works on Thursday.
So from the condition (P_W) <--> (N=TH), we know that Prof.
Pathak cannot work on Wednesday.
But from the table this leaves only Nayar and Ehsan to teach the
three Wednesday morning classes.
Hence the answer is (D).

4. If Nayar does not work on Thursday, then which one of the


following must be true?
(A) Pathak works Tuesday morning
(B) Dhar works Tuesday morning
(C) Ehsan works on Tuesday
(D) Pathak works on Wednesday
(E) Walia works on Tuesday morning
If you remember to think of an if-and-only-if statement as equality,
then this will be an easy problem.
Negating both sides of the condition (P_W) <--> (N=TH) gives
(P=W) <--> (N_TH).
This tells us that Prof. Pathak must work on Wednesday if Nayar does
not work on Thursday. The answer, therefore, is (D).

Type III
Questions Involving Family Relations

CREA – All Rights Reserved I U Training Services


71
Analytical Puzzles Logical Ability

In this kind of questions, clues are given about different members of


the family- their relation, their profession, their
habits/qualities/hobbies, their preferences etc. One has to analyse
and process the whole information and then answer the questions that
follow.

Let us take a look at the following question:

Directions (questions 1 to 5): Study the following information


carefully and answer the questions given below it.
1. A, B, C, D, E, and F are six members in a family in which
there are two married couples.
2. E, a professor is married to a doctor who is mother of C
and F.
3. B, the lawyer is married to A.
4. A has one son and one grandson.
5. Of the two married ladies one is a housewife.
6. There is also one student and one male engineer in the
family.
From point 3, we gather, BA is one couple.
Now, the remaining members of the family are: C, D, E and F.
Point 2 informs us that C and F are children of E. Therefore, ED is the
other couple.
BA should be the elderly couple because ED is the couple who have
children.
It is clear from point 2 that E is a professor married to D, who is a
female doctor.
B is lawyer, whose wife A is a housewife.
A‘s son is E and A is the grandmother of C and F.
The grandson is a male engineer and the granddaughter is a student.

Now, let‘s try to answer the questions.

1. How is A related to C?
a) Grandfather b) Mother c) Sister

CREA – All Rights Reserved I U Training Services


72
Analytical Puzzles Logical Ability

d) Grandmother e) None of these


Clearly, as discussed above, A is the grandmother of C. Hence the
answer is d).

2. Who among the following is the housewife?


a) A b) B c) D d) E e) None of these
As we have already discussed A is the housewife. Therefore the
answer is a).

3. How is C related to F?
a) Brother b) Sister c) Brother or Sister
d) Data inadequate e) None of these
We are sure that among C and F one is a male and the other a female.
But we cannot be sure about their sexes. Therefore, the answer is c).

4. Which of the following represents the group of females in the


family?
a) ADC b) ADF c) BEC d) Data inadequate e) None of these
We are not sure about the sex of the C and F. Therefore, d) is
the right answer.

5. Which of the following is true about the granddaughter in the


family?
a) She is a lawyer b) She is a student
c) She is an engineer d) Data inadequate e) None of these
We gather from the last point in the discussion above that the
granddaughter is a student. Hence the answer is b).

Type IV
Comparison Puzzles

A lot of clues are given with lot of data. One has to analyse, compare
and sort data and answer the questions that follow. These types of
puzzles are the most simple for people with good hang of numbers.

CREA – All Rights Reserved I U Training Services


73
Analytical Puzzles Logical Ability

Let‘s look at a comparison puzzle.

Five students A, E, I, O and U were comparing the scores each


received on a test and a quiz. The following was discovered.
A’s quiz score was 80.
A’s test score equals U’s quiz score.
U’s test score equals A’s quiz score.
A’s quiz score 15 less than U’s quiz score.
O’s test score is 20 more than his quiz score and is 20 more than
I’s test score.
O’s test score is 40 more than E’s quiz score.
I’s quiz score is 10 less than E’s quiz score.

Let‘s try to understand the given clues.


Making a table will definitely help to keep track of the clues/facts
given.
After taking the first three clues into account the table looks like the
one shown below:
A‘s quiz score is 80 which is the same as U‘s test score. A‘s test score
on the other hand is equal to U‘s quiz score. Both the scores are not
yet clear.

Students/ Test Scores Quiz Score Test Score


A 80 Same as U‘s quiz
score
E
I
O
U Same as A‘s test 80
score

After taking the next (fourth) clue into consideration, we get:

Students/ Test Scores Quiz Score Test Score

CREA – All Rights Reserved I U Training Services


74
Analytical Puzzles Logical Ability

A 80 95
E
I
O
U 95 80

Because A‘s quiz score is 15 less than U‘s quiz score. We get:
A = 80 B = A + 15
Therefore, B = 95

After taking the next (fifth) clue into consideration, we get:


Suppose x is O‘s quiz score. Then, O‘s test score becomes x + 20 and
I‘s test score is x.

Students/ Test Scores Quiz Score Test Score


A 80 95
E
I x
O x x + 20
U 95 80

Similarly, when we consider the 6th and 7th clues we come to know
that:
If E‘s quiz score is y then, I‘s quiz score is y – 10 and also O‘s test
score in terms of E‘s quiz score is y + 40. There is no information
given about E‘s test score.

Students/ Test Scores Quiz Score Test Score


A 80 95
E y --
I y - 10 x
O x x + 20 or y + 40
U 95 80

CREA – All Rights Reserved I U Training Services


75
Analytical Puzzles Logical Ability

Now that we have taken all the clues into account, let‘s try to answer
the questions.

Questions
1. If E’s quiz score is 60, what is O’s quiz score?
a) 80 b) 70 c) 60 d) 50 e) 40
If E‘s quiz score is 60 then, y = 60
O‘s quiz score is x. x can be got by finding O‘s test score which is
both in terms of x and y.
O‘s test score is y + 40 = 60 + 40 = 100
Also, O‘s test score is x + 20.
Therefore, x + 20 = 100 or x = 80.
Hence, E‘s quiz score is 80. Therefore the answer is a).

2. Which of the following is true?


i) I’s test score equals O’s quiz score.
ii) E’s quiz score equals U’s quiz score.
iii) A’s quiz score equals U’s test score.
a) i only b) iii only c) i and iii only
d) ii and iii only e) i, ii and iii
One can see from the table above that i) is true. We cannot say the
same about ii). Also iii) is true because A’s quiz score = U’s test
score = 95
Therefore, the answer is c).

3. What is U’s test score?


a) 55 b) 65 c) 80 d) 95 e) 100
It‘s clear from the table above that U‘s test score is 80. Therefore,
the answer is c).

4. If I’s test score is 45, what is O’s test score?


a) 25 b) 35 c) 45 d) 55 e) 65
If I‘s test score is 45 then, from the table, x = 45. O‘s test score is
x + 20.
Therefore, x + 20 = 45 + 20 = 65.

CREA – All Rights Reserved I U Training Services


76
Analytical Puzzles Logical Ability

Hence the answer is e).

5. If E’s quiz score is 50, which of the following is true?


i) I’s test score is 70
ii) O’s test score is 90.
iii) I’s quiz score is 30
a) i only b) ii only
c) iii only d) i and ii only e) i, ii and iii
E‘s quiz score is 50 means y = 50. I‘s test score is y – 10 = 50 – 10
= 40. Hence, i) is false. O‘s test score from the table is y + 40 = 50
+ 40 = 90. Therefore, ii) is true. Also, I‘s quiz score is y – 10 = 50
– 10 = 40. Therefore, iii) is false.
Hence, the answer is b).

6. If O’s quiz score is the same as U’s quiz score, which of the
following must be true?
i) I’s test score is 95 ii) O’s test score is110
iii) I’s quiz score is 70 iv) E’s quiz score is 75
a) i only b) ii and iv only
c) i and iii only d) i and iv only e) ii and iii only
O‘s quiz score is the same as U‘s quiz score means, x = 95.
We have to derive the y value in order to find I‘s test score. O‘s test
score is x + 20 = 95 + 20 = 105. Therefore ii) is false.
Also, O‘s test score is given by y + 40. Therefore, y + 40 = 105 or y
= 65.
Now, I‘s test score is x, which is 95. Therefore, i) is true.
I‘s quiz score is given by y – 10 or 65 – 10 = 55. Therefore, iii) is
false.
E‘s quiz score is y. Therefore, iv) is also false.
Hence, the answer is a).

CREA – All Rights Reserved I U Training Services


77
Analytical Puzzles Logical Ability

Brain Drill
Questions 1-3 are based on the following:
In a group there are five persons A, B, C, D, E. There is a tap dancer, a
preacher and a swimmer in the group. E is the husband of a member of
the group and the two are the only married couple in the group. B is
the brother of C. B is neither a preacher nor a swimmer. None of the
women is either a tap dancer or a swimmer. A and D are unmarried and
neither of them is a tap dancer or a preacher or a swimmer.
1. Which of the following people is E‘s wife?
a) A b) B c) C d) D
2. Who is the swimmer?
a) A b) E c) B d) C
3. Who is a tap dancer?
a) B b) A c) C d) D

Questions 4 - 8 are based on the following:


There are four persons A, B, C and D. One of them is a lecturer and
plays carrom and cricket. A and B are MBAs. A plays polo. Both the
MBAs are swimmers. D is a lawyer. One MBA also plays chess. The
lawyer plays carrom and is a swimmer. All four people play two puzzles
each and follow one profession.
4. Who is the lecturer?
a) A b) B c) C d) D

5. Which one of these people does not swim?


a) Lecturer b) MBA c) Lawyers d) Can‘t say
6. Who plays chess?
a) A b) B c) C d) D
7. Who plays polo and is an MBA?

CREA – All Rights Reserved I U Training Services


78
Analytical Puzzles Logical Ability

a) A b) B c) C d) D
8. What is the total number of puzzles played?
a) 5 b) 6 c) 4 d) 3

Questions 9 - 12 are based on the following:


There is an island called Kya-Kya. The inhabitants always answer any
question with two sentences; one of which is true and the other is
false.
9. You are looking for Jagarnath‘s house and you meet three people –
Amarnath, Ramnath and Somnath. You ask them ―Which is Jagarnath‘s
house?‖
Amarnath says: Jagarnath‘s house is No. 2, I am his
neighbour.
Ramnath says: Amarnath is not my neighbour. Amarnath and
Somnath live in the same house.
Somnath says: Jagarnath‘s house is not No. 2. Amarnath is
Ramnath‘s neighbour.
There are only two houses and four people in Kya-Kya. Two people live
in each house.
From the above, you can decide that
a) Jagarnath stays in house No. 2.
b) Jagarnath does not stay in house No. 2.
c) Jagarnath does not stay in Kya-Kya.
d) Ramnath and Somnath stay together.
10. Who stays with Amarnath ?
a) Ramnath b) Jagarnath c) Somnath d) Can‘t say.

11. The island has one professor. You talk to three men, Anand,
Pramod and Babloo. Anand you observe is wearing a green shoe.
Babloo says: Pramod‘s father is the professor. Pramod is not the
professor‘s son.

CREA – All Rights Reserved I U Training Services


79
Analytical Puzzles Logical Ability

Anand says: I am the professor. On this island only professors can


wear a green shoe.
Pramod says: I am the professor‘s son. Anand is not the professor.
Which of the following is true?
a) Pramod is not Anand‘s son. b) Anand is the professor.
c) Babloo is the professor. d) Pramod is the professor.

12. Based on the above, the professor has


a) one son b) two sons c) three sons d) Can‘t say.

Questions 13 - 15 are based on the following:


From exactly seven people – R, S, T, U, X, Y and Z – a group of
exactly four must be selected in accordance with the following
conditions:
If R is selected, T must also be selected.
If S is selected, U must also be selected.
If X and Y are both selected together, T cannot be selected.
13. If X and Y are both selected together, which of the following
must also be selected?
a) R b) S c) U d) Z
14. If S and Z are both selected, each of the following could also be
selected except:
a) R b) T c) U d) X

15. If U is not selected, which of the following can be, but does
not have to be, selected?
a) R b) S c) T d) X

Questions 16-19 are based on the following


At a martial arts school, the following six classes- A, B, H, J, K and L
are to be scheduled to meet for one demonstration each during an all-
day seminar. Each demonstration is 45 mins. long and the only possible

CREA – All Rights Reserved I U Training Services


80
Analytical Puzzles Logical Ability

starting times for the class meetings are 9 , 10, 11, a. m 2 and 4 p.m..
The schedule of the class meeting is to be established in accordance
with the following conditions.
A cannot meet at the same time that B meets.
H must meet earlier in the day then A
J must meet in the afternoon.
None of the other classes can meet at that time that K meets.
L can meet in the morning or in the afternoon.
16. Any of the following could meet at 4 p.m. EXCEPT
a) A b) B c) H d) J
17. If J meets sometime before K meets which of the following could
be true?
a) A meets some time after J meets b) B meets at 4 p.m.
c) K meets at 2 p.m. d) L meets at 2 p.m.
18. If A meets at 10 a.m. and none of the classes meet at 4 p.m. Which
of the following must be true?
a) H meets sometime before L meets
b) H meets sometime before K meets
c) K meets sometime before B meets
d) L meets at the same time that B meets.
19. If H meets at 2 p.m. which of the following must be true?
a) A meets later in the day than L.
b) B meets at 2 p.m.
c) B meets in the morning.
d) K meets in the morning.

CREA – All Rights Reserved I U Training Services


81
Analytical Puzzles Logical Ability

Questions 20 – 23
Exactly seven persons – P, Q, R, S, T U and V –
participate in and finish all of a series of swimming
races. There are no ties for any position at the finish of
any of the races.
V always finishes somewhere ahead of P.
P always finishes somewhere ahead of Q.
Either R finishes first and T finishes last, or S finishes first
and U or Q finishes last.
20. If in a race V finishes fifth, which of the following must be true ?
a) S finishes first. b) R finishes second.
c) T finishes third d) Q finishes fourth.
21. If in a race R finishes first, V can finish no lower than
a) second b) third c) fourth d) fifth
22. If in a race S finishes second, which of the following can be true?
a) P finishes before R. b) V finishes before S.
c) P finishes before V d) U finishes before V.
23. If in a race S finishes sixth and Q finishes fifth, which of the
following can be true?
a) V finishes first or fourth. b) R finishes second or third.
c) P finishes second or fifth. d) U finishes third or fourth.

Directions (Questions 24-28): Read the following information to


answer the given questions.
Six lectures A, B, C, D, E and F are to be organized in a span of seven
days – from Sunday to Saturday, only one lecture on each day in
accordance with the following:-
1. A should not be organized on Thursday.
2. C should be organized immediately after F.
3. There should be a gap of two days between E and D.

CREA – All Rights Reserved I U Training Services


82
Analytical Puzzles Logical Ability

4. One day there will be no lecture (Friday is NOT that day), just
before that day D will be organized.
5. B should be organized on Tuesday and should not be followed by
D.
24. How many are organized between C and D?
a) None b) One c) Two d) Three

25. Which of the following is the last lecture in the series?


a) A b) C c) B d) Cannot be determined.

26. Which information is NOT required in finding the complete


sequence of organization of lectures?
a) 1 only b) 2 only c) 1 and 2 only d) All are required.

27. Which day will the lecture F be organised?


a) Friday b) Saturday c) Sunday d) Thursday

28. On which day there is no lecture?


a) Sunday b) Friday c) Monday d) Cannot be
determined.

Directions (Questions 29-33): Read the following information to


answer the given questions.

P, Q, R, S, T, U and V occupied seven seats in a row in a seven-seater


bench.
1. U had as many persons to his left as S had to his right, but not
all the other persons in the group were to the left or right of
U.
2. Neither T nor P occupied a corner seat.
3. S sits between P and R.
4. V and U were both on the same side of S, but V was nearer to S
than U.

CREA – All Rights Reserved I U Training Services


83
Analytical Puzzles Logical Ability

5. T did not sit adjacent to P.


29. Who occupied the two corner seats?
a) Q & G b) U & G c) Q & R d) V & R

30. Who sat in the middle of the row?


a) Q b) P c) V d) T

31. Who amongst the following sat adjacent to T?


a) S b) R c) Q d) V

32. U sat between


a) Q & T b) T & V c) Q & V d) V & R

33. Who among the following was adjacent to P?


a) Q b) V c) U d) R

Directions (Questions 34-38): Read the following information to


answer the given questions.
1. A, B, C, D, E and F are six members of a family.
2. One couple has parents and their children in the family.
3. A is the son of C and E is the daughter of A.
4. D is the daughter of F, who is the mother of E.
34. Which of the following pairs is the parent of the couple?
a) AB b) BC c) AF d) CF

35. Who are the male members in the family?


a) C & F b) A, B & E c) A & C d) Cannot be
determined.

36. How many female members are in the family?


a) 2 b) 3 c) 4 d) Cannot be determined.

CREA – All Rights Reserved I U Training Services


84
Analytical Puzzles Logical Ability

37. Which of the following pairs is the parent of the children?


a) BC b) AF c) BF d) Cannot be determined.

38. What relationship do D and E bear to each other?


a) Mother & Son b) Sister & Brother
c) Grandmother & Granddaughter d) Sister

Directions (Questions 39-43): Read the following information


carefully and answer the questions given below.
Six persons A, B, C, D, E and F are sitting in two rows, three in each.
E is not at the end of any row.
D is second to the left of F.
C, the neighbour of E, is sitting diagonally opposite to D.
B is the neighbour of F.
39. Which of the following are sitting diagonally opposite to each
other?
a. F and C b. D and A c. A and C
d. A and F e. A and B

40. Who is facing B?


a. A b. C c. D d. E e. F

41. Which of the following are in the same row?


a. A and E b. E and D c. C and B
d. A and B e. C and F

42. Which of the following are in one of the two rows?


a. FBC b. CEB c. DBF
d. AEF e. ABF

43. After interchanging seat with F, who will be the neighbours of D in


the new positions?
a. C and A b. E and B c. Only B
d. Only A e. Only C

CREA – All Rights Reserved I U Training Services


85
Analytical Puzzles Logical Ability

Directions (Questions 44-48): Directions (Questions 1 to 5): Read


the following information carefully and answer the questions given
below it:
On a shelf are placed six volumes side by side labeled A, B, C,
D, E and F. Three volumes B, C and E have green covers while the
other volumes have yellow covers. A, D and B are new volumes while
the rest are old volumes. A, C and B are law reports, while the rest are
Gazetteers.
44. Which volume is new, yellow - covered and a Gazetteer?
a. B b. D c. C d. F e. None of these

45. Which two volumes are old Gazetteers and have green covers?
a. B, C b. B, E c. B, F d. E, F e. None of these

46. Which is green-covered, old and a law report volume?


a. A b. B c. C d. D e. E

47. Which is the yellow – covered new law report volume?


a. E b. D c. B d. C e. A

48. Which of the following is the old volume of a Gazetteer?


a. C b. D c. E d. B e. None of these

Directions (Questions 49-52): Directions (Questions 1 to 5): Study


the following informations carefully and answer the questions given
below it:
Five friends A, B, C, D and E are sitting on a bench
(1) A is sitting next to B.
(2) C is sitting next to D.
(3) D is not sitting with E.
(4) E is on the left end of the bench.
(5) C is on the second position from the right.
(6) A is on the right of B and E.
(7) A and C are sitting together.

CREA – All Rights Reserved I U Training Services


86
Analytical Puzzles Logical Ability

49. Where is A sitting?


a. Between B and D b. Between D and C c. Between E and D
d. Between C and E e. Between B and C

50. Who is sitting in the centre?


a. A b. B c. C d. D e. E

51. C is sitting between


a. B and D b. A and E c. D and E
d. A and D e. A and B

52. What is the position of D?


a. Extreme left b. Extreme right c. Third from left
d. Second from left e. None of these

53. What is the position of B?


a. Second from right b. Centre c. Extreme left
d. Second from left e. None of these

Directions (Questions 54-58): Directions (Questions 1 to 5): Read


the following information carefully and answer the questions given
below it:
In a car exhibition, seven different companies, viz. Cadilac,
Ambassador, Fiat, Maruti, Mercedes, Bedford and Fargo, were
displayed in a row, facing east such that:
(1) Cadilac car was to the immediate right of Fargo.
(2) Fargo was fourth to the right of Fiat.
(3) Maruti car was between Ambassador and Bedford.
(4) Fiat, which was third to the left of Ambassador car, was
at one of the ends.
54. Which of the following was the correct position of the Mercedes?
a. Immediate left of Cadilac b. Immediate left of Bedford
c. Between Bedford and Fargo d. Fourth to the right of Maruti
e. None of these

CREA – All Rights Reserved I U Training Services


87
Analytical Puzzles Logical Ability

55. Which of the following is definitely true?


a. Fargo car is between Ambassador and Fiat
b. Cadilac car is to the immediate left of Mercedes.
c. Fargo is to the immediate right of Cadilac.
d. Maruti is fourth to the right of Mercedes.
e. None of these

56. Which cars are on the immediate either sides of the Cadilac car?
a. Ambassador and Maruti b. Maruti and Fiat
c. Fiat and Mercedes d. Ambassador and Fargo
e. None of these

57. Which of the following is definitely true?


a. Maruti is to the immediate left of Ambassador.
b. Bedford is to the immediate left of Fiat.
c. Bedford is at one of the ends.
d. Fiat is second to the right of Maruti.
e. None of these

58. Which of the following groups of cars is to the right of the


Ambassador car?
a. Cadilac, Fargo & Maruti b. Maruti, Bedford and Fiat
c. Mercedes, Cadilac and Fargo d. Bedford, Cadilac and Fargo.
e. None of these.

Directions (Questions 59-63): Directions: Read the following


informations and answer the questions given below it:
(1) Seven students P, Q, R, S, T, U and V take a series of
tests.
(2) No two students get the same marks.
(3) V always scores more than P.
(4) P always scores more than Q.
(5) Each time either R scores the highest and T gets the
least, or alternatively S scores the highest and U or Q scores
the least.

CREA – All Rights Reserved I U Training Services


88
Analytical Puzzles Logical Ability

Questions:
59. If S is ranked sixth and Q is ranked fifth, which of the following
can be true?
a. V is ranked first or fourth b. R is ranked second or third.
c. P is ranked second or fifth d. U is ranked third or fourth.
e. T is ranked fourth or fifth.

60. If R gets most, V should be ranked not lower than


a. Second b. Third c. Fourth d. Fifth e. Sixth

61. If R is ranked second and Q is ranked fifth, which of the following


must be true?
a. S is ranked third b. T is ranked sixth
c. P is ranked sixth d. V is ranked fourth
e. U is ranked sixth

62. If S is ranked second, which of the following can be true?


a. U gets more than V b. V gets more than S
c. P gets more than R d. P gets more than V
e. T gets more than Q

63. If V is ranked fifth, which of the following must be true?


a. S scores the highest b. R is ranked second
c. T is ranked third d. Q is ranked fourth
e. U scores the least.

Directions (Q. 64-69): Study the following information and answer


the questions given below it:
A blacksmith has five iron articles A, B, C, D and E, each
having a different weight.
1) A weighs twice as much as B.
2) B weighs four and a half times as much as C
3) C weighs half as much as D.
4) D weighs half as much as E.

CREA – All Rights Reserved I U Training Services


89
Analytical Puzzles Logical Ability

5) E weighs less than A but more than C.


64. Which of the following is the lightest in weight?
a. A b. B c. C d. D e. E

65. E is lighter in weight than which of the other two articles?


a. A, B b. D, C c. A, C d. D, B e. B, E

66. Which of the above given statements is not necessary to


determine the correct order of articles according to their weights?
a. 1 b. 2 c. 3 d. 4 e. None of these

67. E is heavier than which of the following two articles?


a. D, B b. D, C c. A, C d. A, B e. None of these

68. Which of the following articles is the heaviest in weight?


a. A b. B c. C d. D e. E

69. Which of the following represents the descending order of


weights of the articles?
a. A, B, E, D, C b. B, D, E, A, C c. E, C, D, A, B
d. C, A, D, B, E e. A, B, D, E, C

Directions (Q. 70-74): Read the following information carefully and


answer the questions given below it:
Five persons are sitting in a row. One of the two persons at
the extreme ends is intelligent and the other one is fair. A fat person
is sitting on the right of a weak person. A tall person is on the left of
the fair person and the weak person is sitting between the intelligent
and the fat persons.
70. The tall person is at which place, counting from right?
a. First b. Second c. Third
d. Fourth e. None of these

71. Which of the following describes the person on the left of the
weak person?

CREA – All Rights Reserved I U Training Services


90
Analytical Puzzles Logical Ability

a. Intelligent b. Fat c. Fair d. Tall e. Cannot be determined

72. Which of the following persons is sitting at the centre?


a. Fair b. Weak c. Intelligent d. Tall e. Fat

73. On whose left is the fat person sitting?


a. Fair b. Intelligent c. Tall
d. Weak e. Cannot be determined.

74. If the fair person and the fat person exchange their positions,
and so also do the tall and the weak ones, then who will be sitting on
the left of the weak person?
a. Tall b. Fair c. Fat
d. Intelligent e. Cannot be determined.

Directions (Q. 75-78): Study the following information carefully and


answer the questions given below it:
Prashant Arora has three children – Sangeeta, Vimal and
Ashish. Ashish married Monika, the eldest daughter of Mr and Mrs
Roy. The Roys married their youngest daughter to the eldest son of
Mr and Mrs Sharma, and they have two children named Amit and
Shashi. The Roys have two more children, Roshan and Vandana, both
older than Veena. Sameer and Ajay are the sons of Ashish and
Monika. Rashmi is the daughter of Amit.
75. What is the surname of Rashmi?
a. Sharma b. Roy c. Arora
d. Cannot be determined e. None of these

76. How is Sameer related to the father of Monika?


a. Grandson b. Son c. Cousin d. Son-in-law e. None of
these

77. What is the surname of Sameer?


a. Roy b. Sharma c. Arora
d. Cannot be determined e. None of these

CREA – All Rights Reserved I U Training Services


91
Analytical Puzzles Logical Ability

78. How is Mrs Roy related to Ashish?


a. Aunt b. Mother-in-law c. Mother
d. Sister-in-law e. None of these

Directions (Q. 79-82): Study the following information and answer


the questions given below it:
1) Kailash, Govind and Harinder are intelligent.
2) Kailash, Rajesh and Jitendra are hardworking.
3) Rajesh, Harinder and Jitendra are honest.
4) Kailash, Govind and Jitendra are ambitious.
79. Which of the following persons is neither hardworking nor
ambitious?
a. Kailash b. Govind c. Harinder
d. Rajesh e. None of these

80. Which of the following persons is neither honest nor hardworking


but is ambitious?
a. Kailash b. Govind c. Rajesh
d. Harinder e. None of these

81. The cricket ball is lighter than the hockey ball and the volley ball is
lighter than the football. The hockey ball is lighter than the football
but heavier than the tennis ball. Which of the following is the
heaviest?
a. Hockey ball b. Cricket ball c. Foot ball
d. Volley ball e. None of these

82. Five children are sitting in a row. S is sitting next to P but not T.
K is sitting next to R, who is sitting on the extreme left and T is not
sitting next to K. Who are sitting adjacent to S?
a. K and P b. R and P c. Only P d. P and T
e. Insufficient information

CREA – All Rights Reserved I U Training Services


92
Analytical Puzzles Logical Ability

Directions (Q. 83-87): Read the following statements and answer the
questions that follow:
Of the six men of literature A, B, C, D, E and F being
considered here, two belonged to the 17th century, three to the 19th
and one to the 20th. Four were recognized as great poets, three as
great novelists and three as great dramatists. One contributed to
Bengali literature, two to Hindi, two to Marathi and one to Tamil. The
20th – century writer wrote poetry only and contributed to Marathi
literature and the other Marathi writer contributed to poetry, novel
and drama. One Hindi writer and the only Tamil writer belonged to
the 19th century. The former contributed to poetry and novel while
the latter to novel and drama. The Bengali writer belong to the 17th
century and contributed to poetry only. A belonged to the 20 th
century, B wrote drama only, C contributed to Marathi literature, D
was a Hindi poet and novelist and belonged to the 19th century. E also
belonged to the 19th century, and F contributed to poetry only.

83. To which language did B contribute?


a. Bengali b. Hindi c. Marathi d. Tamil e. None of these

84. Among these, who was the Tamil writer?


a. A b. B c. E d. F e. None of these

85. To which branch of literature did a contribute?


a. Poetry b. Novel c. Drama
d. All the three e. None of these

86. Among these, who was the Bengali writer?


a. A b. B c. E d. F e. None of these

87. To which branch of literature did C contribute?


a. Poetry b. Drama c. Novel
d. All the three e. None of these

CREA – All Rights Reserved I U Training Services


93
Analytical Puzzles Logical Ability

Directions (Q. 88-92): Read the following information carefully and


answer the questions that follow:
(i) There is a group of five persons A, B, C, D and E
(ii) One of them is a horticulturist, one is a physicist, one is a
journalist, one is an industrialist and one is an advocate.
(iii) Three of them, A, C and the advocate, prefer tea to
coffee and two of them, B and the journalist, prefer coffee to tea.
(iv) The industrialist and D and A are friends to one another
but two of them prefer coffee to tea.
(v) The horticulturist is C‘s brother.
88. Who is a horticulturist?
a. A b. B c. C d. D e. E

89. Who is an industrialist?


a. E b. C c. B d. D e. A

90. Which of the following groups includes a person who likes tea but
is not an advocate?
a. ACE b. DE c. BCE d. BD e. None of these

91. Who is a physicist?


a. A b. E c. D d. C e. B

92. Which of the statements above is superfluous?


a. (iii) b. (iv) c. (ii) d. (v) e. None

CREA – All Rights Reserved I U Training Services


94
2
Ready to Get Stumped!

CREA – All Rights Reserved I U Training Services


95
PRE-LESSON Challenge – How deep is your
sense of analysis?
Time Limit – 5 minutes
1. TWO ferryboats start a t the same instant from opposite
sides of a river, traveling across the water on routes at
right angles to the shores. Each travels a t a constant
speed, but one is faster than the other. They pass a t a
point 720 yards from the nearest shore. Both boats remain
in their slips for
10 minutes before starting back. On the return trips they
meet 400 yards from the other shore. How wide is the
river?
2. AN UNLIMITED SUPPLY of gasoline is available at one
edge of a desert 800 miles wide, but there is no source on
the desert itself. A truck can carry enough gasoline to go
500 miles (this will be called one "load"), and it can build up
its own refueling stations at any spot along the way. These
caches may be any size, and it is assumed that there is no
evaporation loss. What is the minimum amount (in loads) of
gasoline the truck will require in order to cross the desert?
Is there a limit to the width of a desert the truck can
cross?

CREA – All Rights Reserved I U Training Services


96
Ready to Get Stumped Logical Ability

In This Chapter
 Understand the world‘s toughest puzzles
 Solving strategies with detailed answers

1. THE RETURNING EXPLORER


AN OLD RIDDLE runs as follows. An explorer walks one mile due
south, turns and walks one mile due east, turns again and walks one
mile due north, He finds himself back where he started. He shoots a
bear. What color is the bear? The time-honored answer is: "White,"
because the explorer must have started a t the North Pole. But not
long ago someone made the discovery that the North Pole is not the
only starting point that satisfies the given conditions! Can you think of
any other spot on the globe from which one could walk a mile south, a
mile east, a mile north and find himself back at his original location?

2. THE EARLY COMMUTER


A COMMUTER I S in the habit of arriving a t his suburban station
each evening exactly at five o'clock. His wife always meets the train
and drives him home. One day he takes an earlier train, arriving at the
station a t four. The weather is pleasant, so instead of telephoning
home he starts walking along the route always taken by his wife. They
meet somewhere on the way. He gets into the car and they drive
home, arriving a t their house ten minutes earlier than usual. Assuming
that the wife always drives a t a constant speed, and that on this
occasion she left just in time to meet the five o'clock train, can you
determine how long the husband walked before he was picked up?

3. THE COUNTERFEIT COINS


IN RECENT YEARS a number of clever coin-weighing or ball-weighing
problems have aroused widespread interest. Here is a new and
charmingly simple variation. You have 10 stacks of coins, each
consisting of 10 half-dollars. One entire stack is counterfeit, but you
do not know which one. You do know the weight of a genuine half-dollar
and you are also told that each counterfeit coin weighs one gram more

CREA – All Rights Reserved I U Training Services


97
Ready to Get Stumped Logical Ability

than it should. You may weigh the coins on a pointer scale. What is the
smallest number of weightings necessary to determine which stack is
counterfeit?

4. TWO FERRY BOATS


TWO ferryboats start a t the same instant from opposite sides of a
river, traveling across the water on routes at right angles to the
shores. Each travels a t a constant speed, but one is faster than the
other. They pass a t a point 720 yards from the nearest shore. Both
boats remain in their slips for
10 minutes before starting back. On the return trips they meet 400
yards from the other shore. How wide is the river?

5. THE EFFICIENT ELECTRICIAN


AN electrician is faced with this annoying dilemma. In the basement
of a three-story house he finds bunched together in a hole in the wall
the exposed ends of 11 wires, all alike. In a hole in the wall on the top
floor he finds the other ends of the same 11 wires, but he has no way
of knowing which end above belongs to which end below. His problem:
to match the ends.

To accomplish his task he can do two things: (1) short-circuit the wires
a t either spot by twisting ends together in any manner he wishes; (2)
test for a closed circuit by means of a "continuity tester" consisting
of a battery and a bell. The bell rings when the instrument is applied
to two ends of a continuous, unbroken circuit. Not wishing to exhaust
himself by needless stair-climbing, and having a passionate interest in
operations research, the electrician sat down on the top floor with
pencil and paper and soon devised the most efficient possible method
of labeling the wires. What was his method?

6. THE AMOROUS BUGS

CREA – All Rights Reserved I U Training Services


98
Ready to Get Stumped Logical Ability

FOUR BUGS-A, B, C and D-


occupy the corners of a square
10 inches on a side [Fig. 531].
A and C are male, B and D are
female. Simultaneously A
crawls directly toward B, B
toward C, C toward D and D
toward A. If all four bugs
crawl at the same constant
rate, they will describe four
congruent logarithmic spirals
which meet a t the center of
the square. How far does each bug travel before they meet? The
problem can be solved without calculus.

7. THE FLIGHT AROUND THE WORLD


A GROUP of airplanes is based on a small island. The tank of each
plane holds just enough fuel to take it halfway around the world. Any
desired amount of fuel can be transferred from the tank of one plane
to the tank of another while the planes are in flight. The only source
of fuel is on the island, and for the purposes of the problem it is
assumed that there is no time lost in refueling either in the air or on
the ground. What is the smallest number of planes that will ensure the
flight of one plane around the world on a great circle, assuming that
the planes have the same constant ground speed and rate of fuel
consumption and that all planes return safely to their island base?

8. THE COLLIDING MISSILES


Two MISSILES speed directly toward each other, one a t 9,000 miles
per hour and the other a t 21,000 miles per hour. They start 1,317
miles apart. Without using pencil and paper, calculate how far apart
they are one minute before they collide.

9. MONKEY WITH COCONUTS

CREA – All Rights Reserved I U Training Services


99
Ready to Get Stumped Logical Ability

Three sailors come upon a pile of coconuts. The first sailor takes half
of them plus half a coconut. The second sailor takes half of what is
left plus half a coconut. The third sailor also takes half of what
remains plus half a coconut. Left over is exactly one coconut which
they toss to the monkey. How many coconuts were there in the original
pile? If you will arm yourself with 20 matches, you will have ample
material for a trial-and-error solution.

10. CROSSING THE DESERT


AN UNLIMITED SUPPLY of gasoline is available at one edge of a
desert 800 miles wide, but there is no source on the desert itself. A
truck can carry enough gasoline to go 500 miles (this will be called one
"load"), and it can build up its own refueling stations at any spot along
the way. These caches may be any size, and it is assumed that there is
no evaporation loss. What is the minimum amount (in loads) of gasoline
the truck will require in order to cross the desert? Is there a limit to
the width of a desert the truck can cross?

11. PROFESSOR ON THE ESCALATOR


WHEN Professor Stanislaw Slapenarski, the Polish mathematician,
walked very slowly down the down-moving escalator, he reached the
bottom after taking 50 steps. As an experiment, he then ran up the
same escalator, one step at a time, reaching the top after taking 125
steps. Assuming that the professor went up five times as fast as he
went down (that is, took five steps to every one step before), and that
he made each trip at a constant speed, how many steps would be
visible if the escalator stopped running?

12. THE ABSENT-MINDED TELLER


AN ABSENT-MINDED bank teller switched the dollars and cents
when he cashed a check for Mr. Brown, giving him dollars instead of
cents, and cents instead of dollars. After buying a five-cent
newspaper, Brown discovered that he had left exactly twice a s much a
s his original check. What was the amount of the check?

CREA – All Rights Reserved I U Training Services


100
Ready to Get Stumped Logical Ability

13. WHITE, BLACK AND BROWN


PROFESSOR MERLE WHITE of the mathematics department,
Professor Leslie Black of philosophy, and Jean Brown, a young
stenographer who worked in the university's office of admissions,
were lunching together.
"Isn't it remarkable," observed the lady, "that our last names are
Black, Brown and White and that one of us has black hair, one brown
hair and one white."
"It is indeed," replied the person with black hair, "and have you
noticed that not one of us has hair that matches his or her name?"
"By golly, you're right!" exclaimed Professor White. If the lady's hair
isn't brown, what is the color of Professor Black's hair?

14. THE PLANE IN THE WIND


AN AIRPLANE FLIES in a straight line from airport A to airport B,
then back in a straight line from B to A. It travels with a constant
engine speed and there is no wind. Will its travel time for the same
round trip be greater, less or the same if, throughout both flights, at
the same engine speed, a constant wind blows from A to B?

15. WHAT PRICE PETS?


THE OWNER of a pet shop bought a certain number of hamsters and
half that many pairs of parakeets. He paid $2 each for the hamsters
and $1 for each parakeet. On every pet he placed 2% retail price that
was an advance of 10 per cent over what he paid for it. After all but
seven of the creatures had been sold, the owner found that he had
taken in for them an amount of money exactly equal to what he had
originally paid for all of them. His potential profit, therefore, was
represented by the combined retail value of the seven remaining
animals. What was this value?

16. BEER SIGNS ON THE HIGHWAY


SMITH DROVE a t a steady clip along the highway, his wife beside
him. "Have you noticed," he said, "that those annoying signs for Flatz
beer seem to be regularly spaced along the road? I wonder how far

CREA – All Rights Reserved I U Training Services


101
Ready to Get Stumped Logical Ability

apart they are." Mrs. Smith glanced a t her wrist watch, then counted
the number of Flatz beer signs they passed in one minute. "What an
odd coincidence!" exclaimed Smith. "When you multiply that number by
ten, it exactly equals the speed of our car in miles per hour."
Assuming that the car's speed is constant, that the signs are equally
spaced and that Mrs. Smith's minute began and ended with the car
midway between two signs, how f a r is it between one sign and the
next?

17. TRICKY TRACK


THREE HIGH SCHOOLS -Washington, Lincoln and Roosevelt -
competed in a track meet. Each school entered one man and one only,
in each event. Susan, a student at Lincoln High, sat in the bleachers to
cheer her boy friend, the school's shot-put champion. When Susan
returned home later in the day, her father asked how her school had
done.
"We won the shot-put all right," she said, "but Washington High won
the track meet. They had a final score of 22. We finished with 9. So
did Roosevelt High."
"How were the events scored?" her father asked.
"I don't remember exactly," Susan replied, "but there were a certain
number of points for the winner of each event, a smaller number for
second place and a still smaller number for third place. The numbers
were the same for all events." (By "number" Susan of course meant a
positive integer.)
"How many events were there altogether?"
"Gosh, I don't know, Dad. All I watched was the shot-put.'"
"Was there a high jump?" asked Susan's brother.
Susan nodded. "Who won it?" Susan didn't know. Incredible as it may
seem, this last question can be answered with only the information
given. Which school won the high jump?

18. HOW DID KANT SET HIS CLOCK?


IT IS SAID that Immanuel Kant was a bachelor of such regular habits
that the good people of Koiligsberg would adjust their clocks when

CREA – All Rights Reserved I U Training Services


102
Ready to Get Stumped Logical Ability

they saw him stroll past certain landmarks. One evening Kant was
dismayed to discover that his clock had run down. Evidently his
manservant, who had taken the day off, had forgotten to wind it. The
great philosopher did not reset the hands because his watch was being
repaired and he had no way of knowing the correct time. He walked to
the home of his friend Schmidt, a merchant who lived a mile or so
away, glancing at the clock in Schmidt's hallway a s he entered the
house. After visiting Schmidt for several hours Kant left and walked
home along the route by which he came. As always, he walked with a
slow, steady gait that had not varied in twenty years. He had no notion
of how long this return trip took. (Schmidt had recently moved into
the area and Kant had not yet timed himself on this walk.)
Nevertheless, when Kant entered his house, he immediately set his
clock correctly. How did Kant know the correct time?

19. A TENNIS MATCH


MIRANDA beat Rosemary in a set of tennis, winning six games to
Rosemary's three. Five games were won by the player who did not
serve. Who served first?

CREA – All Rights Reserved I U Training Services


103
3
Get Your Figures Right!

CREA – All Rights Reserved I U Training Services


104
PRE-LESSON Challenge – How deep is your
sense of analysis?
Time Limit – 5 minutes
1. A contractor had employed 100 labourers for flyover
construction task. He did not allow any woman to work
without her husband. Also, at least half the men working
came with their wives. He paid five rupees per day to each
man, four rupees to each woman and one rupee to each child.
He gave out 200 rupees every evening. How many men,
women and children were working with the constructor?
2. Assume that you have just heard of a scandal and you are
the first one to know. You pass it on to four persons in a
matter of 30 minutes. Each of these four in turn passes it
to four other persons in the next 30 minutes and so on. How
long will it take for everybody in the world to get to know
the scandal? Assume that nobody hears it more than once
and the population of the world is approximately 5.6 billion.

CREA – All Rights Reserved I U Training Services


105
Get Your Figures Right! Logical Ability

The puzzles as you must‘ve noticed have been divided into two sections
as the analytical and mathematical puzzles. The mathematical puzzles
have got a lot to do with numbers, equations and mathematical
concepts such as permutation and combination, probability, profit and
loss, percentages. In this chapter we are going to solve some of the
puzzles that were asked in the recruitment tests conducted by some
companies and were mathematics based.

For example, let‘s solve this puzzle that was asked in Infosys
Technologies.

There is a 50m long army platoon marching ahead. The last person
in the platoon wants to give a letter to the first person leading
the platoon. So, while the platoon is marching he runs ahead and
hands over the letter to him and without stopping he runs and
comes back to his original position. In the mean time the whole
platoon has moved ahead by 50m. How much distance did the last
person cover in that time? Assume that he ran the whole distance
with uniform speed.

It is given that the platoon and the last person moved with
uniform speed. Also they both moved for the same time. Hence, the
ratio of the distance they covered – while person moving forward and
backward – are equal. Let‘s assume that when the last person reached
the first person, the platoon moved some x metres forward. Thus,
while moving forward the last person moved (50 + x) metres whereas,
the platoon moved x metres.
Similarly, while moving back the last person moved {50 – (50 – x)}
metres whereas, the platoon moved (50-x) metres in the same time.
Now, as the ratios of time taken are equal, we get,
(50 + x) / x = {50 – (50 – x)} / (50 + x)
(50 + x) / x = x / (50 + x)
we get, x2 = (50 + x) (50 – x) or x = 35.355 meters

CREA – All Rights Reserved I U Training Services


106
Get Your Figures Right! Logical Ability

Therefore the total distance covered by the last person = 50


+ x + x = 50 + 2 (35.355) = 120.71 metres
Please note that at first glance, one might think that the total
distance covered by the last person is 100 metres, as he ran the total
length of the platoon (50 m) twice. True, but that‘s the relative
distance covered by the last person assuming the platoon is stationary.

Let‘s look at another example where we will apply the concept of linear
equations. This puzzle is from Infosys again.

A contractor had employed 100 labourers for flyover construction


task. He did not allow any woman to work without her husband.
Also, at least half the men working came with their wives. He paid
five rupees per day to each man, four rupees to each woman and
one rupee to each child. He gave out 200 rupees every evening.
How many men, women and children were working with the
constructor?

Let‘s assume that there were X men, Y women and Z children


working with the constructor. Converting this information into an
equation we get, X + Y + Z = 100
Also, he pays Rs. 5, Rs. 4 and Re. 1 to a man, woman and a child
respectively. We get the following euation.
5X + 4Y + Z = 200
Eliminating X and Y in turn from these equations, we get
X = 3Z – 200
Y = 300 – 4Z
We know that if a woman works then, her husband also works. At least
half the men working came with their wives. This means the value of Y
lies between X and X/2. Substituting these limiting values in the above
equations we get,
If Y = X
300 – 4Z = 3Z – 200 or 7Z = 500 or Y
= 500 / 7 = 71.428
If Y = X/2

CREA – All Rights Reserved I U Training Services


107
Get Your Figures Right! Logical Ability

300 – 4Z = (3Z - 200) / 2 or 600 – 8Z = (3Z -


200) / 2 or
Z = 800 / 11 = 72.727
Now, Z must be an integer, hence Z = 72. Also, X = 16 and Y = 12.
Therefore, there were 16 men, 12 women, and 72 children working
with the constructor.
Therefore, we see here that approximation is an excellent tool to
derive the answer sometimes!

Another one!

There is a non – zero number that is 5 times the sum of its digits.
What is this number?

The number is 45, simply because 45 = 5 (4 + 5). How does one find
this number?
Let T be the digit in the ten‘s place and U be the digit in the unit‘s
place. Then, the number is 10T + U, and the sum of the digits is T + U.
The following equation can be readily written:
10T + U = 5 (T + U)
5T = 4U
Thus, T / U = 4 / 5. Since T and U are digits, T must be 4 and U must
be 5.
Let‘s solve this one based on sequences and series.
Assume that you have just heard of a scandal and you are the
first one to know. You pass it on to four persons in a matter of
30 minutes. Each of these four in turn passes it to four other
persons in the next 30 minutes and so on. How long will it take for
everybody in the world to get to know the scandal? Assume that
nobody hears it more than once and the population of the world is
approximately 5.6 billion.
You came to know of the scandal and you passed it on to 4 persons in
30 minutes. So, total 1 + 4 = 5 persons would know about it in 30
minutes. By the end of one hour, 16 more persons would know about it.
So, total of 1 + 4 + 16 = 21 persons would know about it in one hour.

CREA – All Rights Reserved I U Training Services


108
Get Your Figures Right! Logical Ability

Similarly, the other 1 + 4 + 16 + 64 = 85 persons would know about it in


one and a half hours and so on…
It can be deduced that the terms of the above series is the power of
4 that is, 40, 41, 42, 43 and so on up to 2n + 1 terms. Also the last term
would be 42n where n is the number of hours. Sum of the above
mentioned series = {4(2n + 1) – 1} / 3
The sum of the series must be 5.6 billions. Hence, equating the sum of
the series with 5.6 billions, we get n = 8 hours.
Scandals travel Fast!! Don‘t they?

Brain Drill
1. Everyday in his business a merchant had to weigh amounts from 1 kg
to 121 kg, to the nearest kg. What is the minimum number of weights
required and how heavy should they be so that he can weigh any
weight between 1 and 121 kg?

2. Two trains start from stations A and B spaced 50 km apart at the


same time and speed. As the trains start, a bird flies from one train
towards the other and on reaching the second train, it flies back to
the first train. This is repeated till the trains collide. If the speed of
the trains is 25 km/h and that of the bird is 100km/h. How much did
the bird travel till the trains collided?

3. From a vessel, 1/3rd of the liquid evaporates on the first day. On


the second day 3/4th of the remaining liquid evaporates. What
fraction of the volume is present at the end of the second day?

4. There is a 4 inch cube painted on all sides. This is cut down into of 1
inch cubes. What is the number of cubes which have no painted sides?

5. A family went for a vacation. Unfortunately it rained for all the 13


days when they were there. But whenever it rained in the mornings,
they had clear afternoons and vice versa. In all they enjoyed 11

CREA – All Rights Reserved I U Training Services


109
Get Your Figures Right! Logical Ability

mornings and 12 afternoons. How many days did they stay there
totally?

6. My husband's age contains the same two digits as my age but in


reverse order. The difference between our ages is 1/11 of their sum.
Also, my husband is older than me. What are our ages?

7. There is a certain 5 digit no.


a) it contains two primes. b) third digit is highest.
c) second digit is lowest.
d) fifth digit lies between first & second digits & is half of
fourth digit.
e) first digit is more than the sum of fourth & fifth digits.
What is the no?

8. A tank has two taps and a waste pipe. The two taps fill the tank
individually in 10 min and quarter of an hour. The waste pipe empties
the tank in 7.5 min. If both the taps as well as the waste pipe are kept
open, when (if ever) will the tank fill?

9. At 6'o clock a clock ticks 6 times. The time between first and last
ticks is 30 seconds. How long does it tick at 12'o clock.

10. Three friends divided some bullets equally. After all of them had
shot 4 bullets, the total number of bullets that remain was equal to
the bullets each had after division. Find the original number divided.

9. A said to B ―Now you should have at least 30 clerks in your


company‖. B replied ―If there is twice the number of the female clerks
or thrice the number of the male clerks, the number will be 30. We
shall expect that number in another 2 or 3 years.‖ Find the total
number of clerks in B‘s company.

10. There is an escalator moving downwards. A takes 50 steps and B


takes 75 steps to reach ground floor. It is given that the time taken

CREA – All Rights Reserved I U Training Services


110
Get Your Figures Right! Logical Ability

by A to take 1 step is equal to time taken by B to take 3 steps. Find


the number of steps in the escalator while it is stationary.

11. If the digits of my present age are reversed then I get the age of
my son. If one year ago my age was twice that of my son, find my
present age.

12. There are 6561 balls. All of them weigh the same except one ball
that is heavier than the others. Find the minimum number of times the
balls have to be weighed for finding out the heavy ball.

13. Two aging mathematics professors, Vishram and Ramesh were


sitting at their favourite fastfood restaurant, Pick n Move,
reminiscing about when they first met, while their waiter, Ramanuj
(who also took Professor Vishram‘s Advanced Mathematics course at
Osmania University) eavesdropped on their conversation as he brought
them their meals.
‗You know what occurred to me the other day Ramesh?‘ asked
Professor Vishram. ‗What‘s that?‘ said Professor Ramesh.
‗Well, when we met, the square of your age contained the same three
digits as the square of my age, just in a different order,‖ Professor
Vishram continued.
‗This will really blow your socks off then Vishram! If you take the
square of the sum of our ages when we met and split it into two 2—
digit numbers, you‘d have my age then and your age now!‖ Professor
Ramesh exclaimed.
Wanting to impress his professor, Ramanuj interjected.
―I thought you were much younger than that Professor Vishram,‖
Ramanuj said. Said Professor Vishram, ―And just how old am I then,
young fellow?‖

14. There are 10 socks of each of the following colours in a drawer:


blue, green, red, yellow and white, for a total of 50 socks. If the socks
are randomly distributed in the drawer (i.e. not in pairs or any other
grouping), and you are blindfolded, what is the minimum number of

CREA – All Rights Reserved I U Training Services


111
Get Your Figures Right! Logical Ability

socks you must draw from the drawer in order to be certain you have
at least 2 socks of the same colour?

15. Two trains, each 2 km long, enter two 1 km long tunnels that are 2
km apart from each other on the same track. The trains enter the
tunnels at exactly the same time. The first train is going at 5 kmph,
and the second train is going at 10 kmph. What is the sum of the
lengths of the two trains that will protrude from the tunnels at the
exact moment that they collide; assuming that neither train changes
its speed prior to collision? The trains are on the same track headed
in opposite directions (i.e. directly toward each other).

16. Madan rents a private car for a Abids—Hitec City—Abids trip. It


costs him Rs.300 everyday.
One day the car driver informed Madan that there were two students
from Sanatnagar who wished to go from Sanatnagar to Hitec City and
back to Sanatnagar. Sanatnagar is halfway between Abids and Hitec
City. Madan asked the driver to let the students travel with him.
On the first day when they came, Madan said, ―If you tell me the
mathematically correct price you should pay individually for your
portion of the trip, I will let you travel for free.‖
How much should the individual student pay for his journey?

Super Brain Drill


1. A boy has as many sisters as brothers, but each sister has only half
as many sisters as brothers. How many brothers and sisters are there
in the family?

2. When my father was 31 I was 8. Now he is twice as old as I am. How


old am I?

3. A book costs Rs. 100/- plus half its price. How much does it cost?

CREA – All Rights Reserved I U Training Services


112
Get Your Figures Right! Logical Ability

4. Two cyclists began a training run simultaneously, one starting from


Bangalore, the other from Mangalore. When the riders were 180 miles
apart, a drone was made to fly between them. Starting on one cyclist‘s
shoulder, the drone flew ahead to meet the other cyclist. On reaching
the latter, the drone at once turned back. The restless drone
continued to shuttle back and forth until the cyclists met; then it
settled on the nose of one of the cyclists.
The drone‘s speed was 30 miles per hour. Each cyclist‘s speed was 15
miles per hour. How many miles did the drone cover?

5. When was the latest year that looked the same upside down on a
calculator screen?

6. How many pluses should we put between the digits of 987,654,321


to get a total of 99, and where?

7. A house has 6 stories, each the same height. How many times as
long is the ascent to the sixth floor as to the third?

8. Each morning Krishna walks to school. At one-fourth of the way he


passes the church tower; at one-third of the way, the railway station.
At the church tower its clock shows 7.30 and at the railway station its
clock shows 7.35.
When does Krishna leave his house, when does he reach school?

9. Every time young Vaivhav sees a stray kitten he picks up the animal
and brings it home. He is always raising several kittens, but he won‘t
tell you how many because he is afraid you may laugh at him.
Someone will ask: ―How many kittens do you have now?‖
―Not many,‖ he answers. ―Three-quarters of their number plus three-
quarters of a kitten.‖
His friends thin he is joking. But he is really posing a problem- an easy
one though.

CREA – All Rights Reserved I U Training Services


113
Get Your Figures Right! Logical Ability

10. A balance has only two weights, 1 ounce and 4 ounces. In three
weighing, split 180 ounces of grits into two packages of 40 and 140
ounces.

11. One invention saves 30% fuel; a second, 45%; and a third, 25%. If
you use all three inventions at once can you save 100%? If not, how
much?

12. A cubical box contains 27 congruent balls; its twin contains 64


smaller congruent balls. All the balls are made of same material. Both
boxes are filled to the top. In each box, each layer has the same
number of balls, and the outside balls of each layer touch the sides.
Which box is heavier?

13. Transposing the two digits of A‘s age gives B‘s age. The difference
between their ages is twice C‘s age and B is ten times as old as C.
What are the three ages?

14. Three brothers shared 24 apples, each getting a number equal to


his age 3 years before. The youngest one proposed a swap:
―I will keep only half the apples I got, and divide the rest between you
two equally. But then the middle brother, keeping half his accumulated
apples, must divide the rest equally between the oldest brother and
me, and then the oldest brother must do the same.‖
They agreed. The result was that each ended with 8 apples. How old
were the brothers?

15. Mother asked Aaina to type a manuscript. Aaina decided: ―I will


type an average of 20 pages a day.‖ She typed the first half of the
manuscript rather lazily, at 10 pages per day. To make up for it, she
typed the second half at 30 pages a day.
―See, I did average 20 pages a day,‖ Vera concluded. ―Half of 10 + 30
is 20.‖ ―No, you didn‘t,‖ her mother said. Who was right?

More Puzzles will be done during the sessions

CREA – All Rights Reserved I U Training Services


114
4
Know Thyself

CREA – All Rights Reserved I U Training Services


115
SAO Logical Ability

Self Assessment Opportunity


Topic – Analytical Puzzles
No of Questions: 40 Time: 30 min

Directions (Q. 1-3): Read the following information carefully and


answer the questions that follow:
A college has a 5–day week, from Monday to Friday. On each day
there are two periods, period 1 and period 2. There are 5-subjects,
Maths, Physics, Chemistry, English and Hindi, and each subject has
two periods in a week. Most of the lecturers are part-time, and
lecturers have to be scheduled keeping in mind availability of
lecturers.

o Maths and Physics lectures are scheduled only for period 1, and
Maths lectures cannot be scheduled on two consecutive days.
Maths is first lecture of the week.
o An English lecture is scheduled for period 1 on Thursday.
o Both the Physics and the Hindi lectures are scheduled on the
same days.
o The second lecture of Hindi is scheduled for period 2 on Friday.
o Only Chemistry has lectures on two consecutive days.

1. Which subject is scheduled for period 2 on Monday?


1. English 2. Maths 3. Hindi 4.
Chemistry

2. Hindi is scheduled on
1. Thursday and Friday 2. Friday and Wednesday
3. Tuesday and Wednesday 4. Tuesday and Friday

3. Which lectures are scheduled for the period 2 on Wednesday and


Thursday?
1. Hindi and Chemistry 2. Chemistry only
3. Hindi only 4. English and Hindi

CREA – All Rights Reserved I U Training Services


116
SAO Logical Ability

Directions (Q. 4-6): Read the following information carefully and


answer the questions that follow:
There are seven chairs numbered from 1 to 7 placed in a row, from
left to right. Seven persons A, B, C, D, E, F and G are seated in chairs
numbered 1 to 7, not necessarily in that order.
o C sits on the chair at the extreme left.
o G sits on an odd numbered chair and has only one neighbour E.
o D sits on an even numbered chair and has B and F for
neighbours.
o F sits on chair numbered 5.

4. Who sits on chair numbered 2?


1. C 2. A 3. B 4. E

5. E sits on chair numbered


1. 2 2. 5 3. 6 4. None of these

6. Which of the following are seated on three consecutively


numbered chairs?
1. A, B, D 2. B, D, G 3. C, A, D 4. D, F, G

Directions (Q. 7-8): Read the following information carefully and


answer the questions that follow:
Three flights A, B, C of 9:00 a.m., 12:30 p.m. and 5:00 p.m., not
necessarily in that order. These flights are from P to Q and from Q
to P. No two flights with the same flight name start simultaneously
from both the cities. It takes 6 hours to reach city Q from city P.
Flight A starts at 9:00 a.m. from city P. Flight B arrives in city P at
11: 00 p.m.

7. Which flight starts at 9:00 a.m. from city Q?


1. C 2. B 3. Q 4. None of these

8. When does flight A arrive in city P?

CREA – All Rights Reserved I U Training Services


117
SAO Logical Ability

1. 5:30 p.m. 2. 6:30 p.m. 3. 11:00 pm. 4. 3:00 p.m.

Directions (Q. 9-12): Read the following information carefully and


answer the questions that follow:
Seven meetings – J, K, L, M, N, O and P – are to be scheduled, one on
each day of a week that begins on Sunday.

o Meeting J must take place on Sunday


o Meeting K must take place after both meeting Land meeting M.
o Meetings N, O and P must take place on three consecutive days,
not necessarily in that order.

9. Which is the latest day of the week on which meeting L can take
place?
1. Tuesday 2. Wednesday 3. Thursday 4. Friday

10. Which of the following must be true about the order of the
meetings?
1. L takes place after J 2. L takes place after O
3. N takes place after O 4. N takes place after P

11. If meeting O is on Saturday, then meeting K must take place on


1. Monday 2. Tuesday 3. Wednesday 4. Thursday

12. Which of the following represents a possible order of meetings on


three consecutive days?
1. JMK 2. KLO 3. MNJ 4. POM

Directions (Q. 13-16): Read the following information carefully and


answer the questions that follow:
Two cities A and B are connected by a highway. Between these two
cities there are four small cities C, D, E and F on the same high-way.
Distance between any two adjacent cities is the same. People from
cities A and B go to the small cities on business. It takes 15 minutes
to travel between any two adjacent cities. At each intermediate city

CREA – All Rights Reserved I U Training Services


118
SAO Logical Ability

buses stop for 5 minutes. A bus starting from city A t 8:00 a.m.,
reaches city F at 8:55 a.m. The actual traveling time, that is not
considering stoppages, from city B to city E is 1 hour. City D is nearer
to city B than city A.

13. A bus leaves city B at 9:12 a.m. At what time does it reach city C?
1. 9:27 a.m. 2. 10.07 a.m. 3. 9:32 a.m. 4. 10:00 a.m.

14. If a bus starting from city A reaches city D at 10:15 a.m., at what
time did it leave city E?
1. 9:20 a.m. 2. 9:35 a.m. 3. 9:15 a.m. 4. None of these

15. The city farthest from city A is


1. F 2. D 3. E 4. B

16. The city nearest to A is


1. E 2. D 3. C 4. F

Questions 17 to 20

Preetham, Raghu, Qadir and Sudhir work as lab in – charges for


―Fanbaxy labs‖, from Monday to Saturday, Each day has two sessions,
morning and evening, from Monday to Friday, and on Saturday there is
only one morning session. Study the following information:

o In a session only one lab in-charge works.


o Except Preetham, every person works for 3 sessions in a week
o Preetham works in morning sessions only, and that too only on
Thursdays and Saturdays.
o Qadir works in 3 consecutive morning sessions.
o Raghu works only in evening sessions, but not on Tuesdays and
Saturdays.
o Sudhir always works on Thursdays.

17. Who works in the session after Qadir‘s on Tuesday?

CREA – All Rights Reserved I U Training Services


119
SAO Logical Ability

1. Raghu 2. Preetham 3. Sudhir 4. None of these

18. Which of the following can never work on the same day?
1. Qadir, Raghu 2. Preetham, Sudhir
3. Qadir, Sudhir 4. Preetham, Qadir

19. Raghu works on


1. Monday, Wednesday, Friday 2. Tuesday, Wednesday, Friday
3. Monday, Tuesday, Saturday 4. Wednesday, Thursday, Friday

20. Sudhir works on


1. Tuesday, Wednesday, Thursday 2. Tuesday, Thursday, Friday
3. Tuesday, Wednesday, Friday 4. Thursday, Friday, Saturday

Questions 21 to 23

Aamir, Bobby, Chithra, Deepti and Esha are 5 prize winners who have
to be scheduled in such a manner that

o Chithra should be the third to go on stage.


o Bobby should not either precede or succeed Deepti.
o Aamir should be the first to go on stage.

21. If Esha is the last person to go on stage, then Bobby will be


_______ to go on stage.
1. second or fourth 2. first or third
3. third or fourth 4. fourth or fifth

22. If Bobby is the second person to go on stage, then Deepti will be


_________ to go on stage.
1. second or fourth 2. first or third
3. fourth or fifth 4. third or fourth

23. If Deepti is the fourth person to go on stage, then the correct


order of persons going on stage after Aamir is

CREA – All Rights Reserved I U Training Services


120
SAO Logical Ability

1. B, C, D, E 2. E, C, D, B 3. B, E, C, D 4. None of these

Questions 24 to 27

Sweta is a marketing officer. She has to visit six different areas A,


B, C, D, E, F within the city, each on a different day of a week starting
Monday.

o She will visit the farthest area first.


o She will visit area C after both areas D and E, but immediately
before B.
o She will visit area F the last.
o She will visit area A on Friday.
o There will be a gap of exactly two days between visits to areas
A and E.

24. Sweta will first visit area


1. E 2. D 3. C 4. B

25. Sweta will visit area E on


1. Tuesday 2. Thursday 3. Friday 4. Wednesday

26. On Wednesday Sweta will visit area


1. B 2. A 3. C 4.None of these

27. The area which is the farthest is


1. F 2. E 3. B 4. D

Questions 28 to 31

Mr. Anthony, Mr. Botham, Ms. Carol, Mr. Dennis and Ms. Emma work
for the same company. Their cabins are in a row numbered from 1 to
5.

CREA – All Rights Reserved I U Training Services


121
SAO Logical Ability

o There is a married couple in the group, and the couple sit in the
cabins at the extreme ends.
o Carol sits in the cabin numbered 4, and she has a lady
neighbour.
o Anthony sits in the cabin which is between Botham‘s and Dennis.
o Dennis also has a lady neighbour.

28. Who sits in the cabin numbered 3?


1. Botham 2. Dennis 3. Anthony 4. Emma

29. Emma sits in the cabin numbered


1. 5 2. 1 3. 2 4. None of these

30. The married couple in the group is


1. Anthony-Carol 2. Dennis-Carol
3. Botham-Emma 4. Anthony-Emma

31. How many cabins are there between Emma‘s and Anthony‘s?
1. 1 2. 3 3. 0 4. 2

Questions 32 to 35

A, B, C, D, E, F are professionals, with a lawyer, a doctor, an engineer,


a manger, a professor and a pilot among them. They are seated around
a round table, on chairs numbered from 1 to 6.
o C sits on chair numbered 5, and is not a lawyer.
o F is a Pilot.
o D‘s profession starts with the first letter of his name, and he
sits on chair numbered 4.
o Manager B and professor A are on chairs numbered 1 and 6
respectively.
o The lawyer is on chair numbered 3.

32. On which numbered chair is F seated?


1. 2 2. 3 3. 4 4. 5

CREA – All Rights Reserved I U Training Services


122
SAO Logical Ability

33. Who is the Lawyer?


1. A 2. C 3. D 4. E

34. What is C‘s profession?


1. Engineer 2. Doctor 3. Lawyer 4. Manager

35. Who are seated on chairs numbered 3, 4 and 5?


1. E, D, C 2. D, C, A 3. B, F, E 4. F, E, D

Questions 36 to 38

There are five houses in a row, left to right, coloured Blue, Green,
Yellow, Orange and Red not necessarily in that order. The owners of
the houses are Avinash, Babu, Chandu, Deepak and Eshwar.
o Eshwar stays in the last house from the left, which is not
coloured orange.
o The green house is immediately to the right of the house
coloured blue, which in turn is the first house in the row.
o Chandu stays in the house coloured yellow, which is in the
middle of the row.
o Babu‘s house is in between Avinash‘s and Chandu‘s.

36. In which coloured house does Eshwar stay?


1. Blue 2. Green 3. Orange 4. Red

37. Who lives in the house coloured orange?


1. Avinash 2. Babu 3. Chandu 4. Deepak

38. Avinash and Babu stay in the houses coloured


1. Blue & Green 2. Green & Yellow
3. Yellow & Orange 4. Orange & Red

Questions 39 and 40

CREA – All Rights Reserved I U Training Services


123
SAO Logical Ability

Arun, Bhaskar, Chandra, Damodar, Esmail and Farooq are seated in a


row on chairs numbered 1 to 6, not necessarily in the order.
o Arun has only one neighbour Chandra
o Farooq sits between Damodar and Esmail.
o Damodar sits on chair numbered 5.
39. Bhaskar sits on chair numbered
1. 1 2. 2 3. 6 4. None of these

40. Who sits on chair numbered 1?


1. Arun 2. Chandra 3. Bhaskar 4. None of these

CREA – All Rights Reserved I U Training Services


124
Number Math

CREA – All Rights Reserved I U Training Services 125


1
Improving Your
Percentages

CREA – All Rights Reserved I U Training Services 126


PRE-LESSON Challenge – See What You Know and
Remember
Time Limit – 5 minutes

1. Two numbers are respectively 35% and 20% more than a


third. What percentage is the first of the second?
2. After deducting 10% from a certain sum, and then 20%
from the remainder, there is Rs.3600 left. Find the
original sum.
3. A man had Rs.4800 in his locker two years ago. In the
first year, he deposited 20% of the amount in his locker.
In the second year, he deposited 25% of the increased
amount in his locked. Find the amount at present in his
locker.
4. If the price of sugar falls down by 10%, by how much
per cent must a householder increase its consumption, so
as not to decrease expenditure in this item?
5. A student has to secure 40% marks to get through. If
he gets 40 marks and fails by 40 marks, find the maximum
marks set for the examination.

CREA – All Rights Reserved I U Training Services 127


Improving Your Percentages Number Math

In This Chapter
 Switching from fractions to decimals to percents and back again
 Investigating both the practical and impractical with percents
 Using percentages to your advantage — in your best interest

Decimals and percents are really just fractions — in a more


manageable format. Doing problems that involve percents of things
involves changing the percents to decimals and then doing the
indicated operations. That‘s not a big deal, if you handle the
decimals correctly. And the computations are much easier than with
fractions, which can have very uncooperative denominators.
In this chapter, you see how to figure percent increase and percent
decrease and determine whether what you see advertised is a good
deal.
The usual move from fractions to percents is through decimals —
the decimal format is the middleman in the process. You probably
already know some of the more common equivalences of percents
and fractions. You know that 50 percent is equivalent to 1⁄2 and 25
percent is equivalent to 1⁄4. Well, I‘m assuming that you know this
and having some of them in mind as you‘re working on percentage
problems is helpful.
The term per cent means ‗for every hundred‘. It can best be
defined as:
―A fraction whose denominator is 100 is called a percentage, and the
numerator of the fraction is called the rate per cent‖.

If two values are respectively x% and y% more than


100  x
a third value, then the first is the 100%
100  y
of the second.
Instance: Two numbers are respectively 35% and 20% more than a
third. What percentage is the first of the second?

CREA – All Rights Reserved I U Training Services 128


Improving Your Percentages Number Math

Explanation: Following the above Precept, we have the


135
required value   100  112.5% .
120
If A is x% of C and B is y% of C, then A is
x
100% of B.
y
Instance: Two numbers are respectively 20% and 25% of a third
number. What percentage is the first of the second?
Explanation: Following the above Precept, we have the required value
20
  100  80%
25

Percentage Expenditures and Saving


1
Instance: A man loses 12 % of his money and, after spending
2
70% of the remainder, he is left with Rs.210. How much had he at
first? Explanation: Let the man be supposed to have Rs. x at first.
1 1 x 7x
After losing 12 % or , he is left with x   Rs.
2 8 8 8
After spending 70% of the money, he is left with 30%
of the remainder,
7x 3 210  10  8
  210   Rs.800
8 10 3 7

x% of a quantity is taken by the first, y% of the


remaining is taken by the second and z% of the
remaining is taken by third person. Now, if A is left
in the fund, then there was

CREA – All Rights Reserved I U Training Services 129


Improving Your Percentages Number Math

A 100 100 100


in the beginning.
(100  x)(100  y )(100  z )
Instance: After deducting 10% from a certain sum, and then 20%
from the remainder, there is Rs.3600 left. Find the original sum.
Explanation: The original sum is naturally more than Rs.3600.
100 100
Therefore, it should be multiplied by and
(100  10) (100  20)
 the required sum =
3600  100  100
 5, 000 .
90  80

x% of a quantity is added. Again, y% of the


increased quantity is added. Again z% of the
increased quantity is added. Now, it becomes A, then
A 100 100 100
the initial amount is given by .
(100  x)(100  y)(100  z )
Instance: A man had Rs.4800 in his locker two years ago. In the
first year, he deposited 20% of the amount in his locker. In the
second year, he deposited 25% of the increased amount in his
locked. Find the amount at present in his locker.
Explanation: The amount is certainly more than rs.4800. And each
year, the new amount is added. So, the sum should be multiplied by
100  20 100  25
and
100 100
4800  120  125
 the required amount =  Rs.7200 .
100  100

When Population Increases for One Year and Then


Decreases for the Next Year.
In the above Precept, when the population decreases
by y% during the second year, while for the first and

CREA – All Rights Reserved I U Training Services 130


Improving Your Percentages Number Math

third years, it follows the same, the population after 3 years


will be
P(100  x)(100  y )(100  z )
100 100 100
Instance: The population of a town is 10,000. It increases by 10%
during the first year. During the second year, it decreases by 20%
and increased by 30% during the third year. What is the population
after 3 years?
Explanation: The required population =
10000  110  80  130
 11440 .
100  100  100

If the price of a commodity increases by r%, then the


reduction in consumption so as not to increase the
 r 
expenditure, is  100  % .
 100  r 
Instance: If the price of sugar falls down by 10%, by how much per
cent must a householder increase its consumption, so as not to
decrease expenditure in this item?
10
Explanation:  100  9.09% .
(100  10)

Percentage Relationship
If first value is r% more than the second value, then
 r 
the second is  100  % less than the first
 100  r 
value.
Instance: If A‘s salary is 25% more than that of B, then how much
percent is B‘s salary less than that of A?
Explanation: If A‘s income is r% more than B‘s income, then

CREA – All Rights Reserved I U Training Services 131


Improving Your Percentages Number Math

 r 
B‘s income is less than A‘s income by  100  %
 100  r 
25
Thus, in this case, answer =  100%  20% .
100  25

If the first value is r% less than the second value


 r 
then, the second value is  100  % more
 100  r 
than the first value.
Instance: If C‘s salary is 30% less than that of D, then how much
percent is D‘s salary more than that of C?
Explanation: If C‘s salary is r% less than D‘s, then D‘s salary is more
 r 
than C‘s salary by  100  %
 100  r 
30 6
Thus, in this case, answer =  100  42 % .
100  30 7

First increase and then decrease


If the value of a number is first increased by x% and
later decreased by x%, the net change is always a
x2
decrease which is equal to x% of x or . If the
100
order of increase and decrease is changed, the result remains
unaffected.
Instance: If the population of a town is increased by 15% in the
first year and is decreased by 15% in the next year, what effect
can be seen in the population of that town?

15
2

Explanation: There is a decrease of % i.e., 2.25%.


100

CREA – All Rights Reserved I U Training Services 132


Improving Your Percentages Number Math

When both values are different


If the value is first increased by x% and then
 xy 
decreased by y% then there is x y %
 100 
increase or decrease, according to the +ve or –ve sign
respectively.
Instance: A shopkeeper marks the prices of his goods at 20%
higher than the original price. After that, he allows a discount of
10%. What profit or loss did he get?
20  10
Explanation: By the Precept : 20  10   8%
100
 He gets 8% profit as the sign obtained is +ve.

Effect on revenue
(i) If the price of a commodity is diminished by x%
and its consumption is increased by y%
(ii) or, if the price of a commodity is increased by
x% and its consumption is decreased by y% then the
effect on revenue =
Inc.%value  Dec.%value
Inc.% value  Dec.%value  and
100
the value is increased or decreased according to the +ve or –ve
sign obtained.
Instance: If the price is increased by 10% and the sale is
decreased by 5%, then what will be the effect on income?
Explanation: Let the price be Rs.100 per good and the sale is also of
100 goods.
So, the money obtained after selling all the 100 goods =
100 x 100 = 10,000
Now, the increased price is Rs.110 per good and the decreased sale
is 95 goods. So, the money obtained after selling all the 95 goods =
110x95=Rs.10,450.
 profit = 10,450 – 10,000 = Rs.450

CREA – All Rights Reserved I U Training Services 133


Improving Your Percentages Number Math

450  100
 % profit   4.5%
10000
By Precept: % effect
Inc. % value  Dec. % value
 Inc. % value  Dec. % value 
100
10  5
 10  5   4.5%
100
 his income increases by 4.5%.

The pass marks in an examination is x%. If a


candidate who secures y marks fails by z marks, then
100( y  z )
the maximum marks, M
x
Instance: A student has to secure 40% marks to get through. If he
gets 40 marks and fails by 40 marks, find the maximum marks set
for the examination.
Explanation: By the above Precept,
100(40  40)
Maximum marks  200 .
40

Brain Drill
1. What is 20% of 150?
1. 30 2. 20 3. 50 4. 60

2. What percentage is Rs.10 of Rs.50?


1. 10% 2. 20% 3. 30% 4. 25%

3. 12% of what weight is 240 gm?


1. 1 kg 2. 500 gm 3. 750 gm 4. 2kg

4. (30% of 70) + (70% of 30) =?

CREA – All Rights Reserved I U Training Services 134


Improving Your Percentages Number Math

1. 56 2. 28 3. 21 4. 42

5. x% of 500 = 40% of 250. Find x.


1. 25 2. 30 3. 50 4. 20

6. The sales of a company increased from 20 lakhs to 50 lakhs in one


year. By what percent did the sale increase?
1. 50% 2. 100% 3. 120% 4. 150%

7. When 75% of a number is added to 75 you get the original


number. Find the number
1. 150 2. 300 3. 180 4. 240

8. A is 5 times as large as B. What percent is B of A?


1. 20% 2. 25% 3. 30% 4.50%

9. Two numbers are respectively 15% and 25% more than a third
number. What percent is the first of the second?
1. 92% 2. 90% 3. 89% 4. 87%

10. Asked to find 15% of a number, a person finds 50% of it. If the
answer obtained is larger by 105. What was the number?
1. 150 2. 300 3. 250 4. 350

11. In a class of 200 students, 30 students are boys. What percent


of students in the class are boys?
1. 20% 2. 15% 3. 25% 4. 30%

12. In a test of 80 questions, 40% of questions are from English and


40% from Math and the remaining from General Knowledge. How
many questions are from General knowledge?
1. 32 2. 60 3. 16 4. 62

13. A survey was conducted on two drinks A and B in a college.


Every student had to choose only one soft drink. 35% students

CREA – All Rights Reserved I U Training Services 135


Improving Your Percentages Number Math

preferred A and 1,300 students preferred B. If all students


participated in the survey, find the total number of students.
1. 1,500 2. 1,650 3. 2,000 4. 2,500

14. Company A owns 60% of Company B‘s stake. Company B sells 40%
of its stake to company A. What is the new stake of Company A in
Company B?
1. 64% 2. 69% 3. 82% 4. 76%

15. After deducting 20% from a certain sum, and then deducting
30% from the remainder, the money left is Rs.1,120. Find the
original sum.
1. Rs.3,000 2. Rs.2000 3. Rs.1,000 4. Rs.4,000

16. A person gives a house for rent. He charges 4/5% of the cost
of the house as the rent per month. In how many months will he be
able to get, by way of rent, the cost of the house?
1. 100 2. 125 3. 120 4. 250

17. A person spends 30% of his salary on food, 20% on house rent
and 10% on conveyance. If he is left with Rs.16,000, find how much
more money does he spend on food than on conveyance.
1. Rs.8,000 2. Rs.9,000 3. Rs.10,000 4.Rs.6,000

18. 30% of A‘s monthly income is the same as 20% of B‘s which in
turn is equal to 40% of C‘s. If A‘s monthly income is Rs.3,000, find
C‘s monthly income.
1. Rs.2,000 2. Rs.2,250 3. Rs.3,500 4. Rs.2,500

9. A‘s salary is 20% more than B‘s and B‘s salary is 10% less than C‘s.
If C‘s salary is Rs.5,000 find A‘s salary.
1. Rs.5,000 2. Rs.4000 3. Rs.5,400 4. Rs.2,000

20. Prasad‘s salary of Rs.10,000 was increased by 20% last month.


This month his salary reduced by 20%. What will be his new salary?

CREA – All Rights Reserved I U Training Services 136


Improving Your Percentages Number Math

1. Rs.9,600 2. Rs.8,000 3. Rs.12,000 4. Rs.9,000

21. If A‘s salary is 50% more than B‘s, then by what percent is B‘s
salary less than A‘s?
2 1
1. 50% 2. 42 % 3. 56.5% 4. 33 %
3 3
22. A man spends 80% of his income. If his income and expenditure
are increased by 40% and 20% respectively. By what percent will his
savings increase?
1. 110% 2. 120% 3. 130% 4. 125%

23. A salesman gets Rs.1,000 p.m. fixed salary. In addition he gets


10% commission on his sales, if he wishes to earn a total of Rs.6,000
in a particular month, what should be his sales in the month?
1. Rs.25,000 2. Rs.4,000 3. Rs.50,000 4. Rs.60,000

24. A man‘s basic pay for a 50 hour week is Rs.250. Overtime is


paid @ 20% above the basic rate. In a certain week he worked
overtime, and his total wages were Rs.280. How many hours did he
work overtime?
1. 3 hours 2. 4 hours 3. 5 hours 4. 6 hours

25. If the price of a raw materials go up by 10%, by what percent


should consumption be reduced so as not to increase the
expenditure on raw materials?
1 1 2 1
1. 9 % 2. 11 % 3. 16 % 4. 33 %
11 9 3 3
26. The price of an article P is 30% of the combined price of Q and
R. If the total price of P, Q and R is Rs.780, find the price of P.
1. Rs.180 2. Rs.380 3. Rs.680 4. Rs.540

CREA – All Rights Reserved I U Training Services 137


Improving Your Percentages Number Math

27. The excise duty on an article, whose basic cost is Rs.1,60,000, is


decreased from 16% to 12%. What is the decrease as a percent of
the original excise duty?
1. 25% 2. 15.6% 3. 17.5% 4. 20%

28. The tax on a commodity is reduced by 20% and its consumption


increased by 20%. Find the effect on the revenue.
1. Increased by 4% 2. Increased by 8%
3. Decreased by 4% 4. Decreased by 8%

29. A manufacturer calculated that in 1992, of the total cost of


manufacturing a unit, materials constituted 50% and wages
constituted 50%. In 1993, he found that the cost of materials had
decreased by 10% while the wages increased by 10%. The
manufacturing cost in 1993 as compared to that in 1992 was
1. 10% more 2. 5% less 3. 5% more 4. The same

30. Ramesh got 33 marks in an examination, which was 10% more


than the pass mark. Find the pass mark.
1. 27 2. 30 3. 29 4. 31

31. A student has to get 40% marks to pass. He got 40 marks and
failed by 30. What is the maximum mark?
1. 200 2. 175 3. 150 4. 125

32. In an exam, Ramesh scored 50% and Suresh scored 60% marks.
Ramesh got 40 marks less than Suresh. Find the maximum mark for
the exam.
1. 200 2. 300 3. 400 4. 500

33. In an examination A got 30% of the maximum mark and failed by


50 marks. In the same examination B got 40% of the maximum
mark which was 30 marks in excess of the pass mark. Find the pass
mark.
1. 290 2. 240 3. 900 4. 1,000

CREA – All Rights Reserved I U Training Services 138


Improving Your Percentages Number Math

34. In an election between two contestants, the victor polled 52%


of the votes and won by 1,000 votes. How many votes did the loser
get?
1. 12,000 2. 13,000 3. 11,000 4. 10,000

35. In an election where there were 2 contestants, the successful


candidate got a majority of 248 votes. If 10% of the voters did not
vote, and 47% of the voters voted for him, find the total number of
eligible voters.
1. 5,500 2. 6,200 3. 6,750 4. 7,000

36. The area of a rectangle is the product of its length and breadth.
If its length is increased by 30% and its breadth is decreased by
20%, find the change in its area.
1. Increases by 10% 2. Decreases by 10%
3. Increases by 4% 4. Decreases by 4%

37. If the length of a rectangle is increased by 20%, by what


percent should its breadth be decreased in order to have the same
area?
1 2
1. 20% 2. 11 % 3. 15% 4. 16 %
9 3
38. The value of a machine depreciates at the rate of 20% per
annum. If its present value is Rs.10,000, what will be its value 2
years later?
1. Rs.8,000 2. Rs.9,000 3. Rs.6,400 4. Rs.6,500

Super Brain Drill


1. There are fifty successive percentage discounts given in a series
of 2%, 4%, 6%, 8%... and so on. What is the net discount?

CREA – All Rights Reserved I U Training Services 139


Improving Your Percentages Number Math

1. 98% 2. 2550% 3. 100% 4. Infinite

2. A manufacturer of detergent cakes increased the price of his


product as a result of which there was a decrease of 20% in its
sales. If the sales turnover increases by 20% due to this change
what was the percentage increase in the price?
1. 20% 2. 36% 3. 40% 4. 50%

3. Ravishankar wishes to buy an air conditioner with the money in


the bank, which currently is earning interest at the rate of 15% per
annum compounded annually. But Tanzar, his friend forecasts that
the inflation rate applicable to AC is going to be 14%, 15% and 16%
respectively for the next 3 consecutive years and advises
Ravishankar to postpone the purchase by 3 years. Does Ravishankar
gains monetarily, if he follows Tanzar‘s advice?
1. Yes 2. No 3. He neither gains nor loses
4. He gains only if the purchase is made in the second year

4. In a recently concluded referendum in New York, regarding


calling back US troops from Iraq, 600 people voted on the
referendum, but after some discussion the opponents increased by
150%. The motion was then rejected by a majority 2 times as great
as that by which it was formerly passed. How many people voted
against the initial referendum?
1. 400 2. 300 3. 200 4. 500

5. The number of students present in the class of Manjunath is M


and out of those M students, 22.1875% are biological idiots. If the
number of biological idiots in the class is N then what can be the
minimum value of M + N?
1. 1955 2. 391 3. 16,200 4. 1221785

6. If 75% of a class of 40 students can sing and only 20% cannot


dance, the maximum number of students who can neither sing nor
dance is

CREA – All Rights Reserved I U Training Services 140


Improving Your Percentages Number Math

1. 0 2. 6 3. 18 4. Can‘t be determined

7. In a laboratory experiment, a sample of air, which is a mixture of


oxygen and water vapour is taken. Water vapour contains hydrogen
and oxygen gases. If air contains a total of 70% oxygen (including
that contained in the water vapour) by weight while water vapour
contains 16.66% of oxygen by weight, how many kilograms of water
vapour is present in 1 kilogram of air?
1. 0.2 2. 0.36 3. 0.32 4. 0.25

CREA – All Rights Reserved I U Training Services 141


2
Gain and Loss

CREA – All Rights Reserved I U Training Services 142


PRE-LESSON Challenge – See What You Know
and Remember
Time Limit – 5 minutes

1. If mangoes are bought at 15 a rupee, how many


must be sold for a rupee to gain 25%?
2. A watch passes through three hands and each
gains 25%. If the third sells it for Rs. 250, what did
the first pay for it?
3. A man sells an article at 5% profit. If he had
bought it at 5% less and sold it for Re. 1 less, he
would have gained 10%. Find the cost price?
4. The marked price of a radio is Rs.480. The
shopkeeper allows a discount of 10% and gains 8%. If
no discount is allowed, find his gain percent.
5. What will be the percentage profit after selling an
article at a certain price if there is a loss of 45%
when the article is sold at half of the previous selling
price?

CREA – All Rights Reserved I U Training Services 143


Gain and Loss Number Math

In This Chapter
 Balancing selling price and cost for a profit
 Discovering costs and how to predict them
 Breaking even in business

The basic idea of making a profit on a venture comes early to young


people. Maybe you had a lemonade stand out in front of your house,
selling cups of that sweet stuff to the passing neighbors? Or maybe
you were involved in organizations or camps that had you fundraising
by selling items such as cookies, popcorn, or candy. And if you
weren‘t lucky enough to be involved in any of those activities, there
were always the school supply sales run by the student council or
the cricket game concession stands run by the glee club. The point
is that most people have had an opportunity to see profit, revenue,
and cost at work in a firsthand situation before they even set foot
into the business world.
The general concept of profit is fairly straightforward and
relatively easy to understand. A profit is simply the difference
between the amount of money you receive for goods and the amount
it costs you to produce them. What makes the process of
determining profit all the more interesting is when you begin to
break down the costs into the different types and then investigate
the different ways that the costs are handled. This investigation is
a far cry from the lemonade stand where you added up how much
the lemons, sugar, and cups cost.
Coming out ahead is a definite (and pretty obvious) goal for the
businessperson. And, before you get ahead, you have to break even.
In other words, you want to earn at least what you‘re spending. You
want to reach the breakeven level of sales and exceed it by as much
as possible. Determining the break-even point involves getting a
handle on the revenue and the costs — and doing the appropriate
computations. In this chapter, you discover the basis of profit,
revenue, cost, and breaking even — and how they all work together
for you.

CREA – All Rights Reserved I U Training Services 144


Gain and Loss Number Math

The Cost Price (CP) of an article is the price at which an article is


purchased. In the case of a manufacturer CP is the cost of
production. In the case of a retailer CP of an article is the cost at
which it is bought from the wholesaler.
Selling Price (SP) is the price at which the article is sold.
Profit / Gain = Selling Price (SP) – Cost Price (CP).
Loss = Cost price (CP) – Selling Price (SP).
Profit / Loss is usually expressed as a percent of the Cost Price.

If P% is the profit, then If L% is the loss, then


SP  CP CP  SP
P  100 L  100
CP CP
CP(100  P) CP(100  L)
SP  SP 
100 100
100  SP 100  SP
CP  CP 
(100  P) (100  L)

The price at which the goods are listed is called the Marked Price
(MP) or List Price or Catalogue Price.
Discount = Marked Price – Selling Price.
Discount is expressed as a percentage of the Marked Price (MP).
MP(100  D)
SP  ; where D is the discount percent
100
Now let‘s solve some problems based on the above precepts.

Instance 1: A man purchases 8 pens for Rs.9 and sells 9 pens for
rupees 8. How much profit or loss does he make?
Explanation: Purchases 8 pens for Rs.9
Sells 9 pens for Rs.8
% profit or loss =
88  99 1700
 100   20.98%
99 81

CREA – All Rights Reserved I U Training Services 145


Gain and Loss Number Math

Since the sign is –ve, there is a loss of 20.98%.

Instance 2: Ram, a dishonest seller, sells his goods at cost price.


But he uses a weight of 960 gm for one kilo weight. Find his gain
percent.
Explanation: Suppose goods cost the dealer Re 1 per kg. He sells
for Re 1 what cost him Re 0.96.
 Gain on Re. 0.96 = Re 1 – Re0.96 = Re 0.04
0.04 1
 Gain on Rs 100 =  100  Rs 4
0.96 6
1
 Gain % = 4 %
6
Error
% gain   100
True value  Error
True weight  False weight
or , % gain   100
False weight
40 1
  100  4 %.
1000  40 6

Instance 3: A sells a good to B at a profit of 20% and B sells it to C


at a profit of 25%. If C pays Rs.225 for it, what was the cost price
for A?
Explanation: During both the transactions there are profits. So our
calculating figures would be 120, 125 and 100. A‘s cost price is
certainly less than C‘s selling price.
100 100
 Required price = 225    Rs 150 .
120 125
Remark: Since we need a value which is less than the give value, so
our multiplying fractions should be less than one. That is why we
100 100
multiplied 225 with and .
120 125

CREA – All Rights Reserved I U Training Services 146


Gain and Loss Number Math

Instance 4: A reduction of 10% in the price of sugar enables a


person to obtain 25 kg more for Rs.225. What is the reduction
price per kg? Find also the original price per kg.
Explanation: Owing to the fall in price, there is a saving of 10% on
45
Rs.225, i.e., Rs.
2
45
For this Rs. , a person purchases 25 kg of sugar. Hence
2
reduced price per kg = Rs.22.5  25 = Re 0.90 = 90 P
100
Original price = 90 P   Re 1 .
100  10

Instance 5: The marked price of a radio is Rs.480. The shopkeeper


allows a discount of 10% and gains 8%. If no discount is allowed,
find his gain percent.
 100  10 
Explanation: Selling price = 480    Rs. 432
 100 
 100 
Cost price = 432    Rs. 400
 100  8 
If there is no discount, SP = Rs.480
480  400
 % profit =  100  20% .
400

Instance 6: What will be the percentage profit after selling an


article at a certain price if there is a loss of 45% when the article
is sold at half of the previous selling price?
Explanation: % profit = 100 – 2 x % loss = 100 – 2 x 45 = 10%
Note: The more general formula for the Instance 6 may be alike:
1
When the second selling price is of the original selling
x
price. Then % profit = x (100 - % loss) – 100

CREA – All Rights Reserved I U Training Services 147


Gain and Loss Number Math

This formula is the same as used in Instance 6. In these


two cases
% profit = 2(100 - % loss) – 100
= 200 – 2 x % loss – 100 = [100 – 2 x % loss]

Brain Drill
1. A person bought an article at Rs.100 and sold it at Rs.150. Find
his gain.
1. 20% 2. 30% 3. 25% 4. 50%

2. A man bought a chair for Rs.500 and sold it at 20% gain. Find its
selling price.
1. Rs.600 2. Rs.550 3. Rs.700 4. Rs.650

3. The cost price of an article is 80% of its selling price. The selling
price is what percent of cost price?
1. 100% 2. 120% 3. 80% 4. 125%

4. The cost price of 21 articles is equal to the selling price of 14


articles. Find the profit.
1. 25% 2. 30% 3. 40% 4. 50%

5. By selling 6 dozen bananas, a shopkeeper loses the selling price of


1 dozen bananas. Find his loss.
2 1
1. 15% 2. 16% 3. 14 % 4. 16 %
7 3

6. A dishonest dealer professes to sell his goods at cost price, but


uses a weight of 950 gm for a kg weight. Find his gain.
1 3 5
1. 5% 2. 5 % 3. 5 % 4. 5 %
9 19 19

CREA – All Rights Reserved I U Training Services 148


Gain and Loss Number Math

7. A man sold 40 pens for Rs.90 and gained 25%. How many pens
could he buy for Rs.90?
1. 30 2. 50 3. 40 4. 45

8. A man purchased an article for Rs.360 and sold it at a gain of


25% of the selling price. Find the selling price of the article.
1. Rs.400 2. Rs.440 3. Rs.480 4. Rs.470

9. Chandu bought a scooter for Rs.20,000. He spent 20% of his


cost price towards repairs. If he wishes to sell the scooter at
Rs.30,000 what is his gain on his overall investment?
1. 10% 2. 15% 3. 25% 4. 20%

10. The percent profit obtained by selling an article at Rs.700 is


same as the percent loss incurred by selling it for Rs.600. Find the
cost price of the article.
1. Rs.675 2. Rs.650 3. Rs.625 4. Rs.620

11. By selling an article for Rs.200, a man loses 20%. What should
be the selling price of the article, if he wants to gain 20%?
1. Rs.250 2. Rs.220 3. Rs.260 4. Rs.300

12. What will be the profit on selling an article at a certain price, if


selling at one-third of that price means 55% loss?
1. 35% 2. 45% 3. 48% 4. None of these

13. A man sold one-fourth of goods at a loss of 25%. At what gain


should he sell the rest so as to gain a profit of 20% on the whole?
1. 42.5% 2. 32.5% 3. 45% 4. 35%

14. Ram sells an article to Bharat at a profit of 25%. Bharat sells


the same to Laxman for Rs.2,100 thereby making a profit of 5%.
What did Ram pay for it?
1. Rs.1,500 2. Rs.1,600 3. Rs.1,650 4. Rs.1,550

CREA – All Rights Reserved I U Training Services 149


Gain and Loss Number Math

15. The difference between selling an article at a gain of 10% and at


a loss of 10% is Rs.136. Find the cost price of the article.
1. Rs.544 2. Rs.600 3. Rs.680 4. Rs.540

16. A trader has certain number of caps. If he sells each cap at


Rs.50, he will have a profit of Rs.500. If he sells each cap at Rs.25,
he will have a loss of Rs.125. Find the number of caps he has.
1. 35 2. 25 3. 30 4. 28

17. A trader sells one-third of the articles at 10% profit, one-


fourth of the articles at 15% profit and the rest at 20% profit. If
his total profit is Rs.185, find the total cost price of the articles
1. Rs.1,200 2. Rs.1,850 3. Rs.2,220 4. Rs.2,000

18. A man sold two houses for Rs.1,00,000 each. On one he gains
20% and on the other he loses 20%. How much does he gain or lose
in the whole transaction?
1. gain 2% 2. gain 4% 3. loss 2% 4. loss 4%

19. A person purchased a chair and a table for Rs.1800. He sold the
chair at a profit of 25% and the table at a loss of 11%. If his total
part was 5%, find the cost price of a table.
1. Rs.800 2. Rs.1000 3. Rs.890 4. Rs.600

20. If a dealer buys two cars and sells them at Rs.6,000 each,
making a profit of 20% of the cost on one and a loss of 20% of the
cost on the other, what is the net result of the transaction?
1. He mad e a gain of Rs.500 2. He made a loss of Rs.500
3. He made a gain of Rs.1,000 4. He made a loss of Rs.1000

21. A man bought two watches for Rs.2,000. He sold one of them at
a loss of 20% and the other at a gain of 20%. On the whole he
neither gained nor lost. Find the cost price of each watch.
1. Rs.1,200 , Rs.800 2. Rs.1,000 , Rs.1,000
2. Rs.1,500 , Rs.500 4. Rs.700 , Rs.1,300

CREA – All Rights Reserved I U Training Services 150


Gain and Loss Number Math

22. A trader sold a rabbit and a peacock for Rs.4,480. He got a


profit of 10% on rabbit and 20% on peacock. If he wants a profit of
20% on rabbit and 10% on peacock, he has to sell them for Rs.4,260.
Find the cost price of a rabbit.
1. Rs.700 2. Rs.800 3. Rs.3,000 4. Rs.2,500

23. A person purchased a television and a radio for Rs.5,550. He


sold the television at a profit of 10% and the radio at a profit of
20%. His total profit was 14%. Find the cost price of television.
1. Rs.2,220 2. Rs.3,330 3. Rs.4,000 4. Rs.5,000

24. An article whose marked price is Rs.150 is sold for Rs.120. Find
the discount
1. 25% 2. 30% 3. 20% 4. 15%

25. A customer saves Rs.45 when he avails a discount of 25%. How


much did he pay?
1. Rs.180 2. Rs.135 3. Rs.15 4. None of these

26. If a man marks his goods 40% higher than his cost price, and
allows a discount of 20%, find his net gain.
1. 12% 2. 20% 3. 30% 4. 28%

27. A trader allows a discount of 15% on the marked price of an


item and still gains 10%. What is the cost price of an article which is
marked price Rs.220?
1. Rs.150 2. Rs.160 3. Rs.170 4. Rs.180

28. A trader gives a discount of 20% on the list price and still
makes a profit of 20%. By what percent the cost price was marked
up?
1. 40% 2. 50% 3. 45% 4. 35%

CREA – All Rights Reserved I U Training Services 151


Gain and Loss Number Math

29. The cost price of an article is Rs.125. It is marked up to yield a


profit of 40%. A discount is given on this, but still 20% profit is
made. What is the selling price of the article?
1. Rs.225 2. Rs.150 3. Rs.175 4. Rs.200

30. Maddy bought a bike at 10% discount and sold it for Rs.39,600
at 10% profit. What was the original price of the bike?
1. Rs.36,000 2. Rs.38,000 3. Rs.34,000 4. Rs.40,000

31. In a clearance sale, two successive discounts are given. A person


bought an article for Rs.336. If the marked price is Rs.600, and
first discount is 20%, find the second discount.
1. 35% 2. 30% 3. 45% 4. 32%

32. Find a single discount equivalent to three successive discounts


of 20%, 25% and 10%.
1. 45% 2. 54% 3. 46% 4. 52%

Super Brain Drill


1. Sunny marks up his goods by 40% and gives a discount of 10%.
Apart from this, he uses faulty balance which reads 800 gms for
1000 gms. What is his net profit/loss percentage?
1. 8% 2. 57.2% 3. 37.6% 4. None of these

2. A publisher printed 3000 copies of Fountainhead at a cost of Rs.


2400. He gave 500 copies free to different philanthropic
institutions. He allowed a discount of 25% on the published price
and gave one copy free for every 25 copies bought at a time. He was
able to sell all the copies in this manner. If the published price is Rs.
3.25, then what is his overall gain or loss percentage?
1. 11.3% 2. 130% 3. 162% 4. 144%

3. A shopkeeper purchases a packet of 50 pens at Rs. 10 per pen. He

CREA – All Rights Reserved I U Training Services 152


Gain and Loss Number Math

sells a part of the packet at a profit of 30%. On the remaining part,


he incurs a loss of 10%. If his overall profit on the whole packet is
10%, the number of pens he sold at profit are
1. 25 2. 30 3. 20 4. 15

4. Amar sold his scooter to Bharat at 20% profit and Bharat sold it
to Sridhar at 10% profit. Sridhar sold it to a mechanic and received
Rs. 23160. If Amar had sold the same moped to the mechanic and
receive the same amount the mechanic paid to Sridhar, what profit
percentage would Amar have made?
1. 52.8% 2. 58.4% 3. 33.3% 4. Can‘t be found

5. By giving a discount of Rs. 200 a trader gains 20%. Had he given a


discount of 45%, he would have lost Rs. 60. What is his marked
price?
1. Rs. 600 2. Rs. 800 3. Rs. 400 4. Can‘t be found

6. With the money I have, I can buy 50 pens or 150 pencils. I kept
10% of it aside for taxi fare. With the remaining, I purchased 54
pencils and P pens. What is the value of P?
1. 32 2. 30 3. 27 4. None of these

7. There are some articles with Aman. The selling price of 12


articles and the cost price of 10 articles put together is equal to
the cost price of 14 articles and the selling price of 8 articles put
together. What is the overall percentage of profit or loss if Aman
sells 12 articles?
1. 20% loss 2. 20% gain 3. 10% gain 4. No loss, no
gain

8. Which of the following series of successive discounts will be most


favourable to the customer?
1. 8%, 10%, 12% 2. 10%, 10%, 10%
3. 10%, 15%, 5% 4. 1%, 1%, 1%, 27%

CREA – All Rights Reserved I U Training Services 153


Gain and Loss Number Math

9. A trader marks an article 4x% above the cost price. He gives a


disount of (x + 5)% and gets a profit of (2x - 5)%. If he gives a
discount of 5x/3%, what would be his profit percent?
1. 16 2/3% 2. 20% 3. 25% 4. 33 1/3%

10. A trader marks a television set p% over his cost price and gives
a discount of p/3% to get a profit of p/3%. What is his percentage
of profit?
1. 33 1/3% 2. 25% 3. 50% 4. None of these

CREA – All Rights Reserved I U Training Services 154


3
Averages and QnQ of
Mixtures

CREA – All Rights Reserved I U Training Services 155


PRE-LESSON Challenge – See What You
Know and Remember
Time Limit – 5 minutes

1. A train travels from A to B at the rate of 20 km


per hour and from B to A at the rate of 30 km/hr.
What is the average rate for the whole journey?
2. A batsman scored 135 runs in his 20th innings and
increased his average by 5 runs. What is his
average after 20 innings?
3. In what proportion must rice at Rs.14 per kg be
mixed with rice at Rs.10 per kg, so that the
mixture be worth Rs.11 per kg.?
4. How much wheat at Rs.3 a kg must be added to 5
kg of wheat at Rs.6 a kg so that the mixture be
worth Rs.4 a kg?
5. A jar full of whisky contains 50% alcohol. A part
of this whisky is replaced by another containing
30% alcohol. Now the alcohol was found to be 36%.
What fraction of whisky was replaced?

CREA – All Rights Reserved I U Training Services 156


Averages and QnQ of Mixtures Number Math

In This Chapter
 Working with averages and applying to real life situations
 Concentrating on the concentrations of mixtures
 Using quality times quantity as the standard

When you read about the average income across the


country, you probably think in terms of a number that falls in the
middle of the entire spectrum of incomes. In general, that‘s a
correct thought — the average usually is the middle or most used
value. But, in fact, the average value can be determined one of three
ways. In other words, the ―middle‖ average is just one way. The
average can be the mean, the median, or the mode. We will be
looking only at mean.
An average, or more accurately an arithmetic mean is, in
crude terms, the sum of n different data divided by n.
For Instance, if a batsman scores 35, 45 and 55 runs in
first, second and third innings respectively, then his average runs in
35  45  55
3 innings is equal to  45 runs.
3
Therefore, the two mostly used formula in this chapter are:
Total of data
Average 
No. of data
And, Total = Average x No. of data

Average related by speed


If a person travels a distance at a speed of x km/hr
and the same distance at a speed of y km/hr, then
the average speed during the whole journey is given by
2xy
km/hr.
x y

If a person travels three equal distances at a speed


of x km/hr, y km/hr and z km/hr respectively, then

CREA – All Rights Reserved I U Training Services 157


Averages and QnQ of Mixtures Number Math

3xyz
the average speed during the whole journey is
xy  yz  xz
km/hr.

Instance 1: A train travels from A to B at the rate of 20 km per


hour and from B to A at the rate of 30 km/hr. What is the average
rate for the whole journey?
Explanation: By the formula, Average speed =
2  20  30
 24 km / hr.
20  30

Instance 2: A person divides his total route of journey into three


equal parts and decides to travel the three parts with speeds of 40,
30 and 15 km/hr respectively. Find his average speed during the
whole journey.
Explanation: By the Precept: Average speed =
3  40  30  15
40  30  30  15  40  15
3  40  30  15
=  24 km / hr.
2250

Instance 3: The average salary of the entire staff in a office is


Rs.120 per month. The average salary of officers is Rs.460 and that
of non-officers is Rs.110. If the number of officers is 15, then find
the number of non-officers in the office.
Explanation: Let the required number of non-officers = x
Then, 110x + 460 x 15 = 120 ( 15 + x)
Or, 120x – 110x = 460 x 15 – 120 x 15 = 15 (460 – 120)
Or, 10x = 15 x 340;  x = 15 x 34 = 510.

Alligation and Mixtures

CREA – All Rights Reserved I U Training Services 158


Averages and QnQ of Mixtures Number Math

A classic type of mathematical word problem is the mixture


problem. But mixture problems go beyond just mixing up antifreeze
with water or chocolate syrup with milk. Mixture problems involve all
sorts of situations where you combine so much of one thing that has
a certain amount of worth or density with so much of something else
that has more worth or more density.
In this chapter, you find problems that multiply numbers of
coins times their monetary value, pints of coolant with their
concentrations and cartons of goodies with their product count. All
these make for some interesting and, yes, even useful problems. So
you can put your spoon away and find out what mixture problems are
all about.
The common theme in all mixture problems is that you take
two or more different amounts (quantities) of two different
concentrations (or qualities) and mix them together to get an
amount (quantity) that‘s the sum of the two ingredient quantities
and a concentration (quality) that‘s somewhere between the
concentration of the two ingredients that you started with. Figure
below illustrates this property or theme with containers.

Alligation is the rule that enables us


1. to find the mean or average value of mixture when the prices of
two or more ingredients which may be mixed together and the

CREA – All Rights Reserved I U Training Services 159


Averages and QnQ of Mixtures Number Math

proportion in which they are mixed are given (this is Allegation


Medial); and
2. to find the proportion in which the ingredients at given prices
must be mixed to produce a mixture at a given price. This is
Alligation Alternate.
Note: (1) The word Alligation literally means linking. The rule takes
its name from the lines or links used in working out questions on
mixture.
(2) Alligation method is applied for percentage value, ratio, rate,
prices, speed etc and not for absolute values. That is, whenever
percent, per hour, per kg, per km etc are being compared, we can
use Alligation.

Rule of Alligation
If the gradients are mixed in a ratio, then

Quantity of cheaper CP of dearer  Mean price



Quantity of dearer Mean price  CP of cheaper

We represent it as under:

CP of unit quantity CP of unit quantity


cheaper (c) of dearer of (d)

Mean price
(m)

(d – m) (m – c)

Then, (cheaper quantity): (dearer quantity) = (d – m): (m – c).

CREA – All Rights Reserved I U Training Services 160


Averages and QnQ of Mixtures Number Math

Solved Instances
Instance1: In what proportion must rice at Rs.3.10 per kg be mixed
with rice at Rs.3.60 per kg, so that the mixture be worth Rs.3.25 a
kg?
Explanation:
C. P. of 1kg. cheaper rice C.P. of 1 kg. dearer rice
(310 paise) (360 paise)

Mean price
(325 paise)

35 15
(Quantity of cheaper rice) 35 7
By the alligation rule:   .
(Quantity of dearer rice) 15 3
 They must be mixed in the ratio 7: 3.

Instance 2: A mixture of a certain quantity of milk with 16 litres of


water is worth 90 P per litre. If pure milk be worth Rs.1.08 per
litre how much milk is there in the mixture?
Explanation: The mean value is 90 P and the price of water is 0 P.
milk water
108 0

90

90 – 0 108 – 90
By the Alligation Rule, milk and water are in the ratio of 5 : 1.
 quantity of milk in the mixture = 5 x 16 = 80 litres.

Instance 3: A butler stole wine from a butt of sherry which


contained 40% of spirit. He replaced what he had stolen by wine

CREA – All Rights Reserved I U Training Services 161


Averages and QnQ of Mixtures Number Math

containing only 16% spirit. The butt was then of 24% strength only.
How much of the butt did he steal?
Explanation:
Wine containing Wine containing
40% Spirit 16% Spirit

Wine containing
24% Spirit

8 16
wine with 40% spirit 8 1
 By alligation rule:  
wine with 16% spirit 16 2
1
i.e., they must be mixed in the ratio (1:2). Thus of the butt of
3
2
sherry was left and hence the butler drew out of the butt.
3

Instance 4: Milk and water are mixed in a vessel A in the proportion


5 : 2, and in vessel B in the proportion 8 : 5. In what proportion
should quantities be taken from the two vessels so as to form a
mixture in which milk and water will be in the proportion of 9 : 4?
5
Explanation: In vessel A, milk = of the weight of mixture
7
8
In vessel B, milk = of the weight of mixture. Now we want to
13
9
form a mixture in which milk will be of the weight of this
13
mixture. By alligation rule:

CREA – All Rights Reserved I U Training Services 162


Averages and QnQ of Mixtures Number Math

5 8
7 13

9
13

1 2
13 91

1 2
 required proportion is : = 7 : 2.
13 91

Instance 5: 300 gm of sugar Explanation has 40% sugar in it. How


much sugar should be added to make it 50% in the Explanation?
Explanation: The existing Explanation has 40% sugar. And sugar is
to be mixed; so the other Explanation has 100% sugar. So by
alligation method:
40% 100%

50%

50% 10%

CREA – All Rights Reserved I U Training Services 163


Averages and QnQ of Mixtures Number Math

 The two mixtures should be added in the ratio 5 : 1.


300
Therefore, required sugar =  1  60 gm .
5

Brain Drill
1. Find the average of 10, 20 and 30.
1. 10 2. 20 3. 30 4. 40

2. The average of 3 numbers is 15. Find the sum of the numbers.


1. 10 2. 25 3. 45 4. 55

3. The average of first 5 multiples of a number is 12. Find the


number.
1. 5 2. 4 3. 3 4. 2

4. The average of 3 numbers is 15. If each number is multiplied by


2, find the average of new set of numbers.
1. 30 2. 25 3. 15 4. 20

5. The average of three numbers is 90. The first number is twice


the third and is 60 less than twice the second. Find the difference
between the largest and the smallest numbers.
1. 60 2. 30 3. 90 4. 120

6. The average score of a batsman in 5 matches is 80 and that of


the first four is 70. Find his score in the fifth match
1. 100 2. 150 3. 130 4. 120

7. A batsman scored 135 runs in his 20th innings and increased his
average by 5 runs. What is his average after 20 innings?
1. 35 2. 30 3. 45 4. 40

CREA – All Rights Reserved I U Training Services 164


Averages and QnQ of Mixtures Number Math

8. The average age of a class of 20 students is 20 years. If


teacher‘s age be included, the average age increases by 1 year. Find
the age of teacher.
1. 40 years 2. 45 years 3. 42 years 4. 41 years

9. The average age of 8 students in a class is 20 years and the


average age of the other 12 students is 15 years. Find the average
age of the class?
1. 17 years 2. 16 years 3. 18 years 4. 19 years

10. The average age of a couple, married 20 years ago, was 24 years
at the time of their marriage. Find the present average of the
family if they have a 14 year old son.
1. 26 years 2. 30 years 3. 34 years 4. 44 years

11. The average age of A, B, C, D four years ago was 51 years. If E


is included, their present average age will be 52 years. Find the
present age of E.
1. 35 years 2. 40 years 3. 45 years 4. 50 years

12. A‘s income is half of B‘s and is thrice C‘s. If C‘s income is
Rs.1,500, find the average income of A, B and C.
1. Rs.4,500 2. Rs.5,500 3. Rs.5,750 4. Rs.5,000

13. The average daily earnings of a person from Monday to


Wednesday is Rs.50 and from Tuesday to Thursday Rs.60. The
earnings on Monday were half of that on Thursday. How much did
he earn on Monday and Thursday together?
1. Rs.90 2. Rs.60 3. Rs.50 4. Rs.40

14. In a certain year the average monthly expenditure of Ramesh


was Rs.2,000 during the first 4 months, Rs.3,000 during the next 4
months, and Rs.2,000 during the last 4 months. If the total savings

CREA – All Rights Reserved I U Training Services 165


Averages and QnQ of Mixtures Number Math

during the year were Rs.8,000, what was his average monthly
income?
1. Rs.4,000 2. Rs.3,000 3. Rs.3,500 4. Rs.2,500

15. The average weekly salary per head in a firm consisting of


executives and non-executives is Rs.80. The average salary per
head of the executives is Rs.500 and that of non-executives Rs.60.
If the number of executives is 15, find the number of non-
executives in the firm.
1. 275 2. 315 3. 300 4. 200

16. Three people contributed Rs.40 each towards charity. A fourth


person contributed Rs.30 more than the average contribution of all
four. Find the total contribution of the four.
1. Rs.150 2. Rs.180 3. Rs.160 4. Rs.200

17. The average weight of A, B and C is 60 kg. If their weights are


in the ratio 1:2:3, find the weight of C.
1. 30 kg 2. 60 kg 3. 90 kg 4. 100 kg

18. The average weight of 10 persons is increased by 2 kg when one


of them weighing 50 kg is replaced by a new person. Find the weight
of the new person.
1. 60 kg 2. 70kg 3. 40 kg 4. 30 kg

19. The average weight of three men A, B and C is 70kg. Another


man D joins the group and the average now becomes 75kg. If
another man E, whose weight is 10 kg less than that of D, replaces
A, then the average weight of B, C, D and E becomes 80kg. Find A‘s
weight.
1. 60kg 2. 70 kg 3. 50 kg 4. 75 kg

20. The average marks of a class of 20 students is 40. It was found


later that the marks of two students were entered as 48 and 40
instead of 8 and 12. Find the correct average.

CREA – All Rights Reserved I U Training Services 166


Averages and QnQ of Mixtures Number Math

1. 18.3 2. 40.66 3. 36.6 4. None of these

21. The average marks of a class in a certain subject is 40. Of


them 10 students got on an average 20 marks. If these 10 were not
sent for the exam the class average would have gone up by 10
marks. Find the number of students in the class.
1. 30 2. 40 3. 25 4. 35

22. If 10 pens are bought at Rs.20 each, and 15 pens are bought at
Rs.10 each, find the average cost for pen.
1. Rs.15 2. Rs.14 3. Rs.13 4. Rs.12

23. Mohan has taken 7 tests and his average is 60%. What would he
have to average on the next 3 tests to raise his overall average to
70%?
1 2 3 1
1. 92 % 2. 90 % 3. 95 % 4. 93 %
2 3 4 3

24. In what proportion must rice at Rs.14 per kg be mixed with rice
at Rs.10 per kg, so that the mixture be worth Rs.11 per kg.?
1. 1:2 2. 1:3 3. 3:1 4. 2:1

25. How much wheat at Rs.3 a kg must be added to 5 kg of wheat at


Rs.6 a kg so that the mixture be worth Rs.4 a kg?
1. 5kg 2. 10kg 3. 8kg 4. 9kg

26. 5 litres of water is added to 20 litres of an Explanation of 25%


alcohol. Find the new strength of alcohol.
2 1
1. 20% 2. 25% 3. 16 % 4. 11 %
3 9

27. Pure milk is sold at Rs.12 per litre. A milk vendor mixes water in
the milk and sells the mixture at Rs.9 per litre. In what ratio did he
mix milk and water?
1. 3:1 2. 3:4 3. 2:5 4. None of these

CREA – All Rights Reserved I U Training Services 167


Averages and QnQ of Mixtures Number Math

28. A jar full of whisky contains 50% alcohol. A part of this whisky
is replaced by another containing 30% alcohol. Now the alcohol was
found to be 36%. What fraction of whisky was replaced?
3 2 3 7
1. 2. 3. 4.
7 7 10 10

29. A container contains 50 litres of alcohol. From this container,


10 litres of alcohol was taken out and replaced by water. This
process was repeated once more. How much alcohol is now left in
the container?
1. 30 litres 2. 32 litres 3. 40 litres 4. 41 litres

Super Brain Drill


1. The average marks of the girls in a class is equal to the number of
boys and the average marks of the boys is equal to the number of
girls. If the class average is 4 less than the average of both the
boys‘ and the girls‘ average marks, what will be th number of
students in the class?
1. 24 2. 48 3. 50 4. 64

2. A, B, C, D and E are five consecutive integers and the average of


these five numbers is less than 1/4th of A. Then A is
1. Positive 2. Negative 3. Even 4. Data Insufficient
3. Find the difference between the average of the first n even
numbers and the average of all the even numbers up to n (n is an
even number)
1. 2. 3. n/2 4. None of these

4. I was born 30 years after my father was born. My sister was


born 25 years after my mother was born. The average age of my
family is 26.25 years right now. My sister will get married 4 years

CREA – All Rights Reserved I U Training Services 168


Averages and QnQ of Mixtures Number Math

from now and will leave the family. Then the average age of the
family will be 107/3 years. What is the age of my father?
1. 30 years 2. 35 years 4. 40 tears 4. 45 years

5. A shipping clerk has to weigh 6 distinct packets. He weighs them


four at a time, weighing all the possible combinations of the packets
from the six. The average weight of all the weighing combinations is
found to be 500 gm. What is the combined weight of all the six
packets?
1. 375 gm 2. 1250 gm 3. 750 gm 4. 1000/3 gm

6. The average of 5 positive numbers is A. The average of the


average of all the possible distinct triplets is B. Which of the
following is true?
1. A>B 2. A<B 3. A = B 4. Can‘t be determined

7. 40% of the students in a class are boys and 75% of the boys
score more than 80% in an exam. If 45% of the class scores more
than 80% in the exam, what fraction of the girls in the class score
less than or equal to 80%?
1. 2/3 2. 3/4 3. 1/4 4. 5/6

8. Vasavi, the rice trader mixes 2 varieties of rice. The cost price
of the first variety of rice is twice the cost price of the second
variety. If Vasavi sells the mixture at the cost price of the costlier
variety and makes a profit of 35%, then what is the ratio in which
the varieties of rice are mixed?
1. 7:13 2. 13:14 3. 1:3 4. Can‘t be found

9. Balu travels a distance of 360 km in 9 hours, part of it in his car


at 70 kmph and the remaining on his scooter at 36 kmph. What is
the ratio of the distances travelled by Balu on scooter and in car?
1. 2:15 2. 5:1 3. 1:5 4. 27:7

CREA – All Rights Reserved I U Training Services 169


4
Permutations and
Combinations

CREA – All Rights Reserved I U Training Services 170


PRE-LESSON Challenge – See What You Know
and Remember
Time Limit – 5 minutes

1. How many different numbers can be formed by


using six out of nine digits 1,2,3,… 9?
2. In how many ways can four persons be seated if
there are six seats available?
3. In how many ways can a party of seven gentlemen
and 6 ladies be seated at a round table so that no two
ladies are seated together?
4. From 12 books in how many ways can a selection of
5 books be made (a) when one specified book is always
included and (b) when one specified book is always to
be excluded?
5. How many different choices can be made of
selections out of 15 Physics, 10 English and 12
Mathematics books when at least one book is to be
selected?

CREA – All Rights Reserved I U Training Services 171


Permutations & Combinations Number Math

In This Chapter
 Using the basic principle of counting
 Working with circular and restricted permutations
 Working with combinations

PERMUTATIONS
Basic Addition Principle
If an operation can be performed in ‗m‘ ways and if another
operation can be performed in ‗n‘ ways and both the operations are
mutually exclusive (i.e. only one operation can be performed at a
time - either the first or the second) then either of the two
operations can be performed in m + n ways.
Instance: There are four books on Physics and five on Mathematics
on display in a certain showroom. In how many ways can Ram buy a
book either of Physics or of Mathematics?
Explanation: There are 5 ways of buying a book on Mathematics and
there are 4 ways of buying a book on Physics. Thus Ram can buy
either a Physics book or a Mathematics book in 5 + 4 = 9 ways.

Basic Multiplication Principle


If one operation can be performed in ‗m‘ ways, and (when it has been
performed in any one of these ways) a second operation can then be
performed in ‗n‘ ways independent of the first; the number of ways
of performing the two operations will be m x n.
If the first operation be performed in any one way, we can
associate with this one way, any of the ‗n‘ ways of performing the
second operation. Thus we shall have ‗n‘ ways of performing the two
operations without considering more than one way of performing the
first. Similarly corresponding to each of the ‗m‘ ways of performing
the first operation, we shall have ‗n‘ ways of performing the second;
hence altogether the number of ways in which the two operations
can be performed is represented by the product m x n.

CREA – All Rights Reserved I U Training Services 172


Permutations & Combinations Number Math

Instance: There are 15 trains for going from Delhi to Bombay and
15 for going back from Bombay to Delhi. In how many ways can a
man go from Delhi to Bombay and return by a different train?
Explanation: There are 15 ways of going from Delhi to Bombay; and
with each of these there is a choice of 14 trains (a different train
is required) for returning. Hence the number of ways of making the
two journeys is 15 x 14, or 210. (Note that if he could have come
back in the same train, then the number of ways would be 15 x 15, or
215).
Instance: Three travellers arrive at a town where there are four
hotels; in how many ways can they take up their quarters, each at a
different hotel?
Explanation: The first traveller has a choice of four hotels, and
when he has made his selection in any one way, the second traveller
has a choice of three. Therefore the first two can make their
choice in 4 x 3 ways; and with any one such choice the third
traveller can select his hotel in 2 ways. Hence the required number
of ways is 4 x 3 x 2, or 24.
Each of the arrangements which can be made by taking some or all
of a number of things is called a permutation. In forming
permutations, we have to consider the order of the distinct things
which make up each arrangement; for instance, the permutations
which can be made by taking the letters a, b, c, d two at a time are
twelve in number, namely ab, ac, ad, ba, bc, bd, ca, cb, cd, da, db, dc.

Number of Permutations of N Dissimilar Things Taken R


at a Time
This is the same thing as finding the number of ways in which we can
fill up ‗r‘ places when we have ‗n‘ different things at our disposal.
The first- place may be filled up in n ways, for any one of the n
things may be taken. When the first place has been filled up in any
one of these ways, the second place can then be filled up in n – 1
ways (since only n – 1 things are now left). Also since each way of
filling up the first place can be associated with each way of filling
up the second, the number of ways in which the first two places can

CREA – All Rights Reserved I U Training Services 173


Permutations & Combinations Number Math

be filled up is given by the product n(n- 1). Now, when the first two
places have been filled up in anyway, n -2 things are left, and the
third place can be filled up in n - 2 ways. Reasoning as before, the
number of ways in which three places can be filled up is n(n-1)(n-2).
Proceeding in a similar manner, and introducing a new factor for
each step, we have the number of ways in which r places can be
filled as: n(n- 1)(n - 2)(n-3) ... to r factors. The rth factor would be n
- (r - 1) or (n - r + 1). Therefore, the number of permutations of n
things taken r at a time, which is represented by nPr, is: n(n - l)(n-
2)...(n-r+1).
Using factorial notations, nPr = n!/(n - r)!. (Note: Factorial of any
number p is nothing but the product of all numbers from 1 through p
including p itself. For example, factorial of 4 would be 1 x 2 x 3 x 4
which is 24).
The number of permutations of n things taken all at a time would
thus be n!.
Instance: In how many ways can four persons be seated if there
are six seats available?
Explanation: The first person may seat himself in 6 ways (since 6
seats are available); the second in 5; the third in 4; and the fourth
in 3; and since each of these ways may be associated with each of
the others, the required answer is 6 x 5 x 4 x 3, or 360.
Instance: In how many ways can six persons be arranged if there
are only four seats available (two persons have to stand)?
Explanation: This problem is different from the previous one, but
mathematically it can be treated in the same manner. Instead of a
person choosing-any of the seats to lead to an arrangement, we have
to imagine a seat choosing any of the persons to lead to an
arrangement. Thus, the first seat can pick any of the 6 persons; the
second any of the remaining 5, and so on. Consequently the answer
again is 6 x 5 x 4 x 3 = 360.
(Note: This problem can also be solved in a more meaningful manner
by using combinations. First select 4 persons out of 6 who would be
the ones to sit, and later arrange these selected persons in all

CREA – All Rights Reserved I U Training Services 174


Permutations & Combinations Number Math

possible manners on the seats. However, this would imply knowledge


of combinations which is dealt with later).
Instance: How many different numbers can be formed by using six
out of nine digits 1,2,3,… 9?
Explanation: The solution is permutations of 9 different things
taken 6 at a time, which is 9P6 = 60480.

Number of permutations of n different items taken r at a


time, when each item may repeat any number of times in
any one arrangement
This is the same thing as finding the number of ways in which we can
fill up r places when we haven different things at our disposal, and if
repetition is allowed.
The first place many be filled up in n ways, since any one of then
things may be taken. When the first place has been filled up in any
one of these ways, the second place can then again be filled up in n
ways (not n - 1, since repetition is allowed); and since each way of
filling up the first place can be associated with each way of filling
up the second, the number of ways in which the first two places can
be filled up is given by the product n x n. When the first two places
have been filled up in any way, the third place can be filled up again
in n ways. Reasoning as before, the number of ways in which three
places can be filled up is n x n x n.
Proceeding in a similar manner, and introducing a new factor for
each step, we have the number of ways in which r places can be
filled as n x n x n x n to r factors. Hence the required number of
arrangements is nr
Instance: In how many ways can 5 prizes be given away to 4 boys,
when each boy is eligible for all the prizes?
Explanation: Any one of the prizes can be given in 4 ways. Thus the
first prize can be given in 4 ways and then any one of the remaining
prizes can also be given in 4 ways (It is possible for the second
prize to be awarded to someone who has already received a prize).
Thus two prizes can be given away in 4 x 4 ways, three prizes in 4 x

CREA – All Rights Reserved I U Training Services 175


Permutations & Combinations Number Math

4 x 4 ways, and so on. Hence the 5 prizes can be given away in 4 x


4x4x4x4 = 1024ways.

Circular Permutations
Circular permutations are related to the arrangement of objects in
the cyclic order. It is interesting to note that in a circular
arrangement, there is neither a beginning nor an end. In case of
such an arrangement we have to consider the relative position of
different objects i.e., the circular permutations are different only
when the relative order of the objects is changed. Otherwise they
are the same.
As the number of circular permutations depends mainly on the
relative position of the objects (assume n objects), we fix the
position of one object and then arrange the remaining (n - 1) out of
the n objects in all possible ways. This can be done in (n - 1)! ways.
Instance: In how many ways can a party of seven gentlemen and 6
ladies be seated at a round table so that no two ladies are seated
together?
Explanation: Let us first arrange the gentlemen around the table
which can be done in (7 - 1)! ways because there are seven
gentlemen. Now when the gentlemen have been seated, there are
seven places (Not six - which would have been the case if they are
sitting in a row) for the ladies. The ladies can then sit in 7P6 ways.
Note that the arrangement for the ladies is not considered to be
circular since a well-defined beginning and an end can be considered
with respect to the already seated gentlemen. Hence the total
number of arrangements would be 6! x 7P6 = 362880 ways.

Permutations of Objects Not All Different


The number of different arrangements of n objects out of which n 1
are alike of the first kind, n2 are alike of the second kind, ..., and nk
are alike of the kth kind, is given by n!/(n1! x n2! x n3! x … x nk!)
where n1, n2, n3,,..., nk are distinct classes.
Let x be the number of arrangements possible for the above. Now in
any one instance of the arrangements, there are n 1 similar objects.

CREA – All Rights Reserved I U Training Services 176


Permutations & Combinations Number Math

If these were however made dissimilar, then they could have been
rearranged between themselves in (n1)! ways without changing the
positions of any other objects. Thus for each arrangement instance
we would have (n1)! more possibilities. Thus for the x arrangements,
we would have x(n1)! arrangements. Repeatedly applying the same
logic for other classes of similar objects (n 2, n3,…, nk) we would have
x(n1)!(n2)!... (nk)! arrangements if all the objects would be made
dissimilar in all classes. This as we already know is nothing but n!.
We can arrive at the formula for x from above easily.
Instance: Find the number of permutations of the letters of the
word ―MATHEMATICS‖.
Explanation: The word ―MATHEMATICS‖ consists of 11 letters of
which 2 are ―A‖, 2 are ―M‖, 2 are ―T‖ and the rest all are different.
Thus the total number of arrangements by the formula becomes 11!
/ (2! x 2! X 2!) = 4989600.
Instance: How many different words (which may be non-meaningful)
can be made out of the letters of the word ―ACCOMMODATION‖,
taking all at a time? In how many of these will the vowels occupy the
even places?
Explanation: The word ―ACCOMMODATION‖ consists of 13 letters,
out of which there are 3 ―O‖, 2 ―A‖, 2 ―C‖, 2‖M‖ and the rest are all
different. Hence the required number of words (or arrangements)
would be 13!/(3! x 2! x 2! x 2!). Since the word consists of 13 letters,
there are six even places which can be filled up by 6 vowels - 2‖A‖,3
―O‖ and 1 ―I‖ in 6!/(3!x2!) or 60 ways. Further the remaining 7 odd
places can be filled up by 7 consonants - 2 ―C‖, 2 ―M‖ are the rest
different in 7! / (2! x 2!) or 1260 ways. The required number of
arrangements is the product of the above two terms yielding the
answer of 75600.

Restricted Permutations
The number of permutations of n different objects taken r at a
time in which m particular objects are always excluded is (n-m)Pr This
formula is obvious since the original objects available for choice is
limited to n - m itself. The number of permutations of n different

CREA – All Rights Reserved I U Training Services 177


Permutations & Combinations Number Math

objects taken r at a time in which m particular objects are always


included is (n – m)P (r – m) X rPm
Instance: How many different numbers of four digits can be
formed with the numbers 1,2,3,4,5,6,7 and 8 when (a) 5 and 6 are
not to be included and (b) when 5 and 6 are to be included?
Explanation: We have totally eight digits. In the first case we
cannot use two of them. The remaining 6 digits can be arranged in
6
P4 ways to form 360 four digit numbers which do not contain 5 and
6. When we want 5 and 6 to be always included, 2 places out of 4 are
to be filled up by them and the remaining 2 places are to be filled up
by the remaining 6 digits. Thus the number of numbers of four
digits in this case would be 4P2 x 6P2 which is 360.
The problem can be solved more meaningfully by using combinations
where we first choose the two numbers from the remaining 6
numbers (other than 5 and 6) and then arrange the chosen 4
numbers in all possible manners to get the total number of
arrangements. (Using the combinations formula this becomes 6C2 x 4!
or 15x24 = 360).
Combinations
Each of the different sub-sets or groups or selections which can be
formed of some or all of the elements of a given set is called a
combination of n objects taken some or all at a time. The number of
these sub - sets or groups or selections are called the combinations.
The permutations are based on the order of different objects in
the arrangements. Whereas, the order of elements has no
significance in the case of combinations.
Combinations of N Different Objects
The number of combinations of n different objects taken r at a
time is given by nCr = n!/((n - r)! x r!).
Let the number of combinations of n objects taken r at a time be x.
In every combination there are r objects and these r objects can be
permuted in r! ways between themselves. Since there are x such
combinations and each combination constitutes r! Permutations, by
the multiplication principle, we get x(r!) permutations. This is
however the number of permutations of n objects taken r at a time

CREA – All Rights Reserved I U Training Services 178


Permutations & Combinations Number Math

which is nPr. Thus x is nPr/r!. Substituting the formula for nPr we can
arrive at the expression for nCr
Note: The number of combinations of n things r at a time is equal to
the number of combinations of n things taken n - r at a time. Thus
n
Cr = nC(n – r). Basically this means that the number of ways r things
can be selected out of n things available is the same as the number
of ways n - r things can be rejected out of the n things available.
Instance: In how many ways can a student choose a programme of 5
courses if 9 courses are available and two courses are compulsory
for all the students?
Explanation: When two courses are compulsory there are only 3
courses to be chosen out of 7 courses which can be done in 7C3 = 35
ways.
Instance: From 12 books in how many ways can a selection of 5
books be made (a) when one specified book is always included and (b)
when one specified book is always to be excluded?
Explanation: When the specified book is always to be included we
have to choose 4 out of 11 which is possible in 11C4 = 330 ways. When
the specified book is always to be excluded we have to choose 5 out
of 11 which is possible in 11C5 = 462 ways.
Instance: From 7 Men and 4 Women a committee of 6 is to be
formed. In how many ways can this be done if the committee
contains (a) exactly 2 women and (b) at least 2 women?
Explanation: In case (a) we have to choose 2 women out of 4 women,
and 4 men out of 7 men. This can be done in 4C2 and 7C4 ways
respectively. Since each of the groups can be associated with every
other we have multiplication principle giving the answer as 4C2 x 7C4 =
210
In case (b) the committee can contain 4,3 or 2 women. Applying each
condition one by one we arrive at the summation of ( 7C2 x 4C4 ) + (7C3
x 4C3) + (7C4 x 4C2) which is equal to 210 + 140 + 21 = 371.
The problem can also be solved by taking all possible combinations
(no distinction between men and women is done) and then removing
from them the combinations where there are 0 or 1 woman only.

CREA – All Rights Reserved I U Training Services 179


Permutations & Combinations Number Math

This can be formulated as 11C6 - (7C6) - (7C5 x 4C1) which is 462 – 7 –


84 = 371
Restricted Combinations
The number of combinations of n objects taken r at a time in which
m particular objects are (a) always included (b) never included is and
(n – m)
C (r – m) X (n-m)Cr respectively.
In the first case, the number of objects to be selected is reduced
along with the number of objects existing from which selection has
to be done. In the second case, the number of objects existing from
which a selection has to be done is reduced.
Instance: In how many ways can a cricket team of eleven players be
chosen out of 19 players when (a) three particular players are to be
always included (b) two particular players are always to be excluded.
Explanation: (a) We just have to choose 8 players from 16 since
three have already been chosen. This would be 16C8 = 12870.
(b) We have to choose 11 out of 17 players only since two of them
are not to be considered. Hence 17C11 = 12376. 4C1) which is 462-7-
84. 371.

Number of ways in which m+n things can be divided into


two groups containing m and n things respectively
This splitting is equal to finding the number of combinations of m + n
things m at a time, for every time we select one group of m things
we leave a group of n things behind. Thus the required number would
be m+nCm = (m + n)!/(m! x n!).
If m = n, the groups are e‘qual, and in this case the number of
different ways of subdivision is (2m)!/(m! x m! x 2!) because in any
one way it is possible to interchange the two groups without
obtaining a new distribution.
The same logic can be recursively applied to obtain the number of
ways in which m + n + p things can be divided into three groups
containing m, n and p things. This would be equal to (m + n + p)!/(m! x
n! x p!). Again if m = n = p, then the number of ways would be
(3m)!/(m! xm! Xm! x 3!).

CREA – All Rights Reserved I U Training Services 180


Permutations & Combinations Number Math

Instance: In how many ways can 15 cadets be divided into three


equal groups 7 In how many ways can they be divided into three
different regiments (each non - interchangeable with the other)
with five cadets bleach?
Explanation: The number of ways they can be divided into three
equal groups is 15! / (5! x 5! x 5! x 3!). In case of the non -
interchangeable regiments it is 15!/ (5! x 5! x 5!).
The total number of ways in which it is possible to make a
selection by taking some or all of n things
Each thing can be dealt with in two ways - either it is taken (or
selected) or it is not. Also each selection may be associated with
every other selection and thus the multiplication principle can be
applied. The final number of ways with n things would be 2 x 2 x 2 x
2 x ... to n factors which is 2n. But this would include the case where
all the things are left out. Rejecting this case we arrive at 2n - 1.
This is also known as the total number of combinations of n things.
Instance: A man has 5 friends; in how many ways may he invite one
or more of them to dinner?
Explanation: He has to invite 5 friends. Both may either be
selected or not. So the number of ways would be 2 - 1 = 31. Another
approach can also be tried as given. The friends can either be
invited in terms of 1 or 2 or3 ... upto 5. Thus he has to select 1 out
of 5, or 2 out of 5, or 3 out of 5.. or 5 out of 5. This formulates to
5
C1 + 5C2 +5C3 + 5C4 + 5C5 = 5 + 10 + 10 + 5 + 1 = 31.
In case of arrangements, we have to consider the possible
arrangements for each chosen selection. This can be worked out
easily group - wise. The final result would then be equal to (5x4!/3!)
+ (3x4!/(2!x20) + (30x4!/2!) + (15x4!) = 758.
The number of ways in which it is possible to make a selection by
taking some or all of p + q + r... things, out of which p are alike of
one kind, q are alike of another kind and so on
The p things may be disposed of in p + 1 ways; for we may take 0, 1,
2, 3, ... p of them. Similarly the q things may be disposed of in q + I
ways and so on. Hence the number of ways in which all the things
maybe disposed of is (p + 1) X (q + 1) X (r + 1)...

CREA – All Rights Reserved I U Training Services 181


Permutations & Combinations Number Math

However, this includes the case in which none of the things are
being chosen and that has to be removed to yield a final answer of
(p + 1) X (q + 1) X (r + 1)...-1
Instance: How many different choices can be made of selections
out of 15 Physics, 10 English and 12 Mathematics books when at
least one book is to be selected?
Explanation: There are (15 + 1) ways of dealing with 15 Physics
books including the case of not choosing any Physics book at all.
Using similar logic, the total number of ways of dealing with all
these books would be (15 + 1)X(10 + 1)X(12 + 1) = 2288. But since at
least one book should be chosen and 2288 would also include the
case when none of them is chosen, we have the final answer as 2287.

Permutations or combinations of n things taken r at a time, when


the things are not all different
These problems have to be solved on a case by case basis. Each case
discussed must consist of fixed number of groups of repeatable
items for which the permutations or combinations are calculable.
Addition of all the exhaustive cases would lead to the total number
of permutations or combinations.
Instance: Find the number of ways in which (a) a selection and (b) an
arrangement, of four letters can be made from the letters of the
word ‗proportion‖,
Explanation: There are 10 letters of six different sorts. They can
be categorized as follows: 3 letters - O; 2 letters - P, R; 1 letter- T,
I, N. The groups can now be structured in the following manner:
 Three alike, one different (1)
 Two alike, two others alike (2)
 Two alike, two different (3)
 All four different (4)
(1) Three alike can be chosen in 1 way and one different in 5 ways.
Total is 5.
(2) Two groups out of a possible of three (O, P, R) can be chosen in
3 ways.

CREA – All Rights Reserved I U Training Services 182


Permutations & Combinations Number Math

(3) One group out of a possible can be chosen in 3 ways and then the
remaining two letters can be chosen from the available 5 different
groups in 5C2 = 10 ways. Total ways are 30.
(4) Four different letters from six groups can be chosen in 6P4 = 15
ways.
Thus the total selections possible are 5 + 3 + 30 + 15 = 53 ways.
In case of arrangements, we have to consider the possible
arrangements for each chosen selection. This can be worked out
easily group-wise. The final result would then be equal to (5 x 4!/3!)
+ (3x4!/(2!x2!)) + (30x4!/2!) + (15x4!) = 758

Brain Drill
Permutations
1. A gentleman has got 6 types of note papers, 7 different types of
pens and 4 different types of inkpots. In how many different ways
can he begin to write a letter?
1. 186 2. 168 3. 210 4. None of
these

2. There are 8 buses running between Delhi and Meerut. In how


many ways can a man go from Delhi to Meerut and return by a
different bus?
1. 56 2. 64 3. 42 4. 49

3. In how many ways 5 letters can be posted in 7 letter boxes


available in the locality?
1. 3125 2. 625 3. 35 4. 16807

4. A carpenter has twelve patterns of chairs and five patterns of


tables. In how many ways can he make a pair of chair and table?
1. 24 2. 78 3. 60 4. 44

CREA – All Rights Reserved I U Training Services 183


Permutations & Combinations Number Math

5. How many three digit numbers can be formed from digits 2, 3, 7,


8, 9 and how many out of these will be greater than 700?
1. 12 2. 24 3. 48 4. 36

6. How many different four letter words can be formed out of the
letters of the word ―LOGARITHMS‖, if repetition of letters is not
allowed. (The words may or may not have a meaning).
1. 720 2. 4320 3. 5040 4. 151200

7. There are 10 multiple choice questions in an examination. How


many sequences of answers can be possible, if the first six
questions have four choices and the last 4 questions have 5 each?
1. 102400 2. 2560000 3. 204800 4. None of
these

8. In how many ways can 100 electorates vote for three candidates
standing for the election of the post of President of their
association?
1. 1000000 2. 300 3. 900 4. 3100

9. How many five different letter words can be formed out of the
word ―LOGARITHMS‖? (The words may or may not have any
meaning).
1. 30240 2. 120 3. 252 4. None of
these
10. If four times the number of permutations of n things 3 together
is equal to five times the number of permutations of n-1 things 3
together, find n.
1. 16 2. 15 3. 14 4. 13

11. How many permutations, those which begin with ‗t‘ and end with
‗e‘, can be made out of the letters of the word ―triangle‖?
1. 120 2. 40320 3. 720 4. 5040

12. If 18Cr = 18Cr+2, find rC5.

CREA – All Rights Reserved I U Training Services 184


Permutations & Combinations Number Math

1. 28 2. 56 3. 70 4. None of
these

13. A book shelf contains 7 mathematics and 5 physics textbooks.


How many groups of 3 mathematics and 3 physics textbooks can be
selected out of them?
1. 175 2. 350 3. 700 4. 35

14. How many arrangements can be made out of the letters of the
word ―draught‖, the vowels never being separated?
1. 1440 2. 720 3. 360 4. 240

15. In a town council there are 25 councilors and 10 aldermen; how


many committees can be formed each consisting of 5 councilors and
3 aldermen?
1. 53130 2. 1593900 3. 144000 4. 6375600

Combinations
1. In how many ways can committee of 4 be chosen from 10
members?
1. 40 2. 10P4 3. 10000 4. None of
these

2. Ramesh Rao wants to choose 3 English books and 4 Hindi books


from a set of 5 English and 5 Hindi books. In how many ways can he
do the selection?
1. 10 2. 25 3. 50 4. 12

3. In how many ways can the letters of the word ―IDEAL‖ be


arranged so that all the consonants are always together?
1. 24 2. 48 3. 120 4. None of
these

CREA – All Rights Reserved I U Training Services 185


Permutations & Combinations Number Math

4. How many different signals can be made by hoisting 6 differently


coloured flags one above the other, when any number of them may
be hoisted at once?
1. 1236 2. 1440 3. 1956 4. None of
these

5. Five boys and four girls are to be seated for a photograph in a


row. It is desired that no two girls sit together. Find the number of
ways in which they can be so arranged.
1. 21600 2. 3600 3. 14400 4. 43200

6. A Council consists of members of two parties there being 6 of


one and 5 of the other. In how many ways can a committee of 5 be
selected so that each party is represented and the larger party
possessed the majority in the committee?
1. 75 2. 200 3. 275 4. 350

7. Find the number of diagonals of a polygon of 80 sides.


1. 3080 2. 1540 3. 6400 4. 3160

8. How many factors are there of 2160 (excluding 1 and 2160)?


1. 36 2. 37 3. 38 4. 39

9. Find the number of ways in which 6 men and 4 women are to sit
for a dinner at a round table so that no two ladies are to sit
together.
1. 1200 2. 2400 3. 60000 4. 43200

10. The letters of the word LABOUR are permuted in all possible
ways and the words thus formed are arranged as in a dictionary.
What is the rank of the word LABOUR?
1. 242 2. 240 3. 244 4. None of
these

CREA – All Rights Reserved I U Training Services 186


Permutations & Combinations Number Math

11. How many different numbers can be formed by taking four of


the digits 3, 4, 7, 5, 8, 1?
1. 120 2. 360 3. 1440 4. None of
these

12. From a bag containing 8 red balls and 7 white balls, in how many
ways can 4 white balls and 5 red balls be selected?
1. 2240 2. 1176 3. 1750 4. 1960

13. In how many ways can 7 differently coloured beads be strung on


a necklace?
1. 360 2. 720 3. 120 4. 5040

14. In how many ways can an international committee of four be


formed out of 6 Asians, 3 Europeans and 4 Americans if the
committee is to have at least one from each of the three regional
groups?
1. 720 2. 360 3. 240 4. 144

15. Find the number of diagonals of an ‗n‘ sided polygon


1. nC2 2. n! 3. nC2 - n 4. nP3

16. Five papers are set in an examination and a minimum is to be


obtained in each paper for passing the examination. In how many
ways can a student fail?
1. 31 2. 10 3. 12 4. 25

Super Brain Drill


1. A fruit seller has a large stock of apples, pears and
bananas. How many different kinds of packages can he make if each
package contains 8 fruits?
1. 45 2. 54 3. 64 4. 512
2. Out of 7 consonants and 4 vowels, how many words can be made
each containing 3 consonants and 2 vowels?

CREA – All Rights Reserved I U Training Services 187


Permutations & Combinations Number Math

1. 720 2. 25200 3. 210 4. None of


these
3. The question paper of mathematics contains ten questions divided
into two groups of five questions each. In how many ways can an
examinee answer six questions taking at least two questions from
each group.
1. 100 2. 150 3. 250 4. 200
4. In how many ways can 4 Hindi and 1 English book be placed on a
shelf so that the English book is always in the middle, the selection
being made from 7 Hindi and 3 English books?
1. 1440 2. 5120 3. 2520 4. 840
5. From 3 capitals, 5 consonants, and 4 vowels, how many words can
be made, each containing 3 consonants and 2 vowels, and beginning
with a capital?
1. 21600 2. 7200 3. 3600 4. 180
6. An eight-oared boat is to be manned by a crew chosen from 11
men, of whom 3 can steer but cannot row, and the rest can row but
cannot steer. In how many ways can the crew be arranged, if two of
the men can only row on the bow side? (The crew consists of eight
rowers and one steerer).
1. 25920 2. 5760 3. 8640 4. 360
7. Nine articles are to be placed in nine boxes one in each box. Five
of them are too big for three of the boxes. Find the number of
possible arrangements.
1. 720 2. 8640 3. 17280 4. 4220
8. A gentleman invited a party of 10 friends to a dinner and places
half of them on one side of a long table and half on the other side.
If 3 particular men desire to sit on one side and two on the other,
find the number of ways in which he can arrange the guests.
1. 144000 2. 288000 3. 72000 4. None of
these
9. In how many ways can three numbers be selected out of first
(2n+ 1) natural numbers so that the three selected are in an
arithmetic progression?

CREA – All Rights Reserved I U Training Services 188


Permutations & Combinations Number Math

1. 2n 2. n3 3. n(n+2) 4. n2
10. A candidate is required to answer 6 out of 10 questions which
are divided into groups, each containing 5 questions and he is not
permitted to attempt more than 4 from any group. In how many
ways can he make up his choice?
1. 184 2. 192 3. 200 4. None of these
11. In an examination the question paper contains three different
sections A, B and C containing 4, 5 and 6 questions respectively. In
how many ways, a candidate can make a selection of 7 questions,
selecting at least 2 questions from each section?
1. 890 2. 1420 3. 2700 4. 3140
12. A tea party is arranged for 16 people along the two sides of a
long table with 8 chairs on each side. 4 men wish to sit on one
particular side and 2 on the other side. In how many ways can they
be seated?
1. 10C4 x 8! 2. 10P4 X (8!)2 3. 10C4 X (8!)2 4. 4! X 2! X (8!)2
13. How many 10 digit nos can be written using the digits 1 and 2?
1. 10C1 + 9C2 2. 210 3. 10C2 4. 10!
14. There are 4 letters and 4 directed envelopes. The number of
ways in which all the letters can be put in the wrong envelope is
1. 8 2. 9 3. 16 4. 18

CREA – All Rights Reserved I U Training Services 189


5
Taking the Time to Work on
Distances

CREA – All Rights Reserved I U Training Services 190


PRE-LESSON Challenge – See What You Know
and Remember
Time Limit – 5 minutes

1. A motor car does a journey in 10hrs, the first half


at 21 km/hr and the second half at 24 km/hr. Find
the distance.
2. Two trains 121 metres and 99 metres in length
respectively are running in opposite directions, one at
the rate of 40 km and the other at the rate of 32 km
an hour. In what time will they be completely clear
of each other from the moment they meet?
3. A man can row 7 km/hr is still water. In a stream
which is flowing at 3 km/hr, it takes him 7 hrs to row
to a place and back. How far is the place?
4. A clock is set right at 12:00 p.m. The clock loses
30 minutes in 24 hours. What will be the true time
when the clock indicates 10:00 a.m. on the 5 th day?

CREA – All Rights Reserved I U Training Services 191


Taking the Time to Work on Distances Number Math

In This Chapter
 Figuring the distance traveled as the sum of two
different distances
 Equating two distances that have different rates of
speed
 Solving problems on Trains, Clocks and Races

This chapter deals with a lot of moving around. People and cars and
trains are moving around all the time. Most problems involve moving
around and the distance traveled. The typical distance problems use
the relationship that distance is equal to rate times time, d = rt. By

Formulae
(i)

(ii)

(iii) Distance = Speed x Time


(iv) If the speed of a body is changed in the ratio a:b, then
the ratio of the time taken changes in the ratio b:a.

(v) x km/hr =

(vi) x metres/sec =

using this equation, you deal with trains meeting somewhere in the
middle of nowhere and wives catching up to forgetful husbands.
If a certain distance is covered at x km/hr and the
same distance is covered at y km/hr then the
2xy
average speed during the whole journey is
x y
km/hr.

CREA – All Rights Reserved I U Training Services 192


Taking the Time to Work on Distances Number Math

Instance: A man covers a certain distance by car driving at 70


km/hr and he returns back to the starting point riding on a scooter
at 55km/hr. Find his average speed for the whole journey.
2  70  55
Explanation: Average speed = km / hr  61.6 km / hr.
70  55
Direct Formula:
Required Distance
=

Instance: A boy walking at a speed of 10 km/hr reaches his school


15 minutes late. Next time he increases his speed by 2 km/hr, but
still he is late by 5 minutes. Find the distance of his school from his
house.
Explanation: Here, the difference in time = 15 – 5 = 10 minutes.
1
= hours.
6
His speed during next journey = 10 + 2 = 12 km/hr.
12  10 1
 Required distance =   10 km .
12  10 6

Direct Formula:

Where, S1 = Speed during first half and


S2 = Speed during second half of journey.

Instance: A motor car does a journey in 10hrs, the first half at 21


km/hr and the second half at 24 km/hr. Find the distance.

CREA – All Rights Reserved I U Training Services 193


Taking the Time to Work on Distances Number Math

x
Explanation: Let the distance be x km. Then km is travelled at a
2
x
speed of 21 km/hr and km at a speed of 24 km/hr.
2
Then time taken to travel the whole journey
x x
  10 hrs
2  21 2  24
=
2  10  21  24
so, x   224 km
(21  24)

If two persons A and B start at the same time in


opposite directions from two points and after passing
each other they complete the journey in ‘a’ and ‘b’ hrs

respectively, then A’s speed : B’s speed = b: a


Instance: A man sets out to cycle from Delhi to Rohtak, and at the
same time another man starts from Rohtak to cycle to Delhi. After
1 4
passing each other they complete their journeys in 3 and 4
3 5
hours respectively. At what rate does the second man cycle if the
first cycles at 8 km per hour?
Explanation: If two persons (or train) A and B start at the same
time in opposite directions from two points, and arrive at the point a
and b hrs respectively after having met, then
A‘s rate: B‘s rate = √b : √a (from the Precept)
Thus in the above case
4
4
1st man ' s rate 5 6

2nd man ' s rate 1 5
3
3

CREA – All Rights Reserved I U Training Services 194


Taking the Time to Work on Distances Number Math

5 2
 2nd man ' s rate   8  6 km / hr.
6 3

TRAINS
This is same as Time & Distance. The only difference is that the
length of the moving object (Train) is also considered in this
chapter.
Some important things to be noticed in this chapter are:
1. When two trains are moving in opposite directions their
speeds should be added to find the relative speed.
2. When they are moving in the same direction the relative
speed is the difference of their speeds.
3. When a train passes a platform it should travel the
length equal to the sum of the length of train & platform both.

 Trains passing a telegraph post or a stationary man


Instance: How many seconds will a train 100 metres long running at
the rate of 36 km an hour take to pass a certain telegraph post?
Explanation: In passing the post the train must travel its own length.
5
Now, 36 km/hr = 36 
 10m / sec.
18
100
 required time   10 sec onds.
10

 Trains crossing a bridge or passing a railway


station
Instance1: How long does a train 110 metres long running at the
rate of 36 km/hr take to cross a bridge 132 metres in length?
Explanation: In crossing the bridge the train must travel its own
length plus the length of the bridge.

CREA – All Rights Reserved I U Training Services 195


Taking the Time to Work on Distances Number Math

5
Now, 36 km/hr = 36   10m / sec.
18
242
 required time   24.2 sec onds.
10

 Trains running in opposite direction


Instance: Two trains 121 metres and 99 metres in length
respectively are running in opposite directions, one at the rate of 40
km and the other at the rate of 32 km an hour. In what time will
they be completely clear of each other from the moment they meet?
Explanation: As the two trains are moving in opposite directions
their relative speed = 40 + 32 = 72 km/hr. i.e. they are approaching
each other at 72 km/hr or 20m/sec.
Total length 121  99
the required time    11 sec onds.
Re lative speed 20

 Trains running in the same direction


Instance: If the trains were running in the same direction, in what
time will they be clear of each other?
20
Explanation: Relative speed = 40 -32 = 8 km/hr= m / sec
9
Total length = 121 + 99 = 220m.

Total length 220


the required time    9  99 sec onds.
Re lative speed 20

 Train passing a man who is walking


Instance : A train 110 metres in length travels at 60 km/hr. In
what time will it pass a man who is walking at 6 km an hour (i) against
it; (ii) in the same direction?

CREA – All Rights Reserved I U Training Services 196


Taking the Time to Work on Distances Number Math

55
Explanation: (i) Relative speed = 60 + 6 = 66 km/hr = m / sec
3
110
 required time   3  6 sec onds.
55
(ii) Relative speed = 60 -6 = 54 km/hr = 15 m/sec.
110 1
 required time   7 sec onds.
55 3
 Miscellaneous
Instance: Two trains start at the same time from Hyderabad and
Delhi and proceed toward each other at the rate of 80 km and 95 km
per hour respectively. When they meet, it is found that one train
has travelled 180 km more than the other. Find the distance
between Delhi and Hyderabad.
Explanation: Faster train moves 95 -80 = 15 km more in 1 hr.
 Faster train moves 180 km more in
1
 180  12 hrs.
15
Since, they are moving in opposite directions, they cover
a distance of 80 + 95 = 175 km in 1 hr.
 in 12 hrs they cover a distance = 175 x 12 = 2100km
 distance = 2100 km.

STREAMS
Normally by speed of the boat or swimmer we mean the speed of the
boat (or swimmer) in still water. If the boat (or the swimmer) moves
against the stream then it is called upstream and if it moves with
the stream, it is called downstream.
If the speed of the boat (or the swimmer) is x and if the speed of
the stream is y then, while upstream the effective speed of the boat
= x – y and while downstream the effective speed of the boat = x + y.

CREA – All Rights Reserved I U Training Services 197


Taking the Time to Work on Distances Number Math

If x km per hour be the man’s rate in still water,


and y km per hour the rate of the current. Then
x + y = man’s rate with current
x – y = man’s rate against current
Adding and subtracting and then dividing by 2.
x = ½ (man’s rate with current + his rate against
current)
y = ½ (man’s rate with current – his rate against current)
Hence we have the following two facts:
(i) A man’s rate in still water is half the sum of his rates with
and against the current.
(ii) The rate of the current is half the difference between the
rates of the man with and against the current.

Instance: A man can row upstream at 10km/hr and downstream at


16km/hr. Find the man‘s rate in still water and the rate of the
current.
1
Explanation: Rate in still water = (10  16)  13km / hr
2
1
Rate of current = (16  10)  3km / hr .
2
A man can row x km/hr in still waters. If in a stream
which is flowing at y km/hr, it takes him z hrs to row
to a place and back, the distance between the two
z( x2  y 2 )
places is .
2x
Instance: A man can row 7 km/hr is still water. In a stream which is
flowing at 3 km/hr, it takes him 7 hrs to row to a place and back.
How far is the place?
(7)2  (3)2
Explanation: By the Precept, distance = 7   20 km .
27

CREA – All Rights Reserved I U Training Services 198


Taking the Time to Work on Distances Number Math

A man rows a certain distance downstream in x hours


and returns the same distance in y hrs. If the
stream flows at the rate of z km/hr then the speed
z( x  y)
of the man in still water is given by km/hr.
yx
Instance: Ramesh can row a certain distance downstream in 6 hours
and return the same distance in 9 hours. If the stream flows at the
rate of 3km per hour find the speed of Ramesh in still water.
Explanation: By the above Precept:
3(9  6)
Ramesh‘s speed in still water =  15 km / hr .
96

RACES
Race is a contest of speed in running, riding, driving, sailing or
rowing. If all the contestants reach the winning post at the same
time, then the race is said to be a dead heat.

Suppose A and B are two contestants in a race. If before the start


of the race, A is at the starting point and B is ahead of A by 20m,
then A is said to give B a start of 20 m. To cover a race of 100 m in
this case, A will have to cover a distance o 100m whereas B has to
cover 80m only.

In a 100 m race, ― A can give B 15m ―or ― A can give B a start of


15m‖, or ―A beats B by 15m ― means that while B runs 85 m, A runs
100 m.

Instance: A can run 1,000 m in 4 minutes while B can in 5 minutes.


How many metres start can A give B in a 1000 m race so that it may
end in a dead heat?
Explanation: The race will end in a dead heat if and only if both run
for 4 minutes.
1000
In 4 minutes B can cover  4  800m.
5

CREA – All Rights Reserved I U Training Services 199


Taking the Time to Work on Distances Number Math

Thus A give B a start of 1,000 m – 800 m = 200 m.

CLOCKS
The face of a clock is a circle whose circumference is divided into
60 equal parts, called minute spaces.

Since the angle in a circle is 3600, each minute space has an angle of

 3600  0
 60   6 at the centre of the dial.
 
The face of the clock is divided into 12 equal parts called hour
spaces.

 3600 
 12   30
0
Each hour space has an angle of .
 
Thus, 5 minute spaces make 1 hour space.

The clock has two hands – the smaller one, called hour hand and the
longer one called the minute hand.

The hour hand completes one rotation or moves through


an angle of 3600 in 12 hours. Therefore, the angular
speed of the hour hand is
0
3600 300  1 
  per min ute.
12 hour  2 
The minute hand completes one rotation or moves
through an angle of 3600 in 60 minutes. Therefore, the
angular speed of the minute hand is

3600
 60 per min ute.
60

CREA – All Rights Reserved I U Training Services 200


Taking the Time to Work on Distances Number Math

Since both the hands move in the same direction their


relative speed is
0
1
6     5.50 per min ute.
0

2
The minute hand gains 55 minutes on the hour hand in one hour.

ANGLE BETWEEN THE HOUR AND MINUTE HANDS OF A


CLOCK AND THEIR FREQUENCY
FREQUENCY
EACH EVERY 12 EVERY 24
POSITION INSTANCE
HOUR HOURS HOURS
Straight Line
6 o‘ clock Once 11 Times 22 Times
(Angle = 1800)
Coinciding
12 o‘ clock Once 11 Times 22 Times
(Angle = 00)
Right Angle
3 ‗o clock Twice 22 Times 44 Times
(Angle = 900)

If a clock gains time it runs fast. If it loses time it


runs slow.
If both the hands start together from the same
position, both hands will coincide after
360  2 5
 65 min utes.
11 11

Instance: A clock is set right at 12:00 p.m. The clock loses 30


minutes in 24 hours. What will be the true time when the clock
indicates 10:00 a.m. on the 5th day?
Explanation: Total time from 12:00 p.m. on day 1 to 10:00 a.m. on 5th
day = 94 hours.

CREA – All Rights Reserved I U Training Services 201


Taking the Time to Work on Distances Number Math

1
The clock loses 30 minutes in a day, therefore 23 hours
2
of the clock = 24 hours of correct clock.
 24  94  2 
Therefore 94 hours of the clock =   hours
 47 
of correct clock = 96 hours of correct clock.
Therefore, correct time is 12:00 p.m.

Brain Drill
1. A cyclist travels a distance of 4800 m in 4 minutes. What is his
speed in kmph?
1. 54 2. 72 3. 76 4. 56

2. The speed of a bus is 60 kmph. The bus starts at Hyderabad at 6


a.m. and reaches Chennai at 6 p.m. on the same day. Find the
distance between Hyderabad and Chennai.
1. 600 km 2. 1,200 km 3. 1,500 km 4. 720 km

3. A car traveling with a speed of 40 kmph covers the distance


between A and B in 3 hours. A person starts from B, with a speed of
20 kmph,. How much time will he take to reach A?
1. 5 hours 2. 6 hours 3. 4 hours 4. None of these

4. Two persons A and B cover a distance of 200m in 20 and 16


seconds respectively. Find the distance covered by the person A
when B reaches his destination
1. 140m 2. 40m 3. 160m 4. 50m

5. The wheel of a car is 220 cm in circumference. The wheel makes


700 revolutions in 2 minutes 6 seconds. Find the speed of the car.
1. 40 kmph 2. 44 kmph 3. 46 kmph 4. None of these.

CREA – All Rights Reserved I U Training Services 202


Taking the Time to Work on Distances Number Math

6. A man starts from his residence on a scooter at a uniform speed


of 25 kmph, and reaches his office in 1 hour 24 minutes. In the
evening he took 1 hour 40 minutes to reach his residence from his
office. At what speed did he travel?
1. 20 kmph 2. 21 kmph 3. 22 kmph 4. 23 kmph

7. Two persons M and N started from New Delhi to Agra. Their


speeds are in the ratio 4 ; 5. If M takes 7 hours more than N to
reach Agra, find the time taken by N to reach Agra.
1. 28 hours 2. 35 hours 3. 63 hours 4. None of these

8. Ramesh goes to college with a speed of 10 kmph and returns to


the house with a speed of 20kmph. If he takes 3 hours in all, find
the distance between the college and house.
1. 10 km 2. 15 km 3. 20 km 4. 18 km

9. A man covered one third of certain distance at 30 kmph and the


remaining at 40 kmph. If he took 50 minutes in all, find the distance
travelled by him.
1. 30 km 2. 20 km 3. 40 km 4. 35 km

10. Walking at (7/8) of his usual speed, a person is late to his office
by 15 minutes. Find his usual time to reach the office.
1. 63 min 2. 91 min 3. 105 min 4. 77 min

11. Rahul starts in his car from Mumbai to Khandala, a distance of


208 km, at an average speed of 40 kmph. Arun starts from Khandala
in his car, 1 hour after Rahul started, and reaches Mumbai 12
minutes before Rahul reaches Khandala. Find the ratio of the
speeds.
1. 10 : 13 2. 5 : 6 3. 3 : 4 4. None of these

12. A non-stop bus service is able to cover the distance between two
cities at an average speed of 60 kmph. An ordinary bus service, with
stoppages at intermediate stations, is able to cover the same

CREA – All Rights Reserved I U Training Services 203


Taking the Time to Work on Distances Number Math

distance at an average speed of 45 kmph. The average number of


minutes, for stoppages, per hour for the ordinary bus service is
1. 5 min 2. 10 min 3. 12 min 4. 15 min

13. A monkey tries to ascend a greased pole 10m high. It ascends


3m in the first minute and slips down 2m in the second minute. If it
continues to ascend in the same fashion, find the time taken by it to
reach the top.
1. 15 min 2. 18 min 3. 20 min 4. 22 min

14. Ramesh drives from city A to city B, which are 400km apart, at a
speed of 50 kmph. And on his return journey he drives at a speed of
40 kmph. What is his average speed for the entire journey?
1. 42 kmph 2. 44.44 kmph 3. 48 kmph 4. 46.65 kmph

15. A person can reach his destination 20 minutes early if his speed
was 45 kmph instead of 30 kmph. How far is his destination?
1. 20km 2. 25 km 3. 30 km 4. 35 km

16. If I walk at 2 kmph I miss the bus by 5 minutes. If I walk at 3


kmph I am 5 minutes early to the bus stop. How far do I have to
walk to the bus stop?
1. 500 m 2. 1,000 m 3. 1,200 m 4. 850 m

17. A man covers distance of 50 km in 5 hours, partly on foot and


partly on bicycle, at speeds of 8 and 13 kmph respectively. Find the
distance covered on bicycle.
1. 26 km 2. 24 km 3. 20 km 4. 18 km

18. An athlete is running around a circular track. He takes 60


seconds for one complete round around the track. This athlete
keeps meeting another athlete, running in the opposite direction,
every 15 seconds. How long does the second athlete take for one
complete round around the track?
1. 20 sec 2. 15 sec 3. 30 sec 4. 60 sec

CREA – All Rights Reserved I U Training Services 204


Taking the Time to Work on Distances Number Math

19. A car leaves from P for Q, 1,080 km away, at 12 noon. At 3 p.m.


another car leaves from Q for P at a speed of 90 kmph. What is the
speed of the first car, if the cars met exactly mid-way between P
and Q?
1. 45 kmph 2. 60 kmph 3. 75 kmph 4. 90 kmph

20. Two trucks leave a warehouse at 9 a.m., with one truck traveling
East at 60 kmph and the other West at 40 kmph along a straight
road. At what time will they be 500 km apart?
1. 12 noon 2. 1 p.m. 3. 2 p.m. 4. 3 p.m.

21.Two persons Rakesh and Roshan start simultaneously from A and


B and Walk towards each other They Meet after (1/2) hour and
continue to walk towards their destinations lf Rakesh reaches 25
minutes after Roshan reached his destination find the ratio speeds
of Rakesh and Roshan.
1. 2:3 2. 1:2 3. 3:4 4. None of these

22. Ram and Shyam walk from a place A to place B, a distance of 45


km, at speeds of 4 kmph and 5 kmph respectively. Shyam reaches B
and immediately starts his return journey. On the way back he
meets Ram at C. How far is C from A?
1. 27 km 2. 36 km 3. 40 km 4. 32 km

23. Two persons A and B walk in the same direction at 4 kmph and 6
kmph respectively. If B catches up with A after 9 minutes, what
was the initial distance between them?
1. 400 m 2. 300 m 3. 200 m 4. 100 m

24. The driver of a car, traveling at 35 kmph, sees a bus 40 m ahead


of him traveling in the same direction. In 40 seconds, not only does
he over take the bus but also is 60 m ahead of the bus. What is the
speed of the bus?
1. 26 kmph 2. 25 kmph 3. 27 kmph 4. 24 kmph

CREA – All Rights Reserved I U Training Services 205


Taking the Time to Work on Distances Number Math

25. A thief sights a policeman 50 m ahead of him. He immediately


turns back and starts running at 4 kmph. The policeman chases him
at a speed of 5 kmph. After what time does the police man catch
the thief?
1. 4 min 2. 4 min 40 sec 3. 3 min 4. 3 min 50 sec

26. Find the time taken by a train 150 m long running at 90 kmph to
cross an electric pole.
1. 7 sec 2. 8 sec 3. 6 sec 4. 5 sec

27. A train 100 metres long passes a platform of 500m in 30


seconds. Find the speed of the train.
1. 54 kmph 2. 90 kmph 3. 36 kmph 4. 72 kmph

28. A train passenger counts the telegraph posts along the railway
track as he crosses them. The posts are at intervals of 60m. How
many telegraph posts, maximum, will he come across in a minute, if
the speed of train is 54 kmph?
1. 12 2. 15 3. 20 4. 16

29. A train traveling at a speed of 54kmph passes a platform in 20


seconds. The length of the train is half that of the platform. Find
the length of the platform.
1. 100 m 2. 200 m 3. 150 m 4. 100 m

30. It takes 12 hours for Train A to travel a certain distance. Train


B, which runs 10 kmph faster than Train A, covers the distance in
(3/4) of the time. How fast is each train traveling?
1. 20 kmph and 30 kmph 2. 30 kmph and 40 kmph
3. 25 kmph and 35 kmph 4. 30 kmph and 45 kmph

31. A train 200 m long is traveling at 72 kmph. In how much time will
the train pass a cyclist traveling at 5 m/sec and in the same
direction?

CREA – All Rights Reserved I U Training Services 206


Taking the Time to Work on Distances Number Math

2 1 3
1. 13 sec 2. 13 sec 3. 13 sec 4. 13 sec
3 3 4
32. Find the time taken for a train, 160m long and traveling at 54
kmph, to pass a scooterist traveling in the opposite direction at a
speed of 18 kmph.
1. 8 sec 2. 10 sec 3. 6 sec 4. 5 sec

33. The speeds of two trains are in the ratio 4 : 5. They are going in
opposite directions along parallel tracks. If each takes 3 seconds to
cross a telegraph post, find the time taken by the trains to cross
each other completely.
1. 5 seconds 2. 7 seconds 3. 8 seconds 4. 3 seconds

34. A train 115 metres long running at a speed of 72 kmph crosses a


125 m long train coming in the opposite direction in 6 seconds. What
is the speed of the second train?
1. 72 kmph 2. 60 kmph 3. 54 kmph 4. 80 kmph

35. The lengths of two trains are 120 m and 80 m long. If they are
moving in the same direction, the faster train takes 40 seconds to
completely pass the slower. If they are moving in opposite
directions, they completely pass each other in 10 seconds. Find their
speeds.
1. 45 kmph, 27 kmph 2. 45 kmph, 28 kmph
2. 47 kmph, 25 kmph 4. None of these

36. The distance between two stations A and B is 200km. A train


starts at 7:00 a.m. from A and moves towards B at an average speed
of 20 kmph. Another train starts from B at 6:10 a.m. and move
towards A at an average speed of 30 kmph. At what time will the
two trains meet?
1. 10:30 a.m. 2. 10:05 a.m. 3. 10:10 a.m. 4. 9:10 a.m.

CREA – All Rights Reserved I U Training Services 207


Taking the Time to Work on Distances Number Math

37. Two trains left Bombay VT for Thane at an interval of 10


minutes and at a speed of 30 kmph what is the speed of a train
bound for VT if it met these two trains at an interval of 4 minutes.
1. 30 kmph 2. 36 kmph 3. 40 kmph 4. 45 kmph

38. The speed of a boat upstream and downstream are 10 kmph and
18 kmph respectively. Find its speed in still water.
1. 7 kmph 2. 14 kmph 3. 4 kmph 4. 8 kmph

39. A man can row 18 km down stream in 3 hours and 15 km upstream


in 5 hours. Find the speed of stream.
1. 1.5 kmph 2. 3 kmph 3. 6 kmph 4. 4.5 kmph

40. A boat travels from A to B and comes back in 5 hours. If the


speed of the boat in still water is 5 kmph and the speed of the
current is 1 kmph, find the distance between A and B.
1. 10 km 2. 12 km 3. 11 km 4. 9 km

41. A boat goes 8 km upstream and back to the starting point in 2


hours. The speed of the stream is 3 kmph. Find the speed of the
boat in still water?
1. 9 kmph 2. 7 kmph 3. 5 kmph 4. 10 kmph

42. A boat covers 12 km upstream and 20 km downstream in 11


hours. It covers 20 km upstream and 12 km downstream in 13 hours.
Find the speed of the stream
1. 2 kmph 2. 1 kmph 3. 0.5 kmph 4. 1.5 Kmph

43. The speed of a river is 5 kmph. A person takes twice as much


time to row up as to row down the same distance. Find the rowing
speed of the person in still water.
1. 10 kmph 2. 12 kmph 3. 8 kmph 4. 15 kmph

CREA – All Rights Reserved I U Training Services 208


Taking the Time to Work on Distances Number Math

44. On a river, B is exactly between A and C. If a boat can go from


A to B and back in 5 hours and from A to C in 6 hours, how long
would it take to go from C to A?
1. 3 hours 2. 2 hours 3. 4 hours 4. None of these

45. In a 100 m race A beats B by 20 m or 5 sec. Find the time taken


by A to cover the race.
1. 25 sec 2. 20 sec 3. 15 sec 4. 23 sec

46. A wrist watch was set accurately at 9 a.m. It then started


losing 1 second every 10 minutes. What time was indicated on the
watch, when the actual time is 7 p.m. on the same day?
1. 6:58:30 p.m. 2. 6:58:45 p.m. 3. 6:59:15 p.m 4. 6:59:00 p.m.

47. The clock of municipal tower gains uniformly. On Monday


morning at 6 a.m. the clock was set 10 minutes slow. On Thursday
morning at 6 a.m. it was observed that the clock was fast by 10
minutes. When did the clock show the correct time during this
period?
1. Wednesday 6 a.m. 2. Tuesday 6 p.m.
3. Tuesday 6 a.m. 4. Wednesday 6 p.m.

Super Brain Drill


1. In an industry, the raw materials and the finished goods are
transported on the conveyor belt. There are two conveyor belts, one
for carrying parts from point P to point Q and another for carrying
parts from point R to point Q. P, Q and R in that order are in
straight line. Sometimes, the belt serves to transport cart, which
can themselves move with respect to the belts. The two belts move
at a speed of o.5 m/s and the cart moves at a speed of 2 m/s with
respect to the belts. A cart goes from point P to R and back to P

CREA – All Rights Reserved I U Training Services 209


Taking the Time to Work on Distances Number Math

taking a total of 64 s. Find the distance PR in meters. Assume that


the time taken by the cart to turn back at R is negligible?
1. 60 m 2. 120 m 3. 90 m 4. 36 m

2. P and Q travels from D to A and break journey at M in between.


Somewhere between D and M, P asks ―How far have we travelled?‖ Q
answers, ―Half as far as the distance from here to M.‖ Somewhere
between M and A, exactly 300 km from the point where P asks the
first question, he again asks, ―How far have we to go?‖ Q replies,
―Half as far as the distance from M to here.‖ The distance between
D and A is:
1. 250 km 2. 350 km 3. 450 km 4. 500 km

4. From a point P, on the surface of radius 3 cm, two cockroaches A


and B started moving along two different circular paths, each having
the maximum possible radius, on the surface of the sphere, that lie
in two different planes which are inclined at an angle of 45 degree
to each other. If A and B take 18 sec and 6 sec respectively, to
complete one revolution along their respective circular paths, then
after how much time will they meet again, after they start from P?
1. 27 s 2. 24 s 3. 18 s 4. 9 s

5. A, B and C start simultaneously from X to Y, turns back and meet


B at a distance of 11 km from Y. B reached Y, turns back and meets C
at a distance of 9 km from Y. If the ratio of the speeds of A and C
is 3:2, what is the distance between X and Y?
1. 99 m 2. 100 m 3. 120 m 4. 142 m

6. Three athletes A, B and C run a race, B finished 24 m ahead of C


and 36 m ahead of A, while C finished 16 m ahead of A. If each
athlete runs the entire distance at their respective constant speeds,
what is the length of the race?
1. 108 m 2. 90 m 3. 80 m 4. 96 m

CREA – All Rights Reserved I U Training Services 210


Taking the Time to Work on Distances Number Math

7. A man driving his bike at 24 kmph reaches his office 5 min late.
Had he driven 25% faster on an average he would have reached 4
min earlier than the scheduled time. How far is his office?
1. 24 km 2. 72 km 3. 18 km 4. Data
Insufficient

CREA – All Rights Reserved I U Training Services 211


Let’s End the Suspense

CREA – All Rights Reserved I U Training Services 212


End of Suspense English Comprehension

ENGLISH COMPREHENSION ANSWERS

Brain Drill Answers


Chapter 1: READING COMPREHENSION

Brain Drill 1

1. C 6.A 11. A 16. B 21. E

2. C 7. C 12. B 17. A 22. A

3. A 8. B 13. C 18. E 23. B

4. E 9. D 14. B 19. A 24. D

5. D 10. B 15. D 20. B 25. C

Brain Drill 2

1. B 6. B 11. A 16. B 21. B

2. C 7. B 12. B 17. D 22. D

3. E 8. C 13. D 18. C 23. D

4. C 9. B 14. E 19. A 24. A

5. D 10. A 15. E 20. A 25. E

Brain Drill 3

1. A 6. D 11. B 16. C 21. D

2. B 7. B 12. A 17. B 22. D

3. D 8. E 13. D 18. D 23. B

4. D 9. C 14. E 19. E 24. A

5. E 10. D 15. D 20. A 25. C

CREA – All Rights Reserved I U Training Services 213


End of Suspense English Comprehension

Super Brain Drill Answers

Chapter 2: READING COMPREHENSION


1.C 6. E 11. C 16. C 21. D

2B 7. E 12. C 17. B 22. E

3. A 8. E 13. E 18. E 23. B

4. C 9. A 14. A 19. A 24. A

5. D 10. D 15. C 20. B 25. B

CREA – All Rights Reserved I U Training Services 214


End of Suspense Logical Thinking

LOGICAL ABILITY ANSWERS


Chapter 1 – ANALYTICAL PUZZLES

1-c 14-a 27-d 40-d 53-d 66-e 79-c


2-b 15-d 28-c 41-a 54-d 67-b 80-b
3-a 16-c 29-c 42-c 55-b 68-a 81-c
4-c 17-d 30-c 43-a 56-e 69-a 82-d
5-a 18-c 31-d 44-b 57-a 70-b 83-b
6-b 19-d 32-a 45-e 58-c 71-a 84-c
7-a 20-a 33-b 46-c 59-d 72-e 85-a
8-b 21-c 34-b 47-e 60-c 73-c 86-d
9-b 22-d 35-d 48-c 61-b 74-b 87-d
10-a 23-d 36-c 49-e 62-a 75-a 88-a
11-b 24-d 37-b 50-a 63-a 76-a 89-c
12-d 25-a 38-d 51-d 64-c 77-c 90-e
13-c 26-d 39-d 52-b 65-a 78-b 91-d
92-e

Chapter 2 – READY TO GET STUMPED


1. THE RETURNING EXPLORER: Is there any other point on the
globe, besides the North Pole, from which you could walk a mile south,
a mile east, and a mile north and find yourself back a t the starting
point? Yes indeed; not just one point but an infinite number of them!
You could start from any point on a circle drawn around the South Pole
a t a distance slightly more than 1 + 1 / 2 ~ miles (about 1.16 miles)
from the Pole- the distance is "slightly more" to take into account the
curvature of the earth. After walking a mile south, your next walk of
one mile east will take you on a complete circle around the Pole, and

CREA – All Rights Reserved I U Training Services 215


End of Suspense Logical Thinking

the walk one mile north from there will then return you to the starting
point. Thus, your starting point could be any one of the infinite number
of points on the circle with a radius of about 1.16 miles from the
South Pole. But this is not all. You could also start a t points closer to
the Pole, so that the walk east would carry you just twice around the
Pole, or three times, and so on.

2. THE EARLY COMMUTER: The commuter has walked for 55


minutes before his wife picks him up. Since they arrive home 10
minutes earlier than usual, this means that the wife has chopped 10
minutes from her usual travel time to and from the station, or five
minutes from her travel time to the station. It follows that she met
her husband five minutes before his usual pick-up time of five o'clock,
or at 4:55. He started walking at four; therefore he walked for 55
minutes. The man's speed of walking, the wife's speed of driving and
the distance between home and station are not needed for solving the
problem. If you tried to solve it by juggling figures for these
variables, you probably found the problem exasperating

CREA – All Rights Reserved I U Training Services 216


End of Suspense Logical Thinking

3. THE COUNTERFEIT COINS: The counterfeit stack can be


identified by a single weighing of coins. You take one coin from the
first stack, two from the second, and three from the third and so on
to the entire 10 coins of the tenth stack. You then weigh the whole
sample collection on the pointer scale. The excess weight of this
collection, in number of grams, corresponds to the number of the
counterfeit stack. For example, if the group of coins weighs seven
grams more than it should, then the counterfeit stack must be the
seventh one, from which you took seven coins (each weighing one gram
more than a genuine half-dollar). Even if there had been an eleventh
stack of ten coins, the procedure just described would still work, for
no excess weight would indicate that the one remaining stack was
counterfeit.

4. TWO FERRY BOATS: When the ferryboats meet for the first
time [see top of Fig.], the combined distance traveled by the boats is
equal to the width of the river. When they reach the opposite shore,
the combined distance is twice the width of the river; and when they
meet the second time [bottom of Fig.], the total distance is three
times the river's width. Since the boats have been moving a t a
constant speed for the same period of time, it follows that each boat
has gone three times as far as when they first met and had traveled a
combined distance of one river-width. Since the white boat had
traveled 720 yards when the first meeting occurred, its total distance
at the time of the second meeting must be 3 X 720, or 2,160 yards.
The bottom illustration shows clearly that this distance is 400 yards
more than the river's width, so we subtract 400 from 2,160 to obtain
1,760 yards, or one mile, as the width of the river. The time the boats
remained at their landings does not enter into the problem. The
problem can be approached in other ways. Many students solved it as
follows. Let x equal the river-width. On the first trip the ratio of
distances traveled by the two boats is x - 720: 720. On the second
trip it is 2x - 400: x + 400. These ratios are equal, so it is easy to

CREA – All Rights Reserved I U Training Services 217


End of Suspense Logical Thinking

solve for x. (The problem appears in Sam Loyd's Cyclopedia o f


Puzzles, 1914, page 80.)

5. THE EFFICIENT ELECTRICIAN: On the top floor the electrician


shorted five pairs of wires (the shorted pairs are connected by
broken lines in the Figure), leaving one free wire. Then he walked to
the basement and identified the lower ends of the shorted pairs by
means of his "continuity tester." He labeled the ends as shown, and
then shorted them in the manner indicated by the dotted lines. Back
on the top floor, he removed all the shorts but left the wires twisted
at insulated portions so that the pairs were still identifiable. He then
checked for continuity between the free wire (which he knew to be
the upper end of F) and some other wire. When he found the other
wire, he was able at once to label it E2 and to identify its mate as E1.
He next tested for continuity between E1 and another end which,
when found, could be marked D2 and its mate D1. Continuing in this

CREA – All Rights Reserved I U Training Services 218


End of Suspense Logical Thinking

fashion, the remaining ends were easily identified. The procedure


obviously works for any odd number of wires.

6. THE AMOROUS BUGS: At any given instant the four bugs form
the corners of a square which shrinks and rotates as the bugs move
closer together. The path of each pursuer will therefore at all times
be perpendicular to the path of the pursued. This tells us that as A,
for example, approaches B, there is no component in B's motion which
carries B toward or away from A. Consequently A will capture B in the
same time that it would take if B had remained stationary. The length
of each spiral path will be the same as the side of the square: 10
inches. If three bugs start from the corners of an equilateral triangle,
each bug's motion will have a component of 1/2 (the cosine of a 60-
degree angle is 1/2) its velocity that will carry it toward its pursuer.
Two bugs will therefore have a mutual approach speed of 3/2 velocity.
The bugs meet at the center of the triangle after a time interval
equal to twice the side of the triangle divided by three times the
velocity, each tracing a path that is 2/3 the length of the triangle's
side.

CREA – All Rights Reserved I U Training Services 219


End of Suspense Logical Thinking

7. THE FLIGHT AROUND THE WORLD: Three airplanes are quite


sufficient to ensure the flight of one plane around the world. There
are many ways this can be done, but the following seems to be the
most efficient. I t uses only five tanks of fuel, allows the pilots of two
planes sufficient time for a cup of coffee and a sandwich before
refueling a t the base, and there is a pleasing symmetry in the
procedure. Planes A, B and C take off together. After going 1/8 of the
distance, C transfers 1/4 tank to A and 1/4 to B. This leaves C with
1/4 tank; just enough to get back home. Planes A and B continue
another 1/8 of the way, then B transfers 1/4 tank to A. B now has 1/2
tank left, which is sufficient to get him back to the base where he
arrives with an empty tank. Plane A, with a full tank, continues until it
runs out of fuel 1/4 of the way from the base. It is met by C which
has been refueled a t the base. C transfers 1/4 tank to A, and both
planes head for home. The two planes run out of fuel 1/8 of the way
from the base, where they are met by refueled plane B. Plane B
transfers 1/4 tank to each of the other two planes. The three planes
now have just enough fuel to reach the base with empty tanks.

CREA – All Rights Reserved I U Training Services 220


End of Suspense Logical Thinking

The entire procedure can be diagramed as shown in Figure, where


distance is the horizontal axis and time the vertical axis. The right
and left edges of the chart should, of course, be regarded as joined.

8. THE COLLIDING MISSILES: The two missiles approach each


other with combined speeds of 30,000 miles per hour, o r 500 miles
per minute. By running the scene backward in time we see that one
minute before the collision the missiles would have to be 500 miles
apart.

9. MONKEY WITH COCONUTS: 15 coconuts

10. CROSSING THE DESERT: The following analysis of the desert-


crossing problem appeared in a recent issue of E u ~ e k a , a
publication of mathematics students a t the University of Cambridge.
Five hundred miles will be called a "unit"; gasoline sufficient to take
the truck 500 miles will be called a "load"; and a "trip" is a journey of
the truck in either direction from one stopping point to the next.
Two loads will carry the truck a maximum distance of 1 and 1/3 units.
This is done in four trips by first setting up a cache a t a spot 1/3 unit
from the start. The truck begins with a full load, goes to the cache,
leaves 1/3 load, returns, picks up another full load, arrives a t the
cache and picks up the cache's 1/3 load. I t now has a full load,
sufficient to take it the remaining distance to one unit.
Three loads will carry the truck 1 and 1/3 plus 1/5 units in a total of
nine trips. The first cache is 1/5 unit from the start. Three trips put
6/5 loads in the cache. The truck returns, picks up the remaining full
load and arrives at the first cache with 4/5 load in its tank. This,
together with the fuel in the cache, makes two full loads, sufficient to
carry the truck the remaining 1 and 1/3 units, as explained in the
preceding paragraph.
We are asked for the minimum amount of fuel required to take the
truck 800 miles. Three loads will take it 766 and 2/3 miles (1 and 1/3
plus 1/5 units), so we need a third cache at a distance of 33 and 1/3
miles (1/15 unit) from the start. In five trips the truck can build up

CREA – All Rights Reserved I U Training Services 221


End of Suspense Logical Thinking

this cache so that when the truck reaches the cache a t the end of
the seventh trip, the combined fuel of truck and cache will be three
loads. As we have seen, this is sufficient to take the truck the
remaining distance of 766 and 2/3 miles. Seven trips are made
between starting point and first cache, using 7/15 load of gasoline.
The three loads of fuel that remain are just sufficient for the rest of
the way, so the total amount of gasoline consumed will be 3 and 7/15,
or a little more than 3.46 loads. Sixteen trips are required.
Proceeding along similar lines, four loads will take the truck a distance
of 1 and 1/3 plus 1/5 plus 1/7 units, with three caches located a t the
boundaries of these distances. The sum of this infinite series diverges
as the number of loads increases; therefore the truck can cross a
desert of any width. If the desert is 1,000 miles across, seven caches,
64 trips and 7.673 loads of gasoline are required.

11. PROFESSOR ON THE ESCALATOR: Let n be the number of


steps visible when the escalator is not moving, and let a unit of time be
the time it takes Professor Slapenarski to walk down one step. If he
walks down the down-moving escalator in 50 steps, then n – 50 steps
have gone out of sight in 50 units of time. It takes him 125 steps to
run up the same escalator, taking five steps to every one step before.
In this trip, 125 - n steps have gone out of sight in 125/5, or 25, units
of time. Since the escalator can be presumed to run a t constant
speed, we have the following linear equation that readily yields a value
for n of 100 steps: (n-50)/50 = (125-n)/25

12. THE ABSENT-MINDED TELLER: To determine the value of


Brown's check, let x stand for the dollars and y for the cents. The
problem can now be expressed by the following equation: 100y + x - 5 =
2 (100x + y). This reduces to 98y - 199x = 5, a Diophantine equation
with an infinite number of integral solutions. A solution by the
standard method of continued fractions gives as the lowest values in
positive integers: x = 31 and y = 63, making Brown's check $31.63. This
is a unique answer to the problem because the next lowest values are:
x = 129, y = 262, which fails to meet the requirement that y be less

CREA – All Rights Reserved I U Training Services 222


End of Suspense Logical Thinking

than 100. There is a much simpler approach to the problem and many
readers wrote to tell me about it. As before, let x stand for the
dollars on the check, 21 for the cents. After buying his newspaper,
Brown has left 22 + 2y. The change that he has left, from the x cents
given him by the cashier, will be x - 5.
We know that y is less than 100, but we don't know yet whether i t is
less than 50 cents. If it is less than 50 cents, we can write the
following equations: 2x = y and 2y = x – 5
If y is 50 cents or more, then Brown will be left with an amount of
cents (2y) that is a dollar or more. We therefore have to modify the
above equations by taking 100 from 2y and adding 1 to 2x. The
equations become: 2x + 1 = y and 2y + 100 = x – 5
Each set of simultaneous equations is easily solved. The first set gives
x a minus value, which is ruled out. The second set gives the correct
values.

13. WHITE, BLACK AND BROWN: The assumption that the "lady"
is Jean Brown, the stenographer, quickly leads to a contradiction. Her
opening remark brings forth a reply from the person with black hair,
therefore Brown's hair cannot be black. I t also cannot be brown, for
then it would match her name. Therefore it must be white. This
'leaves brown for the color of Professor Black's hair and black hair
Professor White. But a statement by the person with black hair
prompts an exclamation from White, so they cannot be the same
person.
I t is necessary to assume, therefore, that Jean Brown is a man.
Professor White's hair can't be white (for then it would match his or
her name), nor can it be black because he (or she) replies to the black-
haired person. Therefore it must be brown. If the lady's hair isn't
brown, then Professor White is not a lady. Brown is a man, so
Professor Black must be the lady. Her hair can't be black or brown, so
she must be a platinum blonde.

14. THE PLANE IN THE WIND: Since the wind boosts the plane's
speed from A to B and retards it from B to A, one is tempted to

CREA – All Rights Reserved I U Training Services 223


End of Suspense Logical Thinking

suppose that these forces; balance each other so that total travel
time for the combined flights will remain the same. This is not the
case, because the time during which the plane's speed is boosted is
shorter than the time during which it is retarded, so the over-all
effect is one of retardation. The total travel time in a wind of
constant speed and direction, regardless of the speed or direction, is
always greater than if there were no wind.

15. WHAT PRICE PETS: Let x be the number of hamsters originally


purchased and also the number of parakeets. Let y be the number of
hamsters among the seven unsold pets. The number of parakeets
among the seven will then be 7 - y. The number of hamsters sold (at a
price of $2.20 each, which is a markup of 10 per cent over cost) will
be x - y, and the number of parakeets sold (at $1.10 each) will be x - 7
+ y.
The cost of the pets is therefore 2x dollars for the hamsters and x
dollars for the parakeets - a total of 3x dollars. The hamsters that
were sold brought 2.2 (x - y) dollars and the parakeets sold brought
1.1 (x - 7 + y) dollars, that is a total of 3.3 x - 1.1y - 7.7 dollars
We are told that these two totals are equal, so we equate them and
simplify to obtain the following Diophantine equation with two integral
unknowns:
3x = 11y + 77
Since x and y are positive integers and y is not more than 7, it is a
simple matter to try each of the eight possible values (including zero)
for y to determine which of them makes x also integral. There are only
two such values: 5 and 2. Each would lead to a solution of the problem
were it not for the fact that the parakeets were bought in pairs. This
eliminates 2 as a value for y because it would give x (the number of
parakeets purchased) the odd value of 33. We conclude therefore
that y is 5.
A complete picture can now be drawn. The shop owner bought 44
hamsters and 22 pairs of parakeets, paying altogether $132 for them.
He sold 39 hamsters and 21 pairs of parakeets for a total of $132.
There remained five hamsters worth $11 retail and two parakeets

CREA – All Rights Reserved I U Training Services 224


End of Suspense Logical Thinking

worth $2.20 retail - a combined value of $13.20, which is the answer


to the problem.

16. BEER SIGNS ON THE HIGHWAY: The curious thing about the
problem of the Flatz beer signs is that it is not necessary to know the
car's speed to determine the spacing of the signs. Let x be the
number of signs passed in one minute. In an hour the car will pass 60x
signs. The speed of the car, we are told, is 10x miles per hour. In 10x
miles it will pass 60x signs, so in one mile it will pass 60x/10x, or 6,
signs. The signs therefore are 116 mile, or 880 feet, apart.

17. TRICKY TRACK: There is space only to suggest the procedure by


which it can be shown that Washington High won the high jump event
in the track meet involving three schools. Three different positive
integers provide points for first, second and third place in each event.
The integer for first place must be a t least 3. We know there are a t
least two events in the track meet, and that Lincoln
High (which won the shot-put) had a final score of 9, so the integer
for first place cannot be more than 8. Can it be 8? No, because then
only two events could take place and there is no way that Washington
High could build up a total of 22 points. I t cannot be 7 because this
permits no more than three events, and three are still not sufficient
to enable Washington High to reach a score of 22. Slightly more
involved arguments eliminate 6, 4 and 3 as the integer for first place.
Only 5 remains as a possibility. If 5 is the value for first place, there
must be a t least five events in the meet. (Fewer events are not
sufficient to give Washington a total of 22, and more than five would
raise Lincoln's total to more than 9.) Lincoln scored 5 for the shot-
put, so its four other scores must be 1. Washington can now reach 22
in only two ways:
4, 5, 5, 5, 3 or 2, 5, 5, 5, 5.
The first is eliminated because it gives Roosevelt a score of 17, and we
know that this score is 9. The remaining possibility gives Roosevelt a
correct final tally, so we have the unique reconstruction of the scoring
shown in the Table

CREA – All Rights Reserved I U Training Services 225


End of Suspense Logical Thinking

Events 1 2 3 4 5 Score
Washington 2 5 5 5 5 22
Lincoln 5 1 1 1 1 9
Roosevelt 1 2 2 2 2 9
Washington High won all events except the shot-put, consequently it
must have won the high jump.

18. HOW DID KANT SET HIS CLOCK? Immanuel Kant calculated
the exact time of his arrival home as follows. He had wound his clock
before leaving, so a glance at its face told him the amount of time
that had elapsed during his absence. From this he subtracted the
length of time spent with Schmidt (having checked Schmidt's hallway
clock when he arrived and again when he left). This gave him the total
time spent in walking. Since he returned along the same route, at the
same speed, he halved the total walking time to obtain the length of
time it took him to walk home. This added to the time of his departure
from Schmidt's house gave him the time of his arrival home.

19. A TENNIS MATCH: Whoever served first, served five games,


and the other player served four. Suppose the first server won x of
the games she served and y of the other four games. The total
number of games lost by the player who served them is then 5 - x + y.
This equals 5 [we were told that the non-server won five games],
therefore x = y, and the first player won a total of 2x games. Because
only Miranda won an even number of games, she must have been the
first server.

Chapter 3 – GET YOUR FIGURES RIGHT!

Brain Drill Answers


1. The minimum number is 5 and they should weigh 1,3,9,27 and 81 kg.
2. 100 3. 50% 4. 8 5. 15 6. 45 & 54 7. 71842
8. 30 min 9. 66s 10. 18 9. 18(12 female and 6 male)
10. 100 11. father-73, son-37 12. 8 14. 6

CREA – All Rights Reserved I U Training Services 226


End of Suspense Logical Thinking

Super Brain Drill Answers

1. Four brothers and three sisters


2. The difference in age is still 23 years, so I must be 23 if my father
is twice as old.
3. Rs. 200
4. The problem is simpler than it looks. The cyclists took 6 hours to
meet. The drone travelled 6 x 30 = 180 miles.
5. 2002
6. 9+8+7+65+4+3+2+1 = 99
7. 2.5 times ( 5/2, not 6/2)
8. From the church tower to the railway station is 1/3-1/4 = 1/12 of
the way. Krishna walks it in 5 minutes, so the whole trip takes 1 hour.
A quarter of this is 15 minutes. So he leaves his house at 7:15 and
reaches school at 8:15.
9. Three-quarter‘s of a kitten is one-quarter of Vaibhav‘s kittens. He
has 4 X ¾ = 3 kittens.
10. Step 1: Split the 180 ounces between two pans of the balance.
Each will hold 90 ounces.
Step 2: Split one 90-ounce batch into 45 and 45 ounces, again
using the balance without the weights.
Step 3: Remove 5 ounces from the 45 ounce batch by
balancing against the two weights. Now you have 40 ounces
for one package, and the rest of the grits is 140 ounces for
the second package.
11. No invention can save 100% on fuel, since energy cannot arise from
nothing. The correct calculation is not 30%+45%+25%=100%, but:
100%-(100%-30%)(100%-45%)(100%-25%) = 71.125%
12. One box has the balls arranged 3 X 3 X 3, the other 4 X 4 X 4.
The diameter of a large ball, then, is 4/3 that of the smaller ball. Its
volume, therefore weight, is 64/27 that of the smaller ball. There are
as 27/64 as many large balls, so the 2 boxes weigh the same.
13. The difference between A‘s and B‘s ages is a multiple of 9 ranging
from 0 to 91 – 19 = 72. C is 0, 4.5, 9, 13.5,… 36. Since 10 times his age

CREA – All Rights Reserved I U Training Services 227


End of Suspense Logical Thinking

is a two-digit number, he is 4.5 or 9. But if he is 9, B is 90 and A is 09


or 9, which contradicts the conditions of the problem. Thus C is 4.5, B
is 45 and A is 54.
14. Before the oldest gave half his apples to his brothers he had 16;
the middle and youngest had 4 each. Before the middle brother
divided his apples he had 8; this means the oldest had 16 – ½ (4) = 14
and the youngest had 2. Before the youngest divided his apples he had
4; the middle brother had 8 – ½ (2) = 7, and the oldest 13. The
youngest brother is 7, the middle brother is 10 and the oldest 16.
15. Her mother was right. To average 20 pages per day, Aaina would
have had to type the second half of the manuscript in no time.

Chapter 4 – SELF ASSESSMENT OPPORTUNITY

1-1 2-4 3-2 4-2 5-3


6-1 7-1 8-2 9-3 10-1
11-3 12-4 13-2 14-1 15-4
16-1 17-3 18-4 19-1 20-2
21-1 22-3 23-1 24-2 25-1
26-3 27-4 28-2 29-1 30-3
31-4 32-1 33-4 34-1 35-1
36-4 37-4 38-1 39-3 40-1

CREA – All Rights Reserved I U Training Services 228


End of Suspense Number Math

Brain Drill Answers


Chapter 1 – IMPROVING YOUR PERCENTAGES
1. 3 2. 2 3. 4 4. 4 5. 4 6. 4 7. 2 8. 1 9. 1
10. 2 11. 2 12. 3 13. 3 14. 4 15. 2 16. 2 17. 1 18. 2
19. 1 20. 1 21. 4 22. 2 23. 3 24. 3 25. 1 26. 1 27. 1
28. 3 29. 4 30. 2 31. 2 32. 3 33. 1 34. 1 35. 2 36.3
37. 4 38. 3
Chapter 2 - GAIN AND LOSS
1. 4 2. 1 3. 4 4. 4 5. 3 6. 4 7. 2 8. 3 9. 3
10. 2 11. 4 12. 1 13. 4 14. 2 15. 3 16. 2 17. 1 18. 4
19. 2 20. 2 21. 2 22. 2 23. 2 24. 3 25. 2 26. 1 27.3
28. 29. 2 30. 4 31. 2 32. 3
Chapter 3 – AVERAGES AND QNQ OF MIXTURES
1. 2 2. 3 3. 2 4. 2 5. 1 6. 4 7. 4 8. 4
9. 1 10. 3 11. 2 12. 4 13. 1 14. 2 15. 2 16.4 17. 3
18. 2 19. 1 20. 3 21. 1 22. 2 23. 4 24. 2 25. 2 26. 1
27. 1 28. 4 29. 2 30.1 31.1 32. 3 33. 4 34. 3
Chapter 4 – PERMUTATIONS AND COMBINATIONS
Permutations
1. 2 2. 1 3. 4 4. 3 5. 4 6. 3 7. 2 8. 4
9. 1 10. 2 11. 3 12. 2 13. 2 14. 1 15. 4
Combinations
1. 4 2. 3 3. 2 4. 3 5. 4 6. 3 7. 1 8. 3
9. 4 10. 1 11. 2 12. 4 13. 1 14. 2 15. 3 16. 1
Chapter 2 – TAKING TIME TO WORK THE DISTANCES
1. 2 2. 4 3. 2 4. 3 5. 2 6. 2 7. 1 8. 3 9. 1
10. 3 11. 1 12. 4 13. 1 14. 2 15. 3 16. 2 17. 1 18. 1
19. 2 20. 3 21. 1 22. 3 23. 2 24. 1 25. 3 26.3 27.4
28.4 29.2 30.2 31.3 32.1 33.4 34.1 35.1 36.1
37.4 38.2 39.1 40.2 41.1 42.2 43.4 44.3 45.2
46.4 47.2

CREA – All Rights Reserved I U Training Services 229


End of Suspense Number Math

Super Brain Drill Answers

Chapter 1 – IMPROVING YOUR PERCENTAGES


1. 3 2. 4 3. 1 4. 3 5. 2 6. 3 7. 4

Chapter 2 - GAIN AND LOSS


1. 1 2. 4 3. 1 4. 4 5. 2 6. 3 7. 4
8. 4 9. 2 10. 1

Chapter 3 – AVERAGES AND QNQ OF MIXTURES


1. 3 2. 1 3. 4 4. 4 5. 3 6. 3 7. 2
8. 2 9. 4

Chapter 4 – PERMUTATIONS AND COMBINATIONS


1. 1 2. 4 3. 2 4. 3 5. 1 6. 1 7. 3 8. 2
9. 4 10. 3 11. 3 12. 3 13. 2 14. 2

Chapter 5 – TAKING TIME TO WORK THE DISTANCES


1. 1 2. 3 3. 3 4. 4 5. 1 6. 4 7. 3

For online tests and more questions visit our


website: www.iusupport.in
For queries on solutions mail us at
contact@iusupport.in

CREA – All Rights Reserved I U Training Services 230

You might also like